Версия для печати темы

Нажмите сюда для просмотра этой темы в обычном формате

Диалог специалистов АВОК _ Беседка _ Взлетит или не взлетит?

Автор: Bers 3.11.2006, 7:20

Есть такая задачка:

"Самолет (реактивный или винтовой) стоит на взлетной полосе с подвижным покрытием (типа транспортера). Покрытие может двигаться против направления взлета самолета, т.е. ему навстречу. Оно имеет систему управления, которая отслеживает и подстраивает скорость движения полотна таким образом, чтобы скорость вращения колес самолета была равна скорости движения полотна.
Внимание, вопрос: сможет ли самолет разбежаться по этому полотну и взлететь?"


Видел эту задачку на нескольких форумах и практически нигде не пришли к единому мнению. Кстати, на одном форуме вопрос обсуждается уже полгода... Но поскольку у нас форум, так сказать, с инженерным уклоном и люди здесь сплошь технически подкованные, то верное решение задачки найдется быстро. А может и не найдется вовсе...

Ну дык что - Взлетит или не взлетит? Есть версии?

Автор: e.o.l. 3.11.2006, 7:55

а чего ему взлетать то, воздух что-ли навстречу вместе с дорожкой движется? cool.gif

Автор: Litvinov 3.11.2006, 8:05

Взлетит, если при определенном разгоне резко затормозить вращающееся полотно. Это как взлет с авианосца «на привязи», только там самолет стоит (привязан) под нагрузкой двигателей, а потом как на катапульте…(крючки падают). Но в любом случае самолету нужно пространство для падения в низ (принцип трамплина – прежде чем лететь нужно немного упасть- провалиться)
... ну и полоса взлетная нужна мало-мальская.
Т.е. дложно быть перед самолетом пространство впереди и внизу smile.gif
или если совсем с свободным местом плохо, то саму полосу «вращающеюся-взлетную» поставить под углом в небо (требуется расчет)

Автор: donk 3.11.2006, 8:37

Веселая задачка biggrin.gif .
Мне думается не взлетит, так как подъемная сила на крыле образуется за счет набегания потока воздуха при разбеге самолета, а не при его неподвижном состоянии и включенных двигателях. Скорость набегания потока при включенных двигателях и неподвижном состоянии самолета будет недостаточна для создания нужной подъемной силы, и самолет останется на дорожке.

2Litvinov: Не уверен, но вроде все авианосцы оборудованы катапультами, т.к. длины палубы все равно не хватает для уверенного взлета.
з.ы. и автоматика дорожки не даст ей резко затормозить bleh.gif .

Автор: Litvinov 3.11.2006, 9:07

Все правильно. Вопрос стоял как? сможет ли самолет разбежаться по этому полотну и взлететь?
Ответ: Самолет взлететь сможет (он для этого конструировался biggrin.gif)
А вот разбежаться по этому полотну – не сможет! Это не разбег, а импульс для разбега. По этому надо маломальскую взлетную палубу с трамплином и условие резкого торможения полотна.
Только зачем изобретать велотренажер, когда есть простая и проверенная конструкция катапульт на тех же авианосцах. И есть аппараты с вертикальным взлетом.

Автор: donk 3.11.2006, 9:08

я, конечно, не летчик, но вроде как закрылки нужны не для торможения - они нужны для изменения профиля крыла на нерасчетных полетных режимах - взлет/посадка. а для торможения самолета используются щитки (аэродинамические тормоза), реверс двигателей, и собственно колесные тормоза.

А тормозить на дорожке он не будет, имхо, потому что при выключении двигателей и пропадании тяги дорожка резко остановится и самолет останется неподвижен... наверное rolleyes.gif ...

Добавлено - 09:11
2Litvinov: Верно, то верно, но вот если он не сможет разбежаться, то и взлететь не сможет. А остальные варианты уже не подпадают под условие задачи wink.gif .

Автор: Litvinov 3.11.2006, 10:54

Еще есть Софизм – направление философии (подмена понятий). На заре цивилизаций многие софисты использовали свои способности для выигрыша во всяких спорах гражданско-процессуального и др. характера

Зы. без гравицапы пицилап не взлетит… biggrin.gif

Автор: Гость_Сергей Ружинский 3.11.2006, 11:02

Не взлетит!

Пока подьемная сила на крыле от НАБЕГАЮЩЕГО потока воздуха не превысит взлетную массу - будет стоять, только колесики будут крутиться.

Автор: Litvinov 3.11.2006, 12:38

Не поймите меня правильно... biggrin.gif
Хочется для «необыкновенной» задачи – найти необыкновенное решение.
e.o.l. – правильный вопрос задал (читай выше)…

вот если бы удалось организовать встречный поток воздуха (мечтательно).
Эдакая (локальная) аэродинамическая труба «без трубы» (оболочки громоздкой) в атмосфере на взлетном поле...rolleyes.gif

Или… срочно изобретать «гравицап» blink.gif

Автор: Иванов В.А. 3.11.2006, 12:45

А я думаю, самолет не будет стоять на этой взлётной полосе, поскольку у него привод не на колеса.
Реактивные и винтовые двигатели "отталкиваются" от воздуха, а не от покрытия. Мне кажется транспортер лишь немного замедлит движение.... ??

Автор: LordN 3.11.2006, 13:08

этот гребаный самолет пытается взлететь на каждом форуме, уже лет двадцать, если не быльше. странно что так много народу попалось на него smile.gif, но все когда-нить случается в первый раз.

читайте внимательно:

Цитата
Оно имеет систему управления, которая отслеживает и подстраивает скорость движения полотна таким образом, чтобы скорость вращения колес самолета была равна скорости движения полотна.
эта задачка из разряда обманок, в условие намерянно вводится фраза чтоб запутать. прочтите и не ломайте себе голову над этим бредом.
самолет летает на ездит, скорость вращения колес ВСЕГДА будет равна скорости полотна.

задача решается просто.
рассматривается два крайних случая:
1. полностью пренебрегают трением подшипников колес в шасси.
тогда самолет взлетит в любом случае

2. колеса наглухо заблокированы
тогда все будет зависить от трения колес о полотно и тяги двигателя. некоторые из совремённых самолетов имеют тягу превышающую вес самолета - сталбыть такой самолет полюбому взлетит после того как сожгет покрышки. если тяга меньше - может и сместа не стронуться.

Автор: AAANTOXA 3.11.2006, 13:33

Два крайних случая:
1 почему это он взлетит???? Напомню самолет летает за счет того что давление над крылом становится меньше давления под крылом. достигается это за счет профиля крыла. При этом обязательное условие - обдув профиля крыла набегающим потоком воздуха. При нашем условии обдува крыла НЕТ!!!!!! Турбина или винт самолета работают только на то чтобы КОЛЕСА кутились быстрее! И только! Самолет "НА МЕСТЕ СТОИТ" - только колеса крутятся. так что он не взлетит полюбому.
2 да действительно многие современные истребители имеют тягу больше своего веса. Теоретически, если поставить самолет на хвост и включить двигатели на форсаж, и при этом обеспечить пространство для отвода газов, то самолет взлетит. Но тутже завалится в любую сторону и упадет так как для устойчивого полёта нужна скорость-иннерция и обдув потоком.

так что ответ на первоначальный вопрос НЕТ!

Автор: Сергей В. 3.11.2006, 13:49

Взлетит!
Только колеса при этом будут вращаться быстрее (насколько это позволят подшипники колеса и привод покрытия).

Цитата
самолет не будет стоять на этой взлётной полосе, поскольку у него привод не на колеса.

Вот-вот.

Автор: Барабан 3.11.2006, 14:21

Кто-нибудь может мне объяснить на пальцах смысл задачи. :wacko:
Ничего не понимаю newconfus.gif

Какие то приводы, подшипники, тяги, закрылки, трения и катапульты.

Самолет стоит на транспортере. Так. Чем больше скорость колес самолета, тем больше скорость ленты транспортера.

То есть, если смотреть со стороны - самолет не движется???

Или я чего-то не догоняю???

Автор: Сергей В. 3.11.2006, 15:30

Стоп, стоп, стоп...
А что там в условии? Двигатели у самолета включены или нет?
Если включены - взлетит, если выключены - не взлетит.
По-моему так.

Автор: AAANTOXA 3.11.2006, 16:53

просьба к админам форума удалить эту бредовую тему- она заразна как пасьянс на кампе.
Сергей В. обьяснитека за счет чего он взлетит??????????по каким таким законам аэродинамики или гравитации??????
Блин куча конструкторов бьются решить задачу сделать самолет взлетающий сместа. А тут оказывается проблемма-то поставил самолет на катки (если кто ходил на вело секцию там зимой тренируются - велек на катки ставят и "едут") он и взлетает сместа . Гениально! Патентуйте быстрее и в Боинг продавайте это идею га га га га :-)))))

цитата Барабан "То есть, если смотреть со стороны - самолет не движется???"
- да черт побери ДА!!!

Автор: LordN 3.11.2006, 16:56

Цитата
цитата Барабан "То есть, если смотреть со стороны - самолет не движется???"
- да черт побери ДА!!!
с чего бы это вдруг он не двигался wink.gif ?

Автор: e.o.l. 3.11.2006, 17:21

Цитата
цитата Барабан "То есть, если смотреть со стороны - самолет не движется???"
- да черт побери ДА!!!

а если стоять на транспортерной ленте - ещё как ого-го laugh.gif

Автор: AAANTOXA 3.11.2006, 18:12

ооооо как все запущено
вообще конечно если учитывать что Земля вращается вокруг Солнца, а Солце вокруг еще бог знает чего то все мы сейчас движемся со страшной скоростью но почему-то не взлетаем

Автор: Сергей В. 3.11.2006, 18:24

Цитата
просьба к админам форума удалить эту бредовую тему- она заразна как пасьянс на кампе.

Вообще-то согласен.
Цитата
обьяснитека за счет чего он взлетит

Если двигатели самолета работают, то он будет двигаться, т.к. отталкивается он от воздуха, а не от взлетной полосы, ну нет у самолета привода на колеса.
Если двигатели не работают, то он будет стоять на месте, только колеса будут крутиться, причем, если пренебречь трением, то скорость колеса и транспортера достигнет скорости света после чего произойдет аннигиляция. biggrin.gif
А вообще подробно здесь:
http://ru.wikipedia.org/wiki/%D0%97%D0%B0%D0%B4%D0%B0%D1%87%D0%B0_%D0%BF%D1%80%D0%BE_%D1%81%D0%B0%D0%BC%D0%BE%D0%BB%D1%91%D1%82

Автор: AAANTOXA 3.11.2006, 19:21

Сергей В. откройте школьный учебник физики и прочтите за счет чего создается подъемная сила.

итак разжую и закроем тему. подробно: самолет стоит на полотне транспортёра. включаем движок, он начинает двигаться вперед. Автоматич начинает под ним двигаться полотно. Тоесть получается что относительно полотна самолет движется но вот относительно других обьектов самолет не двигается! Только колёса крутятся у него. И тем быстрее они крутятся чем мощьнее работает двигатель самолёта, тем быстрее движется полотно транспортёра. Самолет не взлетит при этих условиях. Если кто считает иначе обьясните за счет чего. По ссылке сознательно не пойду там такиеже умники сидят Циалковские блин.
Есть некоторые модели самолетов в которых частично ЧАСТИЧНО! используется принцип обдува воздухом или газами из турбин плоскостей крыла. Это укорачивает пробег но не исключает его полностью.

Скажите все кто считает что он (собака) взлетит почему же эту гениальную идею еще не воплотили в жизнь. Ведь я так думаю проще сделать транспортёр со скоростью полотна 300км/ч чем поворотные на 90град реактивные сопла.

Автор: Иванов В.А. 4.11.2006, 5:45

Если по такому транспортеру поедет велсипед или автомобиль с ДВС, то они действительно будут двигаться только относительно ленты транспортера, а не относительно окружения.
Но если встать на транспортер в роликовых коньках, натянуть над ним трос и двигаться за счет подтягивания руками за трос, то транспортер на ваше движение влияния оказывать не будет (до определенной скорости вращения). Так же двигается и самолет с реактивным двигателем - тросом для него является окружающий воздух.

Автор: LordN 4.11.2006, 14:58

еще раз, одного похоже было мало wink.gif

такие задачки обожают задавать в приёмных комиссиях физ-мат вузов и на различного рода олимпиадах.
такие задачки имеют бесконечное множество решений или, что тоже самое, решения не имеют в принципе.
основной смысл этой задачи - показать своё мышление, мат.аппарат коим владеешь и пользуешься и т.д. и т.п.
Антон, внимательнейшим образом вчитайтесь в эту фразу

Цитата
скорость вращения колес самолета была равна скорости движения полотна
вчитались? а теперь подумайте, сколько вариантов интерпретации фразы можно предложить сходу.

Автор: AAANTOXA 4.11.2006, 15:06

Согласен
он полетит!
а я больше не буду отвечать в этой теме - век зарплаты не видать!

Автор: zr84 4.11.2006, 15:25

Теоритически может взлететь. Нужно сделать численные оценки.
Вот почему:
В полосе которая движется, в силу вязкости воздуха образуется пограничный слой
воздуха с градиентом скоростей. Поэтому на верхней и нижней плоскостях крыла возникнет разность давления(в силу профиля крыла)- которая и составляет подъемную силу.
Вопрос в том, с какой скоростью должена двигаться полоса, чтобы обеспечить необходимую
разность давления для полета?

Автор: tiptop 4.11.2006, 16:06

Цитата(zr84 @ Nov 4 2006, 15:25 )
В полосе которая движется, в силу вязкости воздуха образуется пограничный слой воздуха с градиентом скоростей. Поэтому на верхней и нижней плоскостях крыла возникнет разность давления(в силу профиля крыла)- которая и составляет подъемную силу.

Да, вот если бы ещё, вдобавок, аэродинамическую трубу сзади поставить...

Автор: zr84 4.11.2006, 17:43

Цитата(tiptop @ Nov 4 2006, 16:06 )
Да, вот если бы ещё, вдобавок, аэродинамическую трубу сзади поставить...

Это мысль!
Заключить самолет в цилиндр(обечайку)-значительно увеличиваем шансы на взлет самолета.

Автор: kroudion 4.11.2006, 21:45

Bers, ришпект!
Поржал от души.
Типа ракета не может летать в безвоздушном пространстве ибо ей не от чего отталкиваться

Автор: Гость_Сергей Ружинский 5.11.2006, 11:50

НЕ ВЗЛЕТИТ, блин, все, баста smile.gif

Харьковский ордена Ленина Авиационный Институт им. Н.Е.Жуковского, выпуск - 1989 г., 1 фак. - самолетчик. smile.gif

А давайте еще и задачу про соплю запущенную в космос обсудим, - эту тему тоже уже давно на Форумах обсуждают, но там хоть смешно.

Автор: OVKT 5.11.2006, 12:06

Хм... Странная тема, попробую разобраться логически. Отвлечёмся от самолёта, представим себе человека бегущего по улице. При беге воздух в лицо и в ушах шумит. А что если этот человек бежит в зале по тренажёру имитирующему бег? Никакого ветра в лицо, никакого шума в ушах. Представим неподвижный транспортёр и стоящий на нём самолёт.
Транспортёр начинает двигаться с ускорением, набирая скорость с нуля. В это время самолёт включает двигатели и начинает перемещаться против движения транспортёра. Чем быстрее едет лента, тем быстрее ЕДЕТ самолёт, одновременно оставаясь неподвижным относительно воздуха. Можно даже открыть люк, а в ушах не шумит. И перегрузок никаких.
При чём здесь пример с тросом? Этот пример хорош для летящего самолёта, а здесь мы имеем дело с самолётом изначально стоящим на месте. Самолёт никуда не взлетит.
Взлететь в такой ситуации смогли только 2 персонажа: барон Мюнхгаузен и его Лошадь, которые вылезли из болота, когда барон потянул себя за косичку.

Автор: zr84 5.11.2006, 14:57

Взлететь-значит оторваться то Земли.
Если транспортер (ВПП-взлетно посадочная полоса) при своем движении назад обеспечит НАБЕГАЮЩИЙ ВЕТЕР НУЖНОЙ СКОРОСТИ для самолета( для отрыва чуть-чуть больше), то он оторвется(поднимется) от Земли. Все!!!
Вспомните картину лета- коршуна висящего на одном месте, на высоте, самолет подобно коршуну зависнет над полосой. Т.е., основная задача транспортера ОБЕСПЕЧИТЬ ВЕТЕР-и это возможно(теоретически). В качестве технических средств создающих ветер- экраны(пластины) установленные по полосе и перемещающиеся вместе в транспортером-они создадут ветер(отвлекаясь от вязкости и градиентов).
Гостю С.Ружинскому. Про "сопли"-тоже летают по баллистике. Такие задачи мозги разминают, чтобы не затвердели, и помогают освоить(понять) законы физики
Кстати, если заменить воздух на воду, то в принципе нет разницы "полета"- у самолета в воде(в силу большей вязкости) больше шансов "взлететь". И последнее.
Предлагаю администратору организовать подфорум по всяким-таким, хитрым задачкам по гидравлике и технической термодинамике -это в духе форума.

Автор: Prasolov 5.11.2006, 15:33

Забавно, черт подери!!
Подавляющее большинство ИНЖЕНЕРОВ считают, что самолет имеет привод на колеса.
Разумные, разжеванные до состояния манной каши доводы, которые приводит Иванов В.А. пропускаются.
Именно в этом и есть подвох задачки. Именно на это и купились технари с высшим образованием.
Типа, какого цвета стоп-кран в самолете.

Автор: LVV 6.11.2006, 9:51

Если учесть только факторы указанные в задаче то в целом согласен с zr84.
Разовью его мысль.
1. Самолет отталкивается от воздуха.
2. Следовательно воздух должен двигаться относительно самолета.
3. Раз воздух движется относительно самолета, то самолет взлетит.
Теперь рассмотрим за счет чего воздух будет двигаться. Первая мысль возникает о том, что воздух может двигаться за счет поверхностоного трения о транспортер.
Но кажется что при разумных скоростях движения необходимой скорости потока не создать. Но в этом и состоит "Стоп-кран". Чтобы обеспечить выполение задачи транспортер должен двигаться со скоростью достаточной для создания подъемной силы (в плоть до бесконечности). Естественно принимает что сами колеса и подипники колес выдержат действующие скорости, а их трение равно 0.

В реальности колеса имеют значительное сопротивление качению, сами подшипники также имеют сопротивление вращению. При возникновении момента трения возникнет проскальзывание между колесом и транспортером. Т.е. никогда скорость вращения колеса не будет равна скорости движения транспортера. А это уже грубое отклонение от условий задачи.

Автор: tiptop 6.11.2006, 10:50

Цитата(Prasolov @ Nov 5 2006, 15:33 )
Забавно, черт подери!!
Подавляющее большинство ИНЖЕНЕРОВ считают, что самолет имеет привод на колеса.

Вместо того, чтобы просто отмахнуться - "а, какая-то несуразная задача", люди рассматривают интересный гипотетический вариант - двигатели самолёта не работают, а именно колёса "имеют систему управления, которая отслеживает и подстраивает скорость их движения"...
Согласен с тем, что "такие задачи мозги разминают".

Автор: kroudion 6.11.2006, 11:00

Самое забавное, что после всех объяснений ЛордНа, Иванова, Прасолова, разжёванных донельзя, дискуссия продолжается. Лет на двадцать?
Ребята, вы здесь ещё не упомянули критерий Рейнольдса (за трение всё-таки он отвечает) и критерий Грасгофера, отвечающий как раз за подъёмную силу. А чо? колёса от трения нагреваются, воздух возле них тож, критерием Грасгофера устремляется вверх, создавая под крылом разрежение... Не взлетит, адназначна!

Автор: ncx 6.11.2006, 12:40

если вам мнения выпускника ХАИ мало то попробуйте мнение инженера-ракетчика:

НЕ ВЗЛЕТИТ

с ув.

Автор: tiptop 6.11.2006, 13:07

Цитата(ncx @ Nov 6 2006, 12:40 )
если вам мнения выпускника ХАИ мало то попробуйте мнение инженера-ракетчика:

НЕ ВЗЛЕТИТ

с ув.

Мало! Надо "попробовать" ещё мнение инженера-транспортёрщика... biggrin.gif
А космонавты что-нибудь скажут ?

Автор: kroudion 6.11.2006, 13:14

Поправка (от ncx): Грасгофа, а не Грасгофера (и возможно, что правильнее с двумя "с", лень смотреть).

Дурка продолжается clap.gif

Автор: Mitka 6.11.2006, 13:22

Ах-хренеть!
Вы чё, господа! Не надо прикрываться Авториетом ВУЗа!!!
Не позорьте Альмаматер! С каких пор в решении задач по физике участвуют ИМЕНА ВУЗов?
Дайте чёткое физико-математическое обоснование! (Хаёвцы, блин!)

Предлагагаю свою пародию на подобный метод решения математических споров:
"ПОЛЕТИТ, пАтАмУшА у мИнЯ папа - ПрАкУрОр!!!"

Без обид.

P.S. "Хаёвцы" - выпускники ХАИ.

Автор: ncx 6.11.2006, 14:36

блин
тупняк
вычеркивайте предыдущий мой пост

всмысле был не прав ашыпся

Автор: kroudion 6.11.2006, 20:32

Вот тоже интересные задачки, типа про самолет на транспортёре

Если в холодильнике закрыть включённый в сеть утюг, кто победит?!

Если я куплю холодильник, подключающийся к интернету, могу ли я быть уверенным, что с него не кормятся хакеры?

Интересно, а для холодильника порно - это стиральная машина без корпуса? или посудомойка в воде и мыльной пене?

Ответы опубликованы http://www.holod-konsultant.ru/cgi-bin/yabb/YaBB.pl?board=1;action=display;num=1094199656;start=30

Автор: Litvinov 6.11.2006, 20:40

clap.gif
нормаль-ные вопросы...

Автор: zr84 7.11.2006, 5:50

Еще вариант, самый простой?!
Дождаться сильного, встречного ветра-штормового (10-11 по шкале Ботфорта)-и в полет.
Место для скептиков в кабине есть. Поехали!

Автор: ncx 7.11.2006, 6:50

может мои разъяснения помогут тем кто ещё не понял

основной момент в рассуждениях тех кто считает что самолет НЕ взлетит это то что он останется неподвижен относительно окружающего воздуха
это не так
почему?
да потому, что между системами отсчета "самолет" и "транспортер" НЕТ СВЯЗИ
любое движение транспортера будет отлавливаться колесом шасси

мысленно проведите эксперимент: при неподвижном самолете и транспортере "дёрните" резко поверхность транспортера
что произойдёт?
самолет останется стоять, просто провернется его колесо
то же самое при движущемся самолете

движение траспортера НИКАК не влияет на движение самолета

конечно для технаря необходима оговорка: если пренебречь трением в подшипниках
но это уже мелочи

Автор: LVV 7.11.2006, 6:55

ncx Это уже верный ответ (ИМХО)

Автор: LordN 7.11.2006, 9:26

smile.gif со взлётом разобрались вроде..

предлагаю слегка изменить формулировку задачи и ответить на такой вопрос:
с какой скоростью будет двигаться транспортёр в системе отсчета "наблюдатель на земле"? wink.gif

Автор: ncx 7.11.2006, 11:22

эээээ
ну скажем с такой же скорость што и самаль токо обратно
?

Автор: LordN 7.11.2006, 11:34

Цитата
ну скажем с такой же скорость што и самаль токо обратно
smile.gif
а я думаю что оно не будет двигаться, т.к. условие
Цитата
скорость вращения колес самолета д.б. равна скорости движения полотна
соблюдается и так wink.gif

Автор: Гость_Igor 7.11.2006, 12:03

Цитата(ncx @ Nov 7 2006, 06:50 )
может мои разъяснения помогут тем кто ещё не понял

основной момент в рассуждениях тех кто считает что самолет НЕ взлетит это то что он останется неподвижен относительно окружающего воздуха
это не так
почему?
да потому, что между системами отсчета "самолет" и "транспортер" НЕТ СВЯЗИ
любое движение транспортера будет отлавливаться колесом шасси

мысленно проведите эксперимент: при неподвижном самолете и транспортере "дёрните" резко поверхность транспортера
что произойдёт?
самолет останется стоять, просто провернется его колесо
то же самое при движущемся самолете

движение траспортера НИКАК не влияет на движение самолета

конечно для технаря необходима оговорка: если пренебречь трением в подшипниках
но это уже мелочи

Есть связь. "Вращение колеса полностью компенсируется транспортером". Упростите задачу и рассмотрите две шестеренки. В этом случае одна закреплена на земле другая сверху. Посде этого попробуйте не катая верхнюю шестеренку по нижней сместить верхнюю хоть на миллиметр...

Автор: ncx 7.11.2006, 12:24

ой хаха
а ведь точно
во тупой во тупой biggrin.gif


ЭТОТ ПОСТ ДЛЯ ЛордН о)

Автор: OVKT 7.11.2006, 12:54

Сначала вопрос: где можно найти другие обсуждения самолётиков ???
Прикольнее было бы если лента шла бы по ходу движения самолёта.
Транспортёр и подтягивание канатом можно представить иначе как 2-х человек: 1-й на роликах и на транспортёре, а 2-й пешком на асфальте держит 1-го и толкает против движения ленты. Тогда скорость 1-го чела на роликах будет равна скорости толкания 1-м, и не зависеть от скорости ленты, а ролики будут крутиться с суммарной линейной скоростью.
Но... если 2-й ведущий чел остановится, то 1-й ведомый тоже остановится на месте, а ролики будут прокручиваться вхолостую.
Если же у самолёта вЫключить двигатель, то он НЕ остановится, а будет откатываться назад. С канатом пример НЕудачный: чел НЕ откатывается назад при отсутствии усилий.
Таким образом, на самолёте по условию задачи мы имеем дело с аналогией: ГОРКА_без_ТРЕНИЯ, угол горки заведомо равен углу, уравновешивающему силу тяжести и реактивную подъёмную силу со всякими тангенсами. Включен движок - есть работа по стоянию на месте, вЫключен - падает вниз.
Примечание: по той же причине нельзя улететь за пределы Солнечной системы без развития космических скоростей: упадём на Солнце.
Что и требовалось доказать: такой самолёт НИКУДА НЕ ПОЛЕТИТ !!!

Автор: Сергей В. 7.11.2006, 13:43

Предлагаю к этой теме добавить голосование. biggrin.gif
Варианты ответов:
1. Взлетит.
2. Не взлетит.
3. Взлетит, но низенько.
4. Не взлетит, а зароется в землю.
5. Не хочу отвечать на дурацкие вопросы (в смысле "делать вам всем нечего")

Автор: LordN 7.11.2006, 13:49

головалка прикручена.
голосуем фсе! clap.gif

Автор: ncx 7.11.2006, 13:52

Цитата(Гость_Igor @ Nov 7 2006, 12:03 )
Упростите задачу и рассмотрите две шестеренки. В этом случае одна закреплена на земле другая сверху. Посде этого попробуйте не катая верхнюю шестеренку по нижней сместить верхнюю хоть на миллиметр...

ой усложняете
шестерёнки какието

нарисуйте ваши шестеренки, все степени их свободы и все силы, действующие на систему и тогда поймёте возможно
что нижняя шестеренка не закреплена а движетса вместе с верхней

Автор: ncx 8.11.2006, 8:18

Цитата(LordN @ Nov 7 2006, 11:34 )
а я думаю что оно не будет двигаться, т.к. условие
Цитата
скорость вращения колес самолета д.б. равна скорости движения полотна
соблюдается и так wink.gif

я вот тут подумал Лев Тёзкыч
што для того штобы транспортер не двигалса то к ему надо движок подключать фсё таки

а в случае когда транспортер имеет возможность свободно перемещаца туда-сюда то он бует двигаца в ту же сторону што и самаль токо медленее

давайте представим што силы трения в точке соприкосновения нет ваще
какую картину мы тогда увидим?
самаль движеца, шасси не крутица, транспортер стоит

саэршенна очевидно што если трение всётаки есть то колесо шасси будет вращацца
пральна?
а если есть сила которая вращает колесо то есть такая жэ сила по величине но противоположная по направлению которая буэт действовать на второе тело то бишь транспортер
следовательно транспортер таки будет двигаца

а вот вопрос с какой скоростью мне чото неасилить
интуитивно кажетса што эта скорость будет зависеть от:
1. силы трения (вес самаля плюс к-нт трения)
2. момента инерции колеса шасси
3. массы транспортёра

Автор: kroudion 8.11.2006, 16:25

Изменим начальное условие. Пусть в начальный момент времени транспортёр движется со скоростью V0 (вместе с самолётом, который по-прежднему стоит жопой по направлению движения). А далее всё по тексту классической постановки вопроса.
Вопрос: при каком значении V0 самолёт уже не сможет взмыть в облака? Как при этом будет меняться скорость транспортёра V?
Предлагаю сразу начать голосование.

Автор: OVKT 9.11.2006, 19:34

Из программы "Время": Правительство РФ выделило 5 млрд.руб. на новый нац.проект "Улетит ли самолётик".
Вообще, это задачка по термеху для 18-летних студентов. Надо спросить на форуме DWG, там они наскребут 20 страниц ни о чём.
Докажу моё логическое решение ещё разок на бис: люди, утверждающие, якобы "улетит", основываются на том, что на самолёт не_влияет встречная лента. Допустим, скорость ленты 50м/с, скорость самолёта 20м/с. По мысли этих теоретиков, самолётик не зависящий от ленты, якобы улетит произвольно. Вопрос только... куда улетит?!
Предположим, есть ракета, направленная вертикально вверьх. Никакой ленты транспортёра, только гравитация. Если подъёмная сила больше силы тяжести, ракета подымается. Если уравновесить силы - зависает на месте относительно воздуха, т.к. воздух ей и не нужен, ракета преодолевает гравитацию так же как наш несчастный самолётик преодолевает "имитацию гравитации". Ежели у ракеты выключить двигатель, то она вовсе не зависнет в воздухе, как чел в "примере с канатом", а будет падать. Чтобы не падать, нужно тратить горючее на зависание на месте, отталкиваясь хоть от воздуха пропеллером, хоть реактивно.
Исходное условие задачи именно об этом: самолётик тратит энергию на зависание на месте, относительно воздуха остаётся неподвижным, прекратив движение - откатится назад. См. начало: если лента движется гораздо быстрее.
Любое движение быстрее "ленты" является нарушением условий задачи.
Что и требовалось доказать в очередной раз: НЕТ, НЕ УЛЕТИТ, если соблюдать исходное условие задачи.

Автор: Giedi Prime 9.11.2006, 22:30

Г-но вопрос! Нихрена он не взлетит.

Автор: gh5rjn 9.11.2006, 23:28

Бесплатно не полетит. А вот за деньги - запросто, хоть до Китая.
ЗЫ. Тема отката не раскрыта.

Автор: ncx 10.11.2006, 7:58

Цитата(Mark-off @ Nov 9 2006, 23:20 )
Хех! Я думал-заглохло. А если так: на одной оси два шкива, произвольно вращающихся. Две доски домиком с углом градусов 60. На каждой - продольный желоб  Две веревки, на концах каждой - колесо. Оба равны по весу и форме. Ставим каждое колесо в свой желоб. Веревки пропускаем по шкивам и прокидываем по желобам, противоположным привязанным колесам. Таким образом, каждое колесо будет стоять на веревке другого. Отпускаем колеса одновременно. Что произойдет? А если менять углы? Колеса тоже не ведущие, их тащит сила тяжести...Да, каждое колесо много тяжелей веревок. Поедут или не поедут? biggrin.gif

biggrin.gif это вы так объяснить пытались Ал Сеич?
я вот токо с пятово раза понял о чом у Вас реч
да и то не до конца wink.gif


to opponents:

силы то силы то разрисуйте колеги

на самолёт действует сила тяги
так?
сила это ускорение
так?
чтобы не было ускорения нужно чтобы сумма сил, приложенных к объекту была равна нулю
какая сила компенсирует силу тяги?
сила трения колеса о ленту транспортера?
фиг
сила трения колеса о транспортер лишь вращает колесо шасси

smile.gif

Автор: LVV 10.11.2006, 8:34

Цитата(Сергей В. @ Nov 7 2006, 13:43 )
Предлагаю к этой теме добавить голосование. biggrin.gif

И вынести решение по результатам голосования biggrin.gif

Автор: LVV 10.11.2006, 8:47

Цитата(OVKT @ Nov 9 2006, 19:34 )
Любое движение быстрее "ленты" является нарушением условий задачи.

С этим полность согласен. Могу доказать что никакой транспортер не сможет двигаться со скоростью, вращения колеса.
Цитата
Что и требовалось доказать в очередной раз: НЕТ, НЕ УЛЕТИТ, если соблюдать исходное условие задачи.

ИМХО это говорит о невыполнении условий задачи, т.е. в самой постановке вопроса стоит ошибка, а не о том что самолет не взлетит.
А самолету никуда не деться кроме как взлететь ибо
Цитата
на самолёт действует сила тяги

сила это ускорение

Автор: OVKT 10.11.2006, 20:39

Представим себе самолёт не на шасси, а на... 3-х иголках и гладкий непротыкаемый транспортёр, то бишь трения никакого. Но такого не бывает. Где вы видели, чтобы транспортёр двигался, а расположенный на нём предмет оставался бы неподвижным даже при минимальном трении? Если быстро выдёргивать подстилку из-под предмета, то на ОДИН_МИГ система осуществит такой фокус. Но продолжаться даже 5 секунд такое не может, так что этот пример тоже неадекватный.
Но мы можем представить гладкую пологую ГОРКУ, на вершину ставим предмет, и он скатывается вниз по плоскости. Включаем тумблер и плоскость горки начинает двигаться к вершине, и предмет остаётся неподвижным относительно воздуха, одновременно скатываясь относительно плоскости.
Если предмет заменить самолётом, то он НИКУДА не ПОЛЕТИТ.
Просто условие задачи о "невзлетаемости". Взлетел = нарушил условие.

Автор: Giedi Prime 10.11.2006, 22:43

Блин, вопрос-то понтовый! Для младшего школьного возраста.

НЕ ВЗЛЕТИТ!

Автор: ncx 11.11.2006, 10:49

Цитата(OVKT @ Nov 10 2006, 20:39 )
Представим себе самолёт не на шасси, а на... 3-х иголках и гладкий непротыкаемый транспортёр, то бишь трения никакого. Но такого не бывает. Где вы видели, чтобы транспортёр двигался, а расположенный на нём предмет оставался бы неподвижным даже при минимальном трении? Если быстро выдёргивать подстилку из-под предмета, то на ОДИН_МИГ система осуществит такой фокус. Но продолжаться даже 5 секунд такое не может, так что этот пример тоже неадекватный.
Но мы можем представить гладкую пологую ГОРКУ, на вершину ставим предмет, и он скатывается вниз по плоскости. Включаем тумблер и плоскость горки начинает двигаться к вершине, и предмет остаётся неподвижным относительно воздуха, одновременно скатываясь относительно плоскости.
Если предмет заменить самолётом, то он НИКУДА не ПОЛЕТИТ.
Просто условие задачи о "невзлетаемости". Взлетел = нарушил условие.


чото я не понял зачем про иголки то написали
както мысль осталась не (до) развитой smile.gif

всё пытаетесь умозрительно представить себе картину
а вы сядьте лучше да нарисуйте как я уже говорил все силы действующие на самолет


вы поймите одну вещь: влияние транспортера на движение самолета определяется величиной силы трения
а в случае с колесом эта сила мизерная
да даже если бы не колесо было а лыжы какиенибудь всё равно сила трения много меньше силы тяги
причём эта сила практически НЕ ЗАВИСИТ от скорости движения самолета относительно ленты (или наоборот ленты относительно самолета) кроме случая v=0 когда действует сила трения покоя
с какой бы скоростью не двигалась лента (со скоростью вращения колеса, скоростью движения самолета, субсветовой) сила трения не изменится
а кроме неё в системе присутствуют только сила сопротивления воздуха, вес и тяга

НУ ПОЙМЁТ КТОНИБУДЬ НАКОНЕЦ???? или все только себя слушают?

smile.gif

Автор: OVKT 12.11.2006, 19:12

Непонятно, к какому выводу пришёл предыдущий автор.
Решая задачу, надо понять: какая скорость должна быть у полотна, чтобы обеспечить крайние случаи: 1) Выключенный самолёт на ленте катится взад; 2) Самолёт едет по неподвижной ленте; 3) Самолёт с вяло_включенным движком катится взад, пГенепГеменно отталкиваясь от воздуха; 4) Правильно: самолёт с включенным движком двигаясь компенсирует откат взад, по мысли "взлетевших", такое состояние невозможно?
Допустим, лыжник стоит на движущейся ленте, а палками упирается в неподвижную землю. Рывок - и лыжник проехал конечный путь. Пока переставляет палки - отъехал назад. Аналог: горка. Далеко уедет?
Допустим, палуба корабля, идущего направлением на юг. А самолёт по палубе едет на север со скоростью, СТРОГО равной скорости движения корабля. Взлетаем?
Так же и ранее упоминавшаяся мною летящая вертикально ракета, преодолевающая падение, но остающаяся на месте; и наклонная горка с предметом. Самолёт на горке: двигатели ВКЛючены, от воздуха (или реактивно) отталкивается, работу совершает, шасси без трения, а с места НЕ двигается. Взлетаем? Ну-ну...
А рядом с застрявшим самолётом пилот по канату залез на вершину той же горы.
Или ракета в космосе летит со скоростью 1 км/сек, а на самом деле падает на Солнце. А рядом космонавт по канату дополз до Сатурна.
Просто в условие задачи с самолётиком внесена НЕВЗЛЕТАЕМОСТЬ: взлетел = нарушил условия задачи. Всё очень просто. Все согласны?

Автор: LVV 13.11.2006, 9:14

Цитата(OVKT @ Nov 12 2006, 19:12 )
Допустим, лыжник стоит на движущейся ленте, а палками упирается в неподвижную землю. Рывок - и лыжник проехал конечный путь. Пока переставляет палки - отъехал назад. Аналог: горка. Далеко уедет?

Так самолет отталкивается от воздуха, а не от транспортера
Цитата
Допустим, палуба корабля, идущего направлением на юг. А самолёт по палубе едет на север со скоростью, СТРОГО равной скорости движения корабля. Взлетаем?

В задаче нет такого ограничения.

Автор: ncx 13.11.2006, 14:52

горбатого...

Автор: OVKT 13.11.2006, 19:53

"А теперь горбатый! Я сказал горбатый!" (из фильма).
Представьте себе самолёт, стоящий на неподвижной ленте. Лента стала двигаться, стоячий самолёт откатывается назад. Скорость ленты не увеличивается. Самолёт включается и начинает двигаться, отталкиваяь от водуха. Но самолёт двигается медленно, и на ленте катится НАЗАД относительно воздуха, даже с включёнными двигателями.
Те, у кого самолёт "улетел", такой сценарий даже не допускают ?
Далее самолёт разгоняется и приобретает скорость, равную скорости ленты. И остаётся неподвижным относительно воздуха, хотя и отталкивается от воздуха или реактивно. Вот это положение и есть 100% удовлетворяющее задаче. Ежели самолёт ускорится, то это будет нарушением исходных условий. Также и улетевший самолёт просто двигался быстрее ленты, а это вне условий, т.к. по условиям задачи их скорости строго одинаковые.
Самолёт улетит, если нарушит условия задачи. Строго по условиям улететь не может.

Автор: LordN 14.11.2006, 0:30

Цитата
И остаётся неподвижным относительно воздуха, хотя и отталкивается от воздуха или реактивно.
как вы себе это представляете - одновремённо отталкиваться от воздуха и оставаться относительно его в покое? :wacko:
сказано же уже - весь вопрос в точке отсчета и силе трения колесо/полотно. взлетит он или нет зависит только от скорости потока воздуха относительно самолета и более ни от чего.
сила трения может повлиять на скорость самолета относительно воздуха, как в вашем примере с гравитацией, чисто теоретически допустить такое можно, и об этом тоже сказано было в самом начале, можно прибить гвоздями колёса к полотну, т.о. соблюсти все условия задачи и самолет не полетит. а можно колеса смазать до полного отсутствия силы трения - тогда самолету деваться не куда, только лететь.
можно также приколотить гвоздями воздух к самолету - он тогда тоже лететь не сможет если он пропеллерный и не очень мощная ракета.
если воздух оставить в покое или прибить к земле или даже полотну и полотно будет двигаться навстречу самолету - то самолет полетит (только если колеса не прибиты) rolleyes.gif
ну и так можно долго кругами ходить бродить...
вобщем самолет может лететь, а может и не лететь, все зависит от математика, диспечера выдающего разрешение на взлёт, наличия керосина в баках и трезвого пилота за рулём tomato.gif

Автор: LVV 14.11.2006, 7:46

Цитата(OVKT @ Nov 13 2006, 19:53 )
Лента стала двигаться, стоячий самолёт откатывается назад.

На основании чего? Для того чтобы заставить изменить скорость необходимо к телу приложить силу в течении кокого-то времени. Условно считаем что сопротивление качению и сопротивление в подшипниках равно 0.

Цитата
И остаётся неподвижным относительно воздуха, хотя и отталкивается от воздуха или реактивно. Вот это положение и есть 100% удовлетворяющее задаче.

Доказательсва в студию.

Предположим что у самолета идеальные колеса и подшипники, т.е. сопротивление качению = 0, коэффициент сцепления колеса с транспортером равен бесконечности. Коэффициент проскальзывания колеса постянный и = 0.

Тогда при появлении тяги от двигателя, самолет начнет движение относительно воздуха и транспортера, колесо начнет вращаться . Согласно уловиям задачи транспортер будет стремиться компенсировать движение колеса, при этом увеличивая скорость вращения колеса. Т.к. колесо начинает вращаться быстрее, то и транспортер увеличивает свою скорость. Замкнутый круг.
Если следовать условиям задачи то скорость вращения колеса и транспортера мгновенно увеличится до бесконечности.
При бесконечной скорости колеса и транспортера между ними образуется воздушный клин, который позволит оторваться колесу от транспортера, и продолжить ускорение самолета никаким образом не связанным с транспортером.
Все самолет взлетел.

Так-же доказывается и при реальных коэффициентах сопротивлений, но уже с более сложными математическими выкладками.

Автор: e.o.l. 14.11.2006, 8:19

не удержался, опять скажу, хотя зарёкся в теме не участвовать
при чём тут колёса, при чём трение и пр.???? об чём вы, ребяты??

элементарно, как сказал LordN

Цитата
взлетит он или нет зависит только от скорости потока воздуха относительно самолета и более ни от чего

ни один работающий двигатель любого обычного самолёта (винтовой и тем более реактивный) не обеспечит достаточного обдува воздухом всей поверхности крыла для создания подъёмной тяги, если сам самолёт неподвижен относительно его (воздуха) основной массы!!!! с какой бабушки ему лететь???? bleh.gif bleh.gif
извиняюсь за эмоциональную окраску, накипело biggrin.gif

Автор: LVV 14.11.2006, 8:56

А где сказано что самолет остается неподвижным относительно воздуха?

Автор: e.o.l. 14.11.2006, 9:06

а об этом уже говорилось про некорректное условие задачи, подразумевающие неоднозначные его трактовки
но большинство участников по умолчанию приняли именно такой вариант, как мне показалось (к ним и обращаюсь biggrin.gif )

Автор: zr84 14.11.2006, 10:46

Думал тема умерла!. Ан нет!
Чем больше идет обсуждение вопроса, тем вероятность ветра, бури, шторма.. увеличивается( в пределе 1)- в полет коллеги!
Впрочем можно и сейчас, что ждать-то-установим пару, тройку мощных ветродуек(условия задачи не запрещают)- в полет, только в полет!

Автор: airhandl 14.11.2006, 10:58

Цитата
некорректное условие задачи

задачка-то для второго курса кулинарного техникума, поэтому все оговорки насчет идеального сферического коня в вакууме опущены
Вот уважаемые мужи и треплют друг друга за бороды, учитывая обдув плоскостей двигателем и особенности конструкции закрылков wub.gif
thumbdown.gif
задачка явно из _теоретической_ механики для домохозяек и дизайнеров, скорость ветра относительно земли не учитывается - это еще из раздела кинематика, а не динамика !
итого: неподвижная земля с неподвижным воздухом, переносная скорость транспортера _в_каждый_момент_времени_ равна по величине и противоположна по направлению относительной скорости самолета по транспортеру, абс.скорость (самолет относительно воздуха) нуль => подъемная сила нуль
все
абзац
какаянах сила трения иголок по лыжам, какие воздушные клинья? какие крайние случаи ? по условию перен. и отн. скорости _всегда_ равны
Не следует умножать сущности сверх необходимого (с) Оккама

Автор: ncx 14.11.2006, 11:00

горбатых...
biggrin.gif

думайте друзья, думайте
откройте учебники по физике, найдите все законы ньютона, перечитайте их

блин ну не самому же мне картинку рисовать?

и поменьше эмоций, побольше желания понять доводы оппонента

to LordN
щитаем что условия кроме оговоренных отдельно - нормальные wink.gif
и никаких гвоздей

Автор: zr84 14.11.2006, 11:13

Цитата(airhandl @ Nov 14 2006, 10:58 )
задачка явно из _теоретической_ механики для домохозяек и дизайнеров...

Не знаю таких домохозяек.
Про тер/мех
Равество скоростей по диаметру колеса говорит о том, МНЦ(мгновенный центр скоростей) находится в центре колеса и нет причин появления dV(перемещение самолета), т.к. ВПП аппаратура отслеживает этот момент. Это значит что самолет покоится. Но задача ведь не только об этом. История: молодому студенту Н.Бору задали задачу: как измерить высоту дома барометром? Он кучу вариантов накидал...
Мозги упражняем однако.

Автор: LVV 14.11.2006, 12:14

Цитата(airhandl @ Nov 14 2006, 10:58 )
неподвижная земля с неподвижным воздухом, переносная скорость транспортера _в_каждый_момент_времени_ равна по величине и противоположна по направлению относительной скорости самолета по транспортеру

Еще раз повторяю что условия задачи стоят не так как вы их пытаетесь прочитать, а дословно
Цитата
чтобы скорость вращения колес самолета была равна скорости движения полотна


Не больше и не меньше. Приложите ваши теоритические выкладки у условиям задачи, а не к ассоциациям

Добавлено - 12:22
Цитата(zr84 @ Nov 14 2006, 11:13 )
Равество скоростей по диаметру колеса говорит о том, МНЦ(мгновенный центр скоростей) находится в центре колеса и нет причин появления dV(перемещение самолета), т.к. ВПП аппаратура отслеживает этот момент.

Аппаратура отслеживает скорость вращения колеса, а не движение самолета.
Кто сказал, что колесо движется без проскальзывания по транспортеру?

Автор: LordN 14.11.2006, 12:22

Цитата
итого: неподвижная земля с неподвижным воздухом, переносная скорость транспортера _в_каждый_момент_времени_ равна по величине и противоположна по направлению относительной скорости самолета по транспортеру
как вы себе это представляете? воздух-то неподвижен!!! самолет катится по полотну и остается неподвижен в воздухе? как такое возможно? за счет чего? кто держит самолет? wink.gif привод то не на колесы!!! а на воздух!!!

Автор: LVV 14.11.2006, 12:54

Цитата(airhandl @ Nov 14 2006, 10:58 )
задачка явно из _теоретической_ механики для домохозяек и дизайнеров, скорость ветра относительно земли не учитывается - это еще из раздела кинематика, а не динамика !

Цитата
подъемная сила нуль


На сколько я помню подъемная сила тер.мехом не рассматривается. Вроде как подъемную силу в гидравлике начинают расматривать.
Хотя могу и ошибаться.

Автор: airhandl 14.11.2006, 18:45

2 LVV: Если уж хотите _дословно_ - то скорость _вращения_ колеса по определению угловая, а скорость полотна линейная
Вы что с чем сравниваете ?!

LordN писал:

Цитата
основной смысл этой задачи - показать своё мышление

Автор: LordN 14.11.2006, 19:20

Цитата
скорость _вращения_ колеса по определению угловая, а скорость полотна линейная
точно! но в задаче ни слова про углы. хотя мысль хорошая, мне нравится.

Автор: OVKT 14.11.2006, 19:59

Яндексом "взлетит ли самолет" нашёл подобные темы, до 22-х страниц забавных ответов.
Проблема таких вопросов в том, что если выступит даже академик авиации, то любой школьник скажет ему: "ты академик рассматриваешь одну из сторон, а вот с другой стороны ты неправ".
Однако, заметьте: авиационщики выступают с однотипными ответами...
Ещё пример: река (транспортёр), гидросамолёт на поплавках. Бурное течение против движения гидросамолёта, его сносит течением назад. Пилот героически включает моторы, и - о, чудо! - гидросамолёт, отталкиваясь от воздуха, стал неподвижным относительно воздуха.
Однако, никуда не улетел...

Автор: kroudion 14.11.2006, 20:16

Веселуха продолжается clap.gif
Никто не хочет воспользоваться советом ncx и нарисовать силы, действующие на самолёт, зато прибивают гвоздиками и заглядывают в закрылки, так что становится непонятно, отчего самолёты летают - ведь крыльями они не машут...

Автор: LVV 15.11.2006, 7:33

airhandl
Угловую скорость колеса умножаем на его радиус, получаем линейную скорость наружной поверхности колеса.
Хотя это уже мои ассоциации.

Добавлено - 07:39

Цитата(airhandl @ Nov 14 2006, 18:45 )
то скорость _вращения_  колеса по определению угловая, а скорость полотна линейная

Вывод я думаю такой, т.к. угловая скорость не может быть равной линейной, то сами условия задачи некорректны.

Автор: ncx 15.11.2006, 10:53

предлагаю всем разделицца на два непримиримых лагеря: "влетисты" и "невзлетисты"
теперь, в каждом споре на темы околосвязанные с основной тематикой форума первым делом спрашивать: слыш друг, а ты из "взлетистов" или из ЭТИХ
соответственно если друг говорит што как раз ты - из ЭТИХ, а он из правых, то он автоматически становитса врагом и любые ево доводы игнорируются или подвергаются массовому освистыванию

предлагаю "невзлетистов" считать правыми а "взлетистов" - левыми (это принципиально!)
или наоборот

Автор: Bers 15.11.2006, 10:59

Цитата(ncx)
предлагаю "невзлетистов" считать правыми а "взлетистов" - левыми (это принципиально!)
или наоборот

Помимо правых и левых еще должны быть центристы. Это те, у кого "взлетит, но низенько"... Правда, по итогам голосования, центристов пока на форуме не наблюдается...

Автор: ncx 15.11.2006, 14:58

в данном случае центристы это эксцентрики как это ни парадоксально

Автор: Сергей В. 15.11.2006, 17:55

Цитата
всем разделицца на два непримиримых лагеря: "влетисты" и "невзлетисты"

а по количеству присланных в эту тему постов ввести звания:
"заслуженный взлетист/невзлетист" и "почетный взлетист/невзлетист" biggrin.gif

Автор: OVKT 15.11.2006, 19:34

Смотрим: кадр из мульта про белку, которую сбил поток орехов. У нас та же картинКа. Вы поймите простую вещь: самолёт с ВКЛюченным движком может двигаться взад и стоять на месте при встречном движении, в нашем случае "ленты". Аэроплан улетел у тех, кто думает: включил движок - сразу двигаешься со скоростью звука. Ан нет. Надо мыслить глобально.

 

Автор: LordN 15.11.2006, 19:42

Цитата
Надо мыслить глобально
нет, надо просто мыслить. у белки - ноги. у самолета - крылья и движок с приводом не на колесы, а на самолет biggrin.gif

Автор: LVV 16.11.2006, 7:07

Переходим на доказательства основываясь на картинках из мультфильмов?
Классно. wink.gif
В мультфильмах видел еще ковер самолет, скатерть самобранку и т.д. и т.п.
cool.gif

Добавлено - 07:15
OVKT Прошу Вас попробовать объяснить почему самолет остается на месте. Я ни как не могу понять почему он остается на месте (относительно воздуха).

Автор: LordN 16.11.2006, 7:29

Цитата
Прошу Вас попробовать объяснить почему самолет остается на месте.
он держится когтями за полосу wink.gif как орёль

Автор: OVKT 16.11.2006, 20:05

Ракета падающая на Солнце знаете тоже "отталивается"...
Не хотел я говорить здесь, но... вспомните последнее падение ТУ-154 над Донецком. Самолёт летел, но упал. При работающих двигателях он свалился из не_горизонтального положения, насколько понимаю, свалился в воздухе в сторону хвоста, а потом упал на землю за минуты. Хотя казалось бы... отталкивайся от воздуха...

Автор: LVV 17.11.2006, 7:50

OVKT Причем здесь Солнце, есть конкретная задача, я задал конкретный впрос. Зачем писать про солнце?

Автор: Сергей В. 17.11.2006, 10:21

Цитата(Mark-off @ Nov 16 2006, 21:47 )
Значит, судно на воздушной подушке не сможет двигаться против течения реки?

clap.gif

Автор: airhandl 17.11.2006, 12:13

По сравнению с http://deforum.ru/forum/viewtopic.php?t=21346&postdays=0&postorder=asc&start=0 вялое у нас обсуждение :wacko:
А тема и правда жевалась на http://socionics.org/forums/1/1155040/ShowThread.aspx сайтах
Если попросить Гугль поискать заголовок темы - то на первой же странице он категорично ответит: Без российского титана не взлетит biggrin.gif
Вариант решения, например, http://avia.ru/cgi/discshow.cgi?id=4305534474877965241371059029310&page=22

Цитата
УУУУУУУУ!!!! ОООООООООО !!!!! Нет, решительно пора заканчивать этот затянувшийся ночной кошмар!

В условии задачи сказано - копирую из первого поста:
==Покрытие может двигаеться против направления взлета самолета==
Но там не сказано - "ТОЛЬКО против направления взлета".

Задача имеет единственное непротиворечивое решение.
Самолет движется вперед со скоростью V относительно земли. И полотно движется вперед со скоростью V1, равной половине от скорости самолета. В момент, когда V = Vвзл, самолет спокойно взлетает, причем его колеса вращаются вдвое медленнее обычного.
V =V1 + V2, при этом V1 = V2 = 2*3,14*R*W. Все условия соблюдены.

biggrin.gif

Автор: LVV 17.11.2006, 14:52

airhandl Одним взлетистом прибыло?

Автор: OVKT 17.11.2006, 19:37

Картинка в конце ответа все смотрим!!!
Буду отвечать в стиле ShaggyDoc.
В 1965 году, ещё в советские времена, под руководством нашего головного проектного института, на БАМе по заказу МАИ и ХАИ проводился следующий эксперимент.
Был пущен поезд по прямой трассе, а на поезде был поставлен 1-й в СССР беспилотный самолёт с направлением движения от головы в хвост поезда.
Поезд имел длину 3 км и тянулся 3-мя локомотивами. Схематично см. рисунок.
Те, кто "думает", что предлагаемая в задаче лента "нереальная" - ошибается!
Поезд имел на крыше непрерывную плоскость, создающую взлётную полосу.
Поезд со стоящим самолётом на крыше разогнали с 0 до скорости 60км/ч и начали эксперимент.
В момент начала эксперимента поезд двигался, а самолёт стоял на крыше неподвижным, но относительно воздуха двигался взад со скоростью 60км/ч.
Когда включили двигатели, вопреки ожиданиям, самолёт всё равно ехал задом наперёд даже с включёнными движками.
С увеличением тяги самолёт стал продвигаться по ленте, установленной на крыше поезда, но относительно воздуха продолжал ехать задом (!!!) наперёд.
Когда самолёт приобрёл достаточную скорость, он стал неподвижным относ. воздуха, всё движение компенсировалось движением поезда.
Зная секретные разделы теории относительности Эйнштейна и опасаясь искривления пространственно-временнОго континюума, присутствовавший на эксперименте генералитет МинОбороны СССР во главе с маршалом Жуковым и сталинградцем доцентом Паулюсом, приказали прекратить эксперимент и лишили финансирования.
Тем самым спасли человечество от неминуемого коллапса из-за какого-то там самолётика.
Во время так называемой "перестройки" и "приХватизации", эксперимент пытались повторить под руководством министра обороны Язова в августе 1991 года на Семипалатинском полигоне. Помните, сообщение в программе "Время": "Проведены испытания в интересах народного хозяйства. Мощность самолётика составила 20 килотонн". Это было ОНО!
Однако, легко заметить, что задача удовл. условиям, экспериментально решена на практике в 1965 году: самолёт был неподвижным даже с включёнными движками.
Вы поймите: в данной реальной модели любое опережение самолётом полотна и дальнейшее движение на взлёт будет выходить за пределы условий задачи.
А именно: при движении относительно воздуха для взлёта - скорость вращения колёс будет БОЛЬШЕ скорости встречного полотна, что выпадает за условия задачи.
Самолёт может улететь, если нарушит условия задачи, строго по условиям - улететь НЕ МОЖЕТ!

 

Автор: zr84 20.11.2006, 0:10

Выбрал время-нарисовать картинки с формулками.
Итак: ВЗЛЕТЕТЬ НЕЛЬЗЯ НЕВЗЛЕТЕТЬ
Где будем ставить запятую или жирную точку?
Прикрепляю файл:формат pdf, объем:192кБт

 Plane.pdf ( 192,22 килобайт ) : 128
 

Автор: Bers 20.11.2006, 2:35

Сразу видно, что форум у нас не простой, а инженерный. Что ни пост, то шедевр. На других форумах - десятки страниц флуда, а у нас - с картинками, с формулами, с доказательствами. Любо-дорого посмотреть.

Автор: LVV 20.11.2006, 8:12

Цитата(OVKT @ Nov 17 2006, 19:37 )
А именно: при движении относительно воздуха для взлёта - скорость вращения колёс будет БОЛЬШЕ скорости встречного полотна, что выпадает за условия задачи.

Скорость вращения колес будет выше, скорости движения полотна даже если самолет будет оставаться неподвижным относительно воздуха.

Какое отношение имеет самолет находящийся на поезде к самолету из нашей задачи? Не понятно. Почему самолет на поезде не смог развить скорость выше 60 км/ч тоже не ясно.

Автор: airhandl 20.11.2006, 16:46

LVV Не biggrin.gif
я ваще-то по..ист tongue.gif

Автор: OVKT 20.11.2006, 19:39

Спрашивают: "При чём здесь ракета", приводимая в моих примерах? При том, что реактивное движение не_означает 100% движения вперёд, при противодействующих силах реактивная ракета будет лететь взад, даже реактивно отталкиваясь от своего собственного топлива. У тех, кто этого не понимает - у тех самолёт улетел.
Другие грубые ошибки "улетевших" в том, что им чудится, якобы дорожка таковая - нереальная; потом: что лента не влияет на движение самолёта взад. На примере с поездом видим, что и дорожка реальная, и самолёт может двигаться взад и стоять на месте - стоя на ленте с включенными движками, что и требовалось доказать в очередной раз.
Главный вывод: самолёт улетает, лишь нарушив условие задачи.

Или вот ещё напоследок: к хвосту самолёта цепляем груз массой 20тонн за трос и через блок перекидываем в пропасть, но пока не отпускаем. Вертикальная нагрузка переводится в горизонтальную. Заводим движки, а груз кидаем вниз. Что мы видим: самолёт с включенным реактивнейшим двигателем улетел... в пропасть вслед за грузилом, но пилот успел катапультироваться.
Этот же пилот приземлился - глянул, а к нему тоже прицеплено грузило, но массой 100кг. Этот пилот схватился за лежавший рядом трос из теории 1-го предыдущего автора и стал успешно выкарабкиваться и грузило из пропасти тоже вытащил только благодаря тросу.
Пилота спрашивают: - Что случилось с вашим самолётом? Пилот отвечает: - Он улетел.

Каждый из вас может найти СВОИ варианты ответа здесь 1,5M!:
http://www.avia.ru/cgi/discshow.cgi?id=4305534474877965241371059029310&topiccount=1101&mode=all
На ОЧЕНЬ большом обсуждении мои гипотезы были высказаны неким Lukas 26/07/2003 и некий Доныч ещё 01/08/2003.
Мои гипотезы вчерась закончилися...

Автор: LVV 21.11.2006, 8:29

Силы сопротивления у самолета только от аэродинамического сопротивления об воздух, и сопротивление качению колес. Если тяга двигателей самолета превысит сопротивление качению и аэродинамическое сопротивление самолета, при скорости взлета, то самолет взлетит. В противном случае не взлетит.
Максимальное сопротивление качению колес может составить 0,8 от собственного веса самолета.
Об этом уже LordN писал.

Автор: OVKT 27.11.2006, 21:14

Один из "ответов" здесь: http://amis.h11.ru/answers.htm Там аналогия: "По земле автомобиль тянет за верёвочку самолёт по конвейеру". А конвейер мол не оказывает влияния. Да только если взять верёвочку тонкую, то бечева порвётся, а при подобном объяснении ниточка должна быть сколь угодно тонкой, раз конвейер "не влияет". Что-то не вяжется...

Автор: e.o.l. 28.11.2006, 7:58

Цитата(LVV @ Nov 21 2006, 08:29 )
Силы сопротивления у самолета только от аэродинамического сопротивления об воздух, и сопротивление качению колес.

а по-моему главной силой сопротивления будет сила тяжести wink.gif
подловил? biggrin.gif

Автор: LVV 28.11.2006, 11:38

Цитата(e.o.l. @ Nov 28 2006, 07:58 )
а по-моему главной силой сопротивления будет сила тяжести wink.gif
подловил? biggrin.gif

Нет. если только взлет не вертикальный.

Автор: AAANTOXA 20.1.2008, 0:55

Старую добрую тему откопал )))))))

а что по этому поводу думают более новые учасники форума а ?
я думаю этот вопрос можно задавать при приёме на работу laugh.gif

Автор: Сфинкс 20.1.2008, 10:20

Моё мнение здесь в сообщении №100

Автор: sonsumerek 20.1.2008, 13:56

Цитата(Bers @ 3.11.2006, 7:20) [snapback]56716[/snapback]
Есть такая задачка:

"Самолет (реактивный или винтовой) стоит на взлетной полосе с подвижным покрытием (типа транспортера). Покрытие может двигаться против направления взлета самолета, т.е. ему навстречу. Оно имеет систему управления, которая отслеживает и подстраивает скорость движения полотна таким образом, чтобы скорость вращения колес самолета была равна скорости движения полотна.
Внимание, вопрос: сможет ли самолет разбежаться по этому полотну и взлететь?"


Ну дык что - Взлетит или не взлетит? Есть версии?

тему не читала ))
мнение из зала (от супруга):
реактивный твердотопливный - взлетит
винтовой - нет
p.s. а есть еще самолеты вертикального взлета, винтовые, но ведь речь не о них?..

Автор: ViC 20.1.2008, 14:02

Цитата(sonsumerek @ 20.1.2008, 12:56) [snapback]212480[/snapback]
реактивный твердотопливный - взлетит

это на уголе что ли?

Мой ответ - не взлетит, потому что нет потока воздуха. Самолет ИМХО на месте стоит.
Нет аэродинамики, за счет чего самолет поднимается в воздух.

Автор: andrey R 20.1.2008, 14:04

Цитата(sonsumerek @ 20.1.2008, 13:56) [snapback]212480[/snapback]
мнение из зала (от супруга)

А логика какова в этих утверждениях? Самолёт отталкивается от от воздуха, а не от полотна. При разгоне, сначала набирается скорость электромоторами колёс, основные двигатели включаются при достижении определённой скорости. Бегущая назад дорожка ликвидирует эту скорость относительно земли. Таким образом, вопрос вроде сводится к тому, взлетит ли самолёт на основных двигателях с места. Вроде так.
/тему тоже не читал/ biggrin.gif

Автор: инж323 20.1.2008, 14:09

Не надо разбегаться и крутить колеса.Сопла в пол и вверх!Летать так летать,а эти("систему управления, которая отслеживает ") пусть скорость колес с точностью до сотни об\мин. считают. biggrin.gif

Автор: BUFF 20.1.2008, 14:17

Цитата(andrey R @ 20.1.2008, 14:04) [snapback]212485[/snapback]
А логика какова в этих утверждениях?

нету... у автора задачи - горе от ума... он там вводит странные усложнения, пытаясь приблизиться к реальности - но забывает при этом подумать...
ПДФ прочитал smile.gif
отвечать - западло, все и так очевидно...
твердотопливный реактивный двигатель из зала порадовал даже больше, чем винтовой самолет вертикального взлета biggrin.gif

Автор: R.A.S. 20.1.2008, 15:03

Представил такую аналогию и все стало ясно.
Пловец в воде плывет за счет рук. Но на ногах у него надеты коньки и стоит он на транспортной ленте. Что будет происходить? Если силой трения колесиков в коньках пренебречь, что пловец как плыл, так и будет плыть, относительно неподвижного наблюдателя. Транспортная лента с какой бы скоростью не вращалась не будет иметь влияния на пловца. А вот колесики на роликах будут крутится со скоростью равной скорости транспотрной ленты + скорости пловца.
Так же с самолетом. Вывод самолет двигатся будет, и если трение в шасси позволит, он достигнет скорости при которой подьемная сила превысит силу тяжести и взлетит.

Автор: sonsumerek 20.1.2008, 15:20

Цитата(andrey R @ 20.1.2008, 14:04) [snapback]212485[/snapback]
А логика какова в этих утверждениях? Самолёт отталкивается от от воздуха, а не от полотна. При разгоне, сначала набирается скорость электромоторами колёс, основные двигатели включаются при достижении определённой скорости. Бегущая назад дорожка ликвидирует эту скорость относительно земли. Таким образом, вопрос вроде сводится к тому, взлетит ли самолёт на основных двигателях с места. Вроде так.
/тему тоже не читал/ biggrin.gif

логика была какая то, что-то там про нулевую тягу у реактивных двигателей и бла-бла-бла ))

Цитата(BUFF @ 20.1.2008, 14:17) [snapback]212497[/snapback]
твердотопливный реактивный двигатель из зала порадовал даже больше, чем винтовой самолет вертикального взлета biggrin.gif

было много текста, я только результат напечатала
а чем вам самолет вертикального взлета не угодил то?.. мой благоверный говорит, что такие самолеты не выпускают конечно, но эпизодически испытывают... типа - там проблем только с посадкой biggrin.gif , очень не удобно, знаете ли...

Автор: andrey R 20.1.2008, 15:43

Цитата(sonsumerek @ 20.1.2008, 15:20) [snapback]212519[/snapback]
логика была бла-бла-бла ))
Хорошая логика smile.gif Главное - логичная biggrin.gif
Цитата(sonsumerek @ 20.1.2008, 15:20) [snapback]212519[/snapback]
говорит, что такие самолеты не выпускают конечно

Выпускают. Тока не гражданские smile.gif Но проектов много. Вот тут, к примеру
http://vtol.boom.ru/vtol/
Многоцелевой отечественный
http://airbase.ru/hangar/planes/russia/yak/yak-141/
Ну и знаменитый HARRIER. Ссылку лень искать smile.gif

Автор: BUFF 20.1.2008, 15:50

Цитата(sonsumerek @ 20.1.2008, 15:20) [snapback]212519[/snapback]
мой благоверный говорит, что такие самолеты не выпускают конечно, но эпизодически испытывают...

не верьте Вы ему... наши Як-38, Як-141 (он же Як-41М)
Англия - 5 модификаций Харриеров...
Испания - Матадор 2
беда в том, что все они реактивные, а вашему благоверному - книжки читать, и физику учить, а не гнать пургу про твердотопливный реактивный двигатель на самолетах biggrin.gif
или пусть гонит про ракету с педальным приводом clap.gif

Автор: AAANTOXA 20.1.2008, 19:22

Цитата(BUFF @ 20.1.2008, 15:50) [snapback]212525[/snapback]
не верьте Вы ему... наши Як-38, Як-141 (он же Як-41М)
Англия - 5 модификаций Харриеров...
Испания - Матадор 2
беда в том, что все они реактивные, а вашему благоверному - книжки читать, и физику учить, а не гнать пургу про твердотопливный реактивный двигатель на самолетах biggrin.gif
или пусть гонит про ракету с педальным приводом clap.gif

Вот серийный винтовой с вертик взлётом.
Кроме того сушествуют серийные твердотопливные ускорители которые "навешиваются" на обычный самолёт чтобы укоротить пробег (при взлёте) после чего сбрасываются. Ракетный ускоритель это ведь двигатель?
Но это не важно так как ИМХО в задаче нужно считать что самоль нормального типа нормальной схемы

 

Автор: Kult_Ra 20.1.2008, 19:46

2. Не взлетит
По аналогии с подводной лодкой - нет скорости - тонет и надо продуть балласты, что всплыла. Погружение - налив воды в цистерны.
Самолет не продуть! Значит не всплывёт, нет движения относительно среды - останется на месте.

Автор: BUFF 20.1.2008, 20:02

Цитата(Kult_Ra @ 20.1.2008, 19:46) [snapback]212598[/snapback]
По аналогии с подводной лодкой - нет скорости - тонет и надо продуть балласты, что всплыла. Погружение - налив воды в цистерны.

blink.gif blink.gif blink.gif
Вам осталось танки тут еще обсудить... что-нибудь вроде "если залить в дизельный двигатель бензин - калибр пушки поменяется, или нет"
и варианты ответов: да, есть, так точно, ниасилил...
того, кто правильно ответит на вопрос, но не сможет угадать слово - Аантоха рекомендует на работу к Свинксу... он тоже загадки любит...
пойду, аватару Сантехника с СОКа сюды прилеплю.... в знак того, что тема раскрыта полностью и всеобъемлюще...

Автор: andrey R 20.1.2008, 20:07

http://smiles2k.net/aiwan_smiles/4/index.html

Автор: ViC 20.1.2008, 20:07

Цитата(BUFF @ 20.1.2008, 19:02) [snapback]212609[/snapback]
пойду, аватару Сантехника с СОКа сюды прилеплю.... в знак того, что тема раскрыта полностью и всеобъемлюще...

Ай проказник, ай шалунишка clap.gif пожалей Сантехника, ему и так досталась...аватара...красивая bleh.gif

Автор: инж323 20.1.2008, 20:08

Не,тут по Бунински надо сказать. biggrin.gif

Автор: andrey R 20.1.2008, 20:13

Цитата(ViC @ 20.1.2008, 20:07) [snapback]212613[/snapback]
ему и так досталась...аватара...красивая bleh.gif

Красота требует жертв biggrin.gif А то и шоб красиво, и шоб не досталось...так не бывает tongue.gif

Автор: ViC 20.1.2008, 20:17

Цитата(andrey R @ 20.1.2008, 19:13) [snapback]212616[/snapback]
шоб не досталось...так не бывает tongue.gif

От хулиганье clap.gif Человек о вечном думает, о красивом мечтает, а Вы - опозорить норовите biggrin.gif

Автор: andrey R 20.1.2008, 20:21

Наоборот, завидуем biggrin.gif Серёга же пишет - тема раскрыта хорошо smile.gif

Автор: инж323 20.1.2008, 20:31

Можно и Олимпиаду устроить по задачкам.Дабы моск занять.
Вот ивариант:

Автор: ViC 20.1.2008, 20:35

Такие самолет выпускают в Харькове. Это Ан-74Тк-300. Я была внутри такого на выставке, в день открытых дверей на авиазаводе.

Автор: AAANTOXA 20.1.2008, 20:49

А что Вы делали в день откр дверей на авиазаводе?
Жаль не устраивают день открытых дверей на колбасных фабриках biggrin.gif


А самоль - серийный - не взлетит . А если пофантазировать и представить что у серийного винтового самолётика мощность движка "не ограничена" - взлетит

Автор: Kult_Ra 20.1.2008, 21:06

Цитата(AAANTOXA)
А самоль - серийный - не взлетит . А если пофантазировать и представить что у серийного винтового самолётика мощность движка "не ограничена" - взлетит

А если взять ромашку и отрывать по листку? wub.gif
Был сослуживец-друг-электронщик Саша из Сердобска. И него любимая поговорка "Хорошо, что коровы не летают". А вот верблюды, говорил он, летают, но только только с кручи. dont.gif

Автор: AAANTOXA 20.1.2008, 21:13

а я и не хулиганю
просто если мощность не ограничена то винты будут все сильнее и сильнее "дуть" воздух назад на крылья (при условии что винты тянущие и соотв стоят перед крылом или "на носу" самоля а не толкающие ) . До какогото момента это будет способствовать только тому что самоль будет интенсивнее стремиться сдвинуться вперед но и тем интенсивнее будет под ним двигаться конвеер стремясь компенсировать стремление самоля )))). Но наступит момент когда скорости (обьёма) воздуха который винты отбрасывают назад на крылья хватит для создания подьёмной силы превышающей вес самоля. Тоесть самоль взлетит (подпрыгнет) что с ним дальше станет сказать сложно так как в таком положении он будет крайне неустойчив

Автор: ViC 20.1.2008, 21:57

Цитата(AAANTOXA @ 20.1.2008, 19:49) [snapback]212638[/snapback]
А что Вы делали в день откр дверей на авиазаводе?

Заглядывала, рассматривала и по крышам корпусов и цехов лазила.
Пешая прогулка по крышам на авиазаводе tongue.gif Так сказать "мне с верху видно все, ты так и знай!" tongue.gif

Автор: selin 21.1.2008, 9:40

Предыдущие посты не читал времени нет.Считаю что сила действия = силе противодействия.Самолет не взлетит,как бы не разгонялся,если конечно не вертикального взлета двигатели у него)

Цитата(инж323 @ 20.1.2008, 17:31) [snapback]212621[/snapback]
Можно и Олимпиаду устроить по задачкам.Дабы моск занять.
Вот ивариант:

В Академгородке только могут эту задачку отгадать)))Там они все такие,а нам бы чего попроще:lol:

Автор: Александр Васильев 23.1.2009, 17:52

Задача на теоретическую механику.

Дано: на транспортерной ленте, представляющей собой абсолютно ровную поверхность, стоит реактивный (винтовой) самолет. Лента движется навстречу направлению качения колес самолета при старте, со скоростью равной скорости движении самолета. Иными словами, это аналог стенда для испытания автомобилей, типа «беговой дорожки».
Коллега задал вопрос - взлетит ли самолет при этих условиях. (Транспортерная лента неограниченной длины).
Первое, что пришло мне в голову - взлетит, не взирая на встречное движение ленты, ибо точка приложения силы в случае с самолетом - не колеса, а двигатель. Двигатель развивает реактивную тягу, и самолет движется в направлении, противоположном движению реактивных газов. Если бы мы рассматривали автомобиль в аналогичных условиях, то естественно, никуда бы он не поехал, ибо скорость 1 = скорость 2. Как мне кажется, в этом случае самолет прекрасно взлетит, единственной разницей будет то, что колеса самолета будут крутиться во время его пробега по дорожке в 2 раза быстрее, и все.
Коллега утверждает, что самолет останется на месте. Но тогда возникает вопрос, во что перейдет энергия двигателей? Коллега утверждает, что в сопротивление силе трения качения.
Чувствую что я прав, но объяснить не могу, а коллега уверен в своем мнении.
Или я не прав?
bang.gif bang.gif bang.gif

Автор: Palsan 23.1.2009, 17:54

Возмите лучше вертолет, он точно взлетит. tongue.gif

Автор: Александр Васильев 23.1.2009, 17:57

Коллега, мы не на удав ком, отвечайте по сабжу))

Автор: Странная Белка 23.1.2009, 17:59

А еще лучше возьмите водки, на гитаре поиграйте, спойте и само пройдет biggrin.gif
Ой нимагу! Саша опоздал с вопросом на пару лет biggrin.gif

Автор: Александр Васильев 23.1.2009, 18:05

Имеются две силы, действующие со стороны транспортёра на самолёт — это сила трения колёс о покрытие, которая ничтожна по сравнению с тягой двигателей (но не нулевая, кроме того, есть ещё сопротивление качения в подшипниках колеса!), и сила сопротивления качению вследствие момента инерции колеса, которая зависит от радиуса и массы колеса, а также ускорения, с которым его раскручивает транспортёр — (a — ускорение транспортёра, R — радиус колеса, μ — момент инерции колеса), и теоретически может быть сколь угодно велика. Но для удержания даже самого лёгкого самолёта потребуется поддерживать невероятно большое ускорение ленты транспортёра, причём простейшие расчёты показывают, что в реальных условиях это недостижимо.

К тому же, в таком случае теоретически сколь угодно велика может быть и мощность двигателей самолёта.

Для реального колеса так же должна учитываться сила сопротивления качению вследствие деформации колеса (под весом самолёта и от центробежной силы). Но эта сила так же ничтожна по сравнению с тягой двигателей.

Вывод — транспортёр не способен удержать разгоняющийся по нему самолёт. То есть самолёт всё-таки взлетит.

Автор: Kult_Ra 23.1.2009, 18:11

"Имеются две силы,..."
tongue.gif
две противоборствующие между собой за господство над людьми власти: — «Светская» власть.

«Духовная» власть.


Цитата
везде ставят перед выбором: — «Какой строй лучше: капитализм или социализм?» — А это дурь! И там, и там — толпо-«элитарная» система, невольничий рабовладельческий строй. Только в одной пирамиде наверху «элита» либо из царской семьи, либо из бизнесменов, либо из демократов, и в другой пирамиде «элита» либо из парт.сов.номенклатуры (из «слуг народа»), либо из патриотов-националистов.

«Какая экономика лучше: плановая или рыночная?» — И это тоже дурь. Да ещё какая! План — это то, что надо сделать, чего надо достичь. И план обязательно должен быть. А рынок — один из способов достижения цели. Причём очень хороший способ. Но это всего лишь способ.
bang.gif

Автор: Patorok 23.1.2009, 18:12

А-а-а-а!!! Опять? Модераторы, удалите эту гнусную тему. Все уже давным даввно поняли, что колёса самолёта раскрутятся до скорости света и произойдёт мировой коллапс материи.

Автор: zhukov 23.1.2009, 18:14

Мое мнение, что Вы правы, т.к. при реактивной тяге взаимодействие идет между самолетом и частицами топлива (поэтому пока это единственный способ передвижения в космосе). Сила трения хоть и будет не в пользу работать (к стати она не измениться, т.к. не зависит от скорости), но она куда более мала по сравн с реактивной тягой. Тем более есть подъемная сила крыла, к-я направлена вверх, т.е. противоположная силе реакции опоры (от которой напрямую и зависит трение).
Не учел слово "качения", т.к. не помню значения. Если это подшипники, то может сопротивление и увеличется, но опять не в какое сравнение с реактивной тягой.

Автор: Александр Васильев 23.1.2009, 18:14

 Богатство.mp3 ( 2,55 мегабайт ) : 26


Вот тебе, Ириша, серенада. Сори за качество, писал на кухне на ноутбук)

Автор: zhukov 23.1.2009, 18:16

Пока писал, Вы написали очень похожий пост:)

Автор: ArFey 23.1.2009, 18:20

Я, хоть и не авиатор, но с воздухом дело имею! biggrin.gif. Поэтому попробую порассуждать: А для чего разгоняется самолет и в следствии чего он взлетает? Как я понимаю ситуацию, самолет поднимает подъемная сила крыла, возникающая при набегании на него встречного потока воздуха. И чем больше скорость набегания, - тем больше подъемная сила. В Вашем случае самолет стоит на месте. Т.е. пока он не оторвался от земли - это автомобиль с крыльями. Сами говорили, что автомобиль бы стоял на месте. А автомобили не летают, так что не отмахивайтесь, а покупайте вертолет у palsan'a. Аркадий

Автор: Александр Васильев 23.1.2009, 18:20

ВОПРОС КО ВСЕМ, КТО СЧИТАЕТ ЧТО САМАЛЬ НЕ ВЗЛЕТИТ

КУДА БУДЕТ ДЕВАТЬСЯ ЭНЕРГИЯ ДВИГАТЕЛЕЙ?
НА трение? тогда колеса аннигилируют в первую секунду экмперимента )))

Автор: Александр Васильев 23.1.2009, 18:24

Цитата(ArFey @ 23.1.2009, 18:20) [snapback]343243[/snapback]
Я, хоть и не авиатор, но с воздухом дело имею! biggrin.gif. Поэтому попробую порассуждать: А для чего разгоняется самолет и в следствии чего он взлетает? Как я понимаю ситуацию, самолет поднимает подъемная сила крыла, возникающая при набегании на него встречного потока воздуха. И чем больше скорость набегания, - тем больше подъемная сила. В Вашем случае самолет стоит на месте. Т.е. пока он не оторвался от земли - это автомобиль с крыльями. Сами говорили, что автомобиль бы стоял на месте. А автомобили не летают, так что не отмахивайтесь, а покупайте вертолет у palsan'a. Аркадий


а куда денется энергия движков самоелета в в том случае, если он останется на месте, с включенными двигателями?))) на трение уйдет? тогда колеса взорвутся, нет, аннигилируют в первую же секунду эксперимента))

Автор: Patorok 23.1.2009, 18:26

Цитата(Александр Васильев @ 23.1.2009, 19:24) [snapback]343249[/snapback]
а куда денется энергия движков самоелета в в том случае, если он останется на месте, с включенными двигателями?))) на трение уйдет? тогда колеса взорвутся, нет, аннигилируют в первую же секунду эксперимента))

На самом деле они аннигилируют мгновенно.

Автор: AAANTOXA 23.1.2009, 18:34

о щщщщщит тему подняли - конечно пятница вечер делать нечего )))))
Александр заменим самолет на более наглядное что-нибудь .
Например человек бежит по тренажеру "беговая дорожка" . Бежит быстрее - и дорожка под ним быстрее .... никуда не убежит так ведь.
... но тут движитель и двигатель не тот скажете - у самолета реактивный двигатль крылья и колеса а тут ноги.
Хорошо заменяем ноги на колеса - человека ставим на скейтборд.
реактивный двигатель - пусть на спине у него реактивный ранец маленький.
крылья - ну в руки пусть крылья какие нибудь возьмет ))))
вот эту конструкцию ставим на беговую дорожку . включаем ранец. скейтборд поехал вперед . но дорожка под ним тутже автоматически начала вращаться и скейт стоит на месте только колеса крутятся .... и так все быстрее и быстрее пока колеса не развалятся . но так как у нас задача чисто теоретическая то и разваливаться они не должны просто трение в них возрастет и все .

Автор: Александр Васильев 23.1.2009, 18:36

О! ща закрою тему ...........

представим, что вместо ленты мы имеем по два колеса, расположенными под каждым колесом самолета. типа роликов, на которых стоит самолет. получается, 6 колес.
см рисунок. Равносильно ленте.

При старте двигателей самолет просто вырвет из этих роликов, а машину - нет. В этом и разница.
Или есть мнение, что чтоб погасить энергию 4х движков, вырабатывающих по 50000 л.с., достаточно подложить ролики под колеса?))))
вариант с подачей вращения на нижние ролики ничего не меняет, ибо самолет толкается не от роликов, в вследствии реактивного движения, вот так вот!!!!!!!!!!!))



Автор: ArFey 23.1.2009, 18:38

Цитата(Александр Васильев @ 23.1.2009, 17:24) [snapback]343249[/snapback]
а куда денется энергия движков самоелета в в том случае, если он останется на месте, с включенными двигателями?))) на трение уйдет? тогда колеса взорвутся, нет, аннигилируют в первую же секунду эксперимента))

Как говорил ваш бывший президент, давайте отделим мух от котлет! Если Вы приняли допущение, что транспортер может двигаться со скоростью самолета на разгоне при взлете, необходимо точно так же допустить, что колеса самолета вращаются с беспредельной скоростью без аннигиляции. Так как вся энергия движков уходит на движение самолета вперед, а транспортер не напрягаясь возвращает его назад, следовательно энергия уходит на вращение колес! Отвечать надо сначала на вопрос сдвинется ли самолет с места при таком фантастическом транспортере! Если не сдвинется, - то и не взлетит!!! Аркадий

Автор: Александр Васильев 23.1.2009, 18:39

http://forum.abok.ru/index.php?s=&showtopic=34192&view=findpost&p=343255


Чел с ранцем поедет вперед, если движок за спиной достаточно сильный.


Антоха простой вопрос. С самолетом, куда денется энергия движков, если самаль со включенными движками никуда не сдвинется?

Автор: Palsan 23.1.2009, 18:42

Куда деваться, куда деваться ........ Уйдет на компенсацию энергии привода транспортера. В соседнем форуме-клоне тоже писать?


Автор: AAANTOXA 23.1.2009, 18:45

ладно еще нагляднее :
выкинем нафиг у самолета колеса (из-за них ИМХО все непонимание). пусть на полозьях взлетает. мощи у него вполне хватит (ну истребитель например реактивный у него тяга движков равна весу - он на полозьях по бетонке поползет как миленький).
тоесть теперь так как мы перешли от самолета к скейтбордисту то:
ставим человека на СНОУБОРДЕ (аналог полозьев) на беговую дорожку. На спине реактивный ранец - очень мощный зараза. в руках крылья. Ранец включили и человек на сноуборде попоз волоком по дорожке ........
куда энергия денется - на трение скольжения
а у колес тоже самое только трение качения

Автор: Александр Васильев 23.1.2009, 18:48

Цитата(Palsan @ 23.1.2009, 18:42) [snapback]343262[/snapback]
Куда деваться, куда деваться ........ Уйдет на компенсацию энергии привода транспортера. В соседнем форуме-клоне тоже писать?



ХВы берете огромную энергию от движков, и передаете ее на другую систему. Через что энергия перейдет к системе ТРАНСПОРТЕР? через 3 колеса самолета? Абсурд)

Цитата(AAANTOXA @ 23.1.2009, 18:45) [snapback]343264[/snapback]
ладно еще нагляднее :
выкинем нафиг у самолета колеса (из-за них ИМХО все непонимание). пусть на полозьях взлетает. мощи у него вполне хватит (ну истребитель например реактивный у него тяга движков равна весу - он на полозьях по бетонке поползет как миленький).
тоесть теперь так как мы перешли от самолета к скейтбордисту то:
ставим человека на СНОУБОРДЕ (аналог полозьев) на беговую дорожку. На спине реактивный ранец - очень мощный зараза. в руках крылья. Ранец включили и человек на сноуборде попоз волоком по дорожке ........
куда энергия денется - на трение скольжения
а у колес тоже самое только трение качения


разные условия. мы рассматривам ситуацию, когда движки работают, а самаль стоит.
куда энергия движков в этом случае перейдет?

хотя мне и так понятно, что самаль полетит, мы обсуждаем щас просто от противного. хочу вас завести в тупик)

Автор: AAANTOXA 23.1.2009, 18:52

неееет уважаемый ИМХО мы хотите взорвать нам моск ))))))
"разные условия. мы рассматривам ситуацию, когда движки работают, а самаль стоит.
куда энергия движков в этом случае перейдет?"
- на преодоление силы трения качения .
Ушел собирать наглядную модель из радиоуправляемого самолета и беговой дорожки ))))) к понедельнику доделаю

Автор: Александр Васильев 23.1.2009, 18:53

Есть одна система: самолет с двигателями и окружающее простанство, в котором работают законы физики. Самолет получает импульс от реактивных газов, и двигается вперед.
И другая система: лента транспортера, которая движется в противоположном направлении.
Если пенебречь трением качения, то смотри тогда мой рисунок,и все понятно, самолет взлетит.
Если не пренебрегать, то самолет не взлетит по единственной причине - сопростивление трению качения. И энергия движков должна перейти в другую систему. Передатчик - колеса.
Ну не абсурд?*))

Автор: ArFey 23.1.2009, 18:53

То Александр Васильев: Ни в коем разе не сочтите за переход на личности, но в этом случае Вам надо четко разграничивать свое абстрактное и конкретное мышление. В конкретном случае, да взлетит, т.к. ролики, транспортер, бригада молдаван под колесами не смогут двигаться с той же скоростью в противоположную сторону! В абстрактном случае (а именно такой была задача) то, что под колесами может двигаться (вращаться, скользить, бежать и т.д.) с той же скоростью но в противоположном направлении. Поэтому в нашей абстрактной задаче самолет с места не "сорвет". Да, кстати. На вашем рисунке с роликами правый нижний ролик должен вражаться в противоположную сторону. Аркадий

Автор: Александр Васильев 23.1.2009, 18:58

Цитата(ArFey @ 23.1.2009, 18:53) [snapback]343272[/snapback]
То Александр Васильев: Ни в коем разе не сочтите за переход на личности, но в этом случае Вам надо четко разграничивать свое абстрактное и конкретное мышление. В конкретном случае, да взлетит, т.к. ролики, транспортер, бригада молдаван под колесами не смогут двигаться с той же скоростью в противоположную сторону! В абстрактном случае (а именно такой была задача) то, что под колесами может двигаться (вращаться, скользить, бежать и т.д.) с той же скоростью но в противоположном направлении. Поэтому в нашей абстрактной задаче самолет с места не "сорвет". Да, кстати. На вашем рисунке с роликами правый нижний ролик должен вражаться в противоположную сторону. Аркадий


Никто не ответит мне на вопрос... Вот самаль на ленте час стоит,и жжет топливо. Сколько энергии выделится при этом?
И куда она денется?

Автор: Сфинкс 23.1.2009, 18:59

Положите чела с ранцем на СПИНУ. Включили реактивный движок - неужели взлетит? Подняли того чела на высоту с реактивным движком вниз - внеужели висит на месте?
Где "транспортер"? Сила тяжести.

Автор: Giedi Prime 23.1.2009, 19:06

Баянистая загадка. Про неё длаже в википедии написано. Нивзлетит он никогда.

Автор: Александр Васильев 23.1.2009, 19:07

Есть одна система: самолет с двигателями и окружающее простанство, в котором работают законы физики. Самолет получает импульс от реактивных газов, и двигается вперед.
Самолет получает импульс к движению, относительно ОКРУЖАЮЩЕГО пространства. И ему все равно, в какую сторону крутится лента, и это повлияет только на вращение колес.


Лента на встречу - колеса быстрее, лента в одном направлении с самолетом - колеса медленне крутиться будут. Лента попутно с самолетом с его скоростью - колеса ВООБЩЕ НЕ БУДУТ КРУТИТЬСЯ, НО САМОЛЕТ ВЗЛЕТИТ. biggrin.gif




Цитата(Giedi Prime @ 23.1.2009, 19:06) [snapback]343282[/snapback]
Баянистая загадка. Про неё длаже в википедии написано. Нивзлетит он никогда.


ВИКИП\ЕДИЮ люди пишут - это раз).
Там есть разные мнения по этому поводу, это 2).

Автор: ArFey 23.1.2009, 19:19

Цитата(Александр Васильев @ 23.1.2009, 18:07) [snapback]343283[/snapback]
Есть одна система: самолет с двигателями и окружающее простанство, в котором работают законы физики. Самолет получает импульс от реактивных газов, и двигается вперед.
Самолет получает импульс к движению, относительно ОКРУЖАЮЩЕГО пространства. И ему все равно, в какую сторону крутится лента, и это повлияет только на вращение колес.
Лента на встречу - колеса быстрее, лента в одном направлении с самолетом - колеса медленне крутиться будут. Лента попутно с самолетом с его скоростью - колеса ВООБЩЕ НЕ БУДУТ КРУТИТЬСЯ, НО САМОЛЕТ ВЗЛЕТИТ. biggrin.gif
ВИКИП\ЕДИЮ люди пишут - это раз).
Там есть разные мнения по этому поводу, это 2).

Да поймите Вы, самолет не ракета! Пока самолет не набрал определенной скорости, перемещаясь навстречу воздуху он не может опираться на крылья - нет подъемной силы крыла (точно так же самолет оторвался бы от земли без включенных двигателей, если бы мы могли дуть ему на крылья достаточное количество воздуха с нужной скоростью)! Он опирается на колеса Т.е. он - автомобиль. а автомобиль в нашей теоретической задаче будет стоять на месте. Аркадий

Автор: Сфинкс 23.1.2009, 19:20

Есть 2 эквивалента данной системе:
1) Человек на роликах, привязанный за канат спереди. Лента движется под ногами, а человек перебирает руками канат и двигается по канату.
2) Самолёт на поезде: поезд со скоростью и с ускорением движется в одну сторону, а самолёт в другую. Взлетит ли? Надо проверить.

Автор: BUFF 23.1.2009, 19:31

Цитата(Александр Васильев @ 23.1.2009, 19:07) [snapback]343283[/snapback]
Есть одна система: самолет с двигателями и окружающее простанство, в котором работают законы физики. Самолет получает импульс от реактивных газов, и двигается вперед.
Самолет получает импульс к движению, относительно ОКРУЖАЮЩЕГО пространства. И ему все равно, в какую сторону крутится лента, и это повлияет только на вращение колес.
Лента на встречу - колеса быстрее, лента в одном направлении с самолетом - колеса медленне крутиться будут. Лента попутно с самолетом с его скоростью - колеса ВООБЩЕ НЕ БУДУТ КРУТИТЬСЯ, НО САМОЛЕТ ВЗЛЕТИТ. biggrin.gif
ВИКИП\ЕДИЮ люди пишут - это раз).
Там есть разные мнения по этому поводу, это 2).


Цитата(Александр Васильев @ 23.1.2009, 18:58) [snapback]343275[/snapback]
Никто не ответит мне на вопрос... Вот самаль на ленте час стоит,и жжет топливо. Сколько энергии выделится при этом?
И куда она денется?

Туда же, куда и при движении самолета. Из под хвоста - в воздух. И на нагрев колес и транспортера. И никакого парадокса здесь нет, это вопрос выбора системы отчета. Вы циклитесь на механике, а смотреть нужно учебник физики за 6 класс - 1 закон Ньютона. Инерциальные системы отчета.
Относительно транспортера (условной точки на ленте транспортера) - самолет будет двигаться со скоростью V=Vсамолета+Vтранспортера, расходуя на это энергию.
Относительно земли он будет двигаться со скоростью V= Vсамолета-Vтранспортера=0

Вы задаете вопрос "куда денется энергия?", и считаете это аргументом в пользу взлета, упуская, что еще такое же количество энергии тратится, что бы самолет остался неподвижным относительно земли и воздуха. Пока он неподвижен (V=Vсамолета-Vтранспортера=0) подъемной силы не возникает, и никто никуда не летит... Задачка для 6 класса - нас тетушка(физик) такими за обедом на даче занимала...

А разные мнения, и популярность задачи - из-за бездарных (или гениально провокационных) условий задачи...
Еще можно порешать задачку на угловые скорости - чтобы доказать, что линейная скорость самолета (создаваемая двигателем), переданная на колеса, полностью компенсируется линейной скоростью транспортера... Только смысла нет - цель задачки не в этом...


Автор: izKLD 23.1.2009, 19:35

А какой диаметр роликов?
После того как колесо самолёта соскочет с роликов, будет жёсткий удар о землю, что может вызвать неисправность, т.е. 50 на 50 mellow.gif
biggrin.gif

Автор: izKLD 23.1.2009, 19:41

Сила вырабатываемая двигателем, тратится и на изгиб конструкций самолёта, а после идёт на колёса. mellow.gif tomato.gif

Автор: Александр Васильев 26.1.2009, 18:51

Цитата(BUFF @ 23.1.2009, 19:31) [snapback]343294[/snapback]
Туда же, куда и при движении самолета. Из под хвоста - в воздух. И на нагрев колес и транспортера. И никакого парадокса здесь нет, это вопрос выбора системы отчета. Вы циклитесь на механике, а смотреть нужно учебник физики за 6 класс - 1 закон Ньютона. Инерциальные системы отчета.
Относительно транспортера (условной точки на ленте транспортера) - самолет будет двигаться со скоростью V=Vсамолета+Vтранспортера, расходуя на это энергию.
Относительно земли он будет двигаться со скоростью V= Vсамолета-Vтранспортера=0

Вы задаете вопрос "куда денется энергия?", и считаете это аргументом в пользу взлета, упуская, что еще такое же количество энергии тратится, что бы самолет остался неподвижным относительно земли и воздуха. Пока он неподвижен (V=Vсамолета-Vтранспортера=0) подъемной силы не возникает, и никто никуда не летит... Задачка для 6 класса - нас тетушка(физик) такими за обедом на даче занимала...

А разные мнения, и популярность задачи - из-за бездарных (или гениально провокационных) условий задачи...
Еще можно порешать задачку на угловые скорости - чтобы доказать, что линейная скорость самолета (создаваемая двигателем), переданная на колеса, полностью компенсируется линейной скоростью транспортера... Только смысла нет - цель задачки не в этом...



1) Человек на роликах, привязанный за канат спереди. Лента движется под ногами, а человек перебирает руками канат и двигается по канату вперед.
Гениально! Тянет человек себя за канат - это эквивалент толчка от двигателей у самоелеат. И ка бы ни крутилась лента, человек все равно будет перемещаться вперед.


Автор: Странная Белка 26.1.2009, 20:05

Нарисовали бы векторов, формул бы понаписали и пришли бы к единственно верному ответу. Тут, судя по всему, двух вариантов быть не может. biggrin.gif
Саша, а серенада не грузится у меня sad.gif

Автор: Giedi Prime 27.1.2009, 19:48

Чтобы этот несчастный аэроплан взлетел, он должен двигаться относительно воздуха, а относительно воздуха он как раз неподвижен.

Автор: HasBolla 28.1.2009, 14:13

Цитата(Giedi Prime @ 27.1.2009, 19:48) [snapback]344661[/snapback]
Чтобы этот несчастный аэроплан взлетел, он должен двигаться относительно воздуха, а относительно воздуха он как раз неподвижен.

А что его держит? Отталкивается он от воздуха, а не от транспортера. Можно представить ситуацию, что самолет подвешен на веревочке над землей (ну к примеру кольцо на на тросе, как собака по двору бегает). Если запустить двигатели, то движение начнеться и он взлетит.

Автор: AAANTOXA 28.1.2009, 14:46

интересно былобы услышать мнение на этот счет извесных людей . Путина Медведева например ))))

Автор: Faraon 28.1.2009, 15:00

Мдяяя:-))) Непотопляемая тема! :-)

Автор: zeman 28.1.2009, 15:04

Путин: "Пусть попробует взлететь. Посмотрим как это у него без керосина получится"
Медведев:

Автор: проектировщег 28.1.2009, 15:04

читал только топ, остальное лень

взлетит конечно

тягу создают реактивные двигатели а не колеса

Автор: JJJJ 28.1.2009, 15:23

Самолет поднимает в воздух подъемная сила крыла. Она, в свою очередь, возникает вследствии разности давлений на плоскостях крыла. А разность давлений появляется вследствии разности скоростей набегающего потока воздуха.
Вопрос?
А сам поток воздуха будет, если самолет на движущемся транспортере?
Ну винты на крыле какой то поток создадут, но этого явно недостаточно.
Ответ - не взлетит.
Впрочем если товарищ Сталин прикажет ...

Автор: проектировщег 28.1.2009, 16:03

почитал немного

близко мнение лордаН.. задачу можно интерпретировать как угодно.. и исходя из тех или иных ограничений подвести под "взлетит" или "не взлетит"

Автор: проектировщег 29.1.2009, 9:44

а вот кстати что говорит http://ru.wikipedia.org/wiki/%D0%A1%D0%B0%D0%BC%D0%BE%D0%BB%D1%91%D1%82_%D0%BD%D0%B0_%D1%82%D1%80%D0%B0%D0%BD%D1%81%D0%BF%D0%BE%D1%80%D1%82%D1%91%D1%80%D0%B5

Цитата
Невыполнимость условий
В случае, когда и транспортёр, и самолёт покоятся относительно земли, условие о равенстве скоростей выполняется в широком диапазоне интерпретаций. Но при наличии у самолёта скорости, во многих интерпретациях условие о равенстве скоростей либо становится невыполнимым вообще, либо противоречит утверждению о направленности скорости транспортёра в направлении, обратном разгону. Т.е. попытки решить данную задачу исходя из этого условия изначально бессмысленны, как (проводя аналогию, например, из химии) попытки строить логические рассуждения исходя из условия "допустим, что спирт безалкогольный".

Автор: "Лёлик" 29.1.2009, 15:22

Судя по результатам - мнения разделились. Соответственно, я бы добавил вариант - взлетит (не взлетит) с вероятностью 50% rolleyes.gif .
Моё мнение - если бы однозначно да, то этот вариант уже существовал на упомянутых авианосцах smile.gif .
Видимо, актуально было бы для автомобиля (лодки и т.д.) с реактивным двигателем, не использующего крыло для подъёма (взлёта).
И ещё, что настораживает - пресловутое покрытие, которое в принципе можно представить как противодействующую силу, возрастающую как-то пропорционально скорости. Но в условии про ограничение мощности двигателя и прочности конструкции тоже ничего не говорится...
Так что вариант "зароется в землю" мне как-то роднее, напоминает "хотели как лучше..."

Автор: Сфинкс 30.1.2009, 15:09

Чтобы решить эту сложнейшую задачу, требуется ввести дополнительное условие, тогда ответ будет таким:
1) Понедельник. Взлетит. 2) Вторник. Не взлетит. 3) Среда. Взлетит. 4) Четверг. Не взлетит. Пятница - Воскр.: выходной!

Автор: avalanche_smk 9.2.2009, 11:31

Цитата(BUFF @ 20.1.2008, 16:17) [snapback]212497[/snapback]
твердотопливный реактивный двигатель из зала порадовал даже больше, чем винтовой самолет вертикального взлета biggrin.gif


Heinkel Lerche - проект винтового самолета с ВВП, но в 44-45 у немцев не было достаточно мощного двигателя. В дальнейшем неоднократно строились, один из последних стоит на вооружении ВВС США.

Автор: vadim999 27.2.2009, 14:35



Нет, не дожен взлететь! Так все и голосуем! Результаты голосования засекретить! А то, как в нашем пентагоне узнают про энто, потребут деньгу на разработку и строительство АПЛ-авианосцев, а наша двуглавая вертикаль власти, дасть! Типа, спасаем росийского производитель! Кризис Пы маш! А тут, мя, есчо "Люська, дура, заневестилась. И Никиточка мальнький, чуть не умер от колиту. И того купить, и сего купить, а на сотенну, только воду пить! А сырку к чайку, али ливерной, тут двусотенну, там трехсотенну! А гдеж их брать?"

Автор: andrey357 1.3.2009, 13:27

Взлетит.
Какая разница, как движется взлетная полоса? Самолет "толкается" от окружающего воздуха своими моторами и взаимодейтсвует с воздухом. Ему по барабану как движется ВПП: хорошо, если поомгает (экономится топливо)...

Автор: Akela 2.3.2009, 13:42

Я уже устал с этим бороться. Неужели никто не помнит курс теортической механики ?
Подобная задача ( формулировка другая, но суть та же) висела у нас на стенде в институте.

Ведь условие, что "скорость движения полотна равна скорости вращения колеса" означает, что скорость движения оси колеса равна 0.
А раз ось неподвижна, значит неподвижен и сам самолет.

Раз самолет неподвижен относительно земли, значит он не летит.

Кто не верит, может нарисовать эпюру скоростей колеса и все сразу станет ясно.

Автор: Ernestas 2.3.2009, 14:16

Цитата(andrey357 @ 1.3.2009, 14:27) [snapback]358366[/snapback]
Взлетит.
Какая разница, как движется взлетная полоса? Самолет "толкается" от окружающего воздуха своими моторами и взаимодейтсвует с воздухом. Ему по барабану как движется ВПП: хорошо, если поомгает (экономится топливо)...

так вот это противодействие взлетной полосы (пока колеса не оторвались rolleyes.gif ) соизмеримо и равно тяге двигателей самолета.
а самолетик относительно наблюдателя на земле - стоит!
а помести наблюдателя на движущуюся ВВП - летит!! (почти), движется так точно. biggrin.gif

Автор: Vict 2.3.2009, 14:26

Цитата(Akela @ 2.3.2009, 12:42) [snapback]358726[/snapback]
Я уже устал с этим бороться.
Неужели до сих пор(с институтских времен) так и боритесь? biggrin.gif

Автор: Akela 2.3.2009, 14:28

Цитата(Ernestas @ 2.3.2009, 14:16) [snapback]358742[/snapback]
...противодействие взлетной полосы ...равно тяге двигателей самолета.
а самолетик относительно наблюдателя на земле - стоит!
а помести наблюдателя на движущуюся ВВП - летит!! (почти), движется так точно. biggrin.gif


Совершенно верно ! Ключевая фраза - " противодействие взлетной полосы равно [ не "соизмеримо", а именно строго равно - по условию задачи] тяге двигателей самолета".
Иначе говоря - двигатель тянет самолет вперед, а движущаяся полоса тянет назад с точно такой же силой. Самолет остается на месте.

Автор: Akela 2.3.2009, 14:41

Цитата(Vict @ 2.3.2009, 14:26) [snapback]358751[/snapback]
Неужели до сих пор(с институтских времен) так и боритесь? biggrin.gif



Приходится...
Временами начинает казаться, что я наконец победил - и тут раз - опять эта тема возникает ! Так, наверно, до старости и будурассказывать про этот самолет и его колеса bang.gif

Автор: инж323 2.3.2009, 14:43

Цитата(Akela @ 2.3.2009, 14:41) [snapback]358762[/snapback]
Приходится...
Временами начинает казаться, что я наконец победил - и тут раз - опять эта тема возникает ! Так, наверно, до старости и будурассказывать про этот самолет и его колеса bang.gif

И потом уже совсем, на Новый год выступитете по ТВ и скажете- Все, я устал, я ухожу, пусть летает. biggrin.gif

Автор: Vict 2.3.2009, 14:47

Цитата(Akela @ 2.3.2009, 13:41) [snapback]358762[/snapback]
Временами начинает казаться, что я наконец победил -
Победить подобное невозможно! Можно только смириться... smile.gif
Вон тема, про выстрел из АК 47 в космосе - альтернатива этой... biggrin.gif


Цитата(инж323 @ 2.3.2009, 13:43) [snapback]358763[/snapback]
Все, я устал, я ухожу, пусть летает. biggrin.gif
biggrin.gif biggrin.gif

Автор: Akela 2.3.2009, 14:50

Цитата(инж323 @ 2.3.2009, 14:43) [snapback]358763[/snapback]
... Все, я устал, я ухожу, пусть летает. biggrin.gif


+5 ! clap.gif

Автор: elexm 3.3.2009, 10:07

Цитата(Akela @ 2.3.2009, 13:42) [snapback]358726[/snapback]
Ведь условие, что "скорость движения полотна равна скорости вращения колеса" означает, что скорость движения оси колеса равна 0.

Взлетит!!!
Колеса не причем - отвлекающий фактор. rolleyes.gif
Взлетит когда тяга двигателей превысит вес самолета.
1."Сушка" при выполнении фигуры "кобра" зависает (нулевая скорость), после чего увеличивает тягу двигателей и продолжает полет.
2.Запуск кордовых моделей самолетов ( корд - двигатель внутреннего изгорания).
Первый номер держит модель, второй номер запускает двигатель.
Когда двигатель завелся, начальная скорость 0.
Первый номер отпускает модель - самолет улетает...
Другое дело для тяжелых самолетов : Боинг если не побегает - не взлетит.

Автор: AAANTOXA 3.3.2009, 10:42

Цитата(elexm @ 3.3.2009, 10:07) [snapback]359100[/snapback]
Взлетит!!!
Колеса не причем - отвлекающий фактор. rolleyes.gif
Взлетит когда тяга двигателей превысит вес самолета.
1."Сушка" при выполнении фигуры "кобра" зависает (нулевая скорость), после чего увеличивает тягу двигателей и продолжает полет.
2.Запуск кордовых моделей самолетов ( корд - двигатель внутреннего изгорания).
Первый номер держит модель, второй номер запускает двигатель.
Когда двигатель завелся, начальная скорость 0.
Первый номер отпускает модель - самолет улетает...
Другое дело для тяжелых самолетов : Боинг если не побегает - не взлетит.

мдя
1) не видел такой фигуры (кобра Пугачева видел а просто кобра - не знаю вас из дас) но нормальный (не максимальный) взлетный вес Су 30 например - 26 тонн. А максимальная форсажная тяга 2х12,5 тонн. Меньше единицы - тоесть вертикально вверх он лететь как ракета не может.
2) гениальные выводы. тут люди пишут про векторы сил ... "боинг если не побегает он не взлетит" - ну правильно Бобик он на то и Бобик )))
-----
по умолчанию тяга двигателя паралельна плоскости - в таком случ подьемую силу создает крыло .
если вектор тяги двигателей направить вверх под каким-то углом и соотношение тяга/вес больше единицы то естессно самолет просто как ракета улетит без пробега. Что и происходит со всякими кордовыми и радиоуправляемыми ....

Автор: elexm 3.3.2009, 11:32

Су-27 (Су-30)
ДВИГАТЕЛИ.
2xТРДДФ АЛ-31Ф (2x125,5 кН/2x12800 кгс).
П-42 оснащен форсированным ТРДДФ АЛ-31Ф (2x133,3 кН/2x13600 кгс).
МАССЫ И НАГРУЗКИ, кг:
- нормальная взлетная 22500 - 23000 (с неполной заправкой топлива в конфигурации истребителя завоевания господства в воздухе);

С сайта:
http://www.aviaport.ru/directory/aviation/300.html

Автор: AAANTOXA 3.3.2009, 12:08

да фиг с ним не в конкретной модели самолета дело ))))))) Су , Миг , F ..... Даже если тяга в 2 раза больше веса . Самолет все равно на взлетной полосе стоит "ГОРИЗОНТАЛЬНО".
возьмите ракету какую нибудь . "положите" ее горизонтально на транспортер и пусть она попробует взлететь

Автор: Vict 3.3.2009, 12:14

Akela, возвращайтесь!!! biggrin.gif

Автор: elexm 3.3.2009, 12:16

Возьмите спичку, обмотайте ее плотно фольгой (от сигарет), второй зажженой спичкой погрейте фольгу. Если все аккуратно сделано (требуется небольшая практика) улетает спичка...
Вместо ракеты возьмите ПТУРС - улетает, даже если БМП движется задом...

Автор: AAANTOXA 3.3.2009, 12:49

ph34r.gif я надеюсь что он провокатор потому что если он реально так думает то от инженерии лучше отлучить ))))))

Автор: elexm 3.3.2009, 13:34

Цитата(AAANTOXA @ 3.3.2009, 12:08) [snapback]359165[/snapback]
да фиг с ним не в конкретной модели самолета дело ))))))) Су , Миг , F ..... Даже если тяга в 2 раза больше веса . Самолет все равно на взлетной полосе стоит "ГОРИЗОНТАЛЬНО".
возьмите ракету какую нибудь . "положите" ее горизонтально на транспортер и пусть она попробует взлететь

Дело, таки в модели. Тежелым самолетам без пробежки не взлететь.
"Катюша" стреляет прямой наводкой - пример полета при горизонтальном старте.

Цитата(AAANTOXA @ 3.3.2009, 12:49) [snapback]359190[/snapback]
ph34r.gif я надеюсь что он провокатор потому что если он реально так думает то от инженерии лучше отлучить ))))))

Безпочвенные надежды, выбросьте из головы. rolleyes.gif

Автор: Taras 3.3.2009, 16:03

Внимательней вчитывайтесь в сам вопрос.

Цитата
Внимание, вопрос: сможет ли самолет разбежаться по этому полотну и взлететь?"

Т.е. получается, что изначально сказано: если самолет разбежится - то он взлетит, если не разбежится - не взлетит. По условию задачи - самолет, получается, стоит на месте, т.к. его горизонтальная скорость равна нулю. Следовательно - не разбежится, значит не взлетит. Все просто, ответ в самой задаче, и не надо выдумывать про всякие лыжи обратной стреловидности, подводные лодки на беговой дорожке и "катюши" с горизонтальным взлетом.=)

Автор: AAANTOXA 3.3.2009, 16:10

Цитата(elexm @ 3.3.2009, 13:34) [snapback]359215[/snapback]
Дело, таки в модели. Тежелым самолетам без пробежки не взлететь.
"Катюша" стреляет прямой наводкой - пример полета при горизонтальном старте.

ужос уууужосссс
тоесть подьемная сила у тяжелого и легкого самолета создается по разным законам физики ?
----
стрелять прямой наводкой это вовсе не значит держать ствол (направляющие пусковой установки ракеты) горизонтально. Стрелять прямой наводкой это значит что цель находится в зоне прямой видимости а не за горизонтом(препятствием).

Автор: elexm 3.3.2009, 16:25

Цитата(Taras @ 3.3.2009, 16:03) [snapback]359317[/snapback]
Внимательней вчитывайтесь в сам вопрос.

Т.е. получается, что изначально сказано: если самолет разбежится - то он взлетит, если не разбежится - не взлетит. По условию задачи - самолет, получается, стоит на месте, т.к. его горизонтальная скорость равна нулю. Следовательно - не разбежится, значит не взлетит. Все просто, ответ в самой задаче, и не надо выдумывать про всякие лыжи обратной стреловидности, подводные лодки на беговой дорожке и "катюши" с горизонтальным взлетом.=)

При внимательном рассмотрении получается:
разбежаться не сможет т.к. корень слова бег.
(Бег — один из способов передвижения (http://ru.wikipedia.org/wiki/%D0%9B%D0%BE%D0%BA%D0%BE%D0%BC%D0%BE%D1%86%D0%B8%D1%8F) человека и животных)

Цитата(AAANTOXA @ 3.3.2009, 16:10) [snapback]359323[/snapback]
стрелять прямой наводкой это вовсе не значит держать ствол (направляющие пусковой установки ракеты) горизонтально. Стрелять прямой наводкой это значит что цель находится в зоне прямой видимости а не за горизонтом(препятствием).

Считаете, что невозможно установить пусковую установку горизонтально ?
Имелось ввиду горизонтальное расположение установки перед пуском.

Автор: Taras 3.3.2009, 16:47

Цитата(elexm @ 3.3.2009, 16:25) [snapback]359336[/snapback]
При внимательном рассмотрении получается:
разбежаться не сможет т.к. корень слова бег.
(Бег — один из способов передвижения (http://ru.wikipedia.org/wiki/%D0%9B%D0%BE%D0%BA%D0%BE%D0%BC%D0%BE%D1%86%D0%B8%D1%8F) человека и животных)

Не путайте с локомоцией. "Разбег" - термин, обозначающий расстояние, необходимое самолету для разгона и взлета. Не уходите от темы. Просто Вам по делу ответить нечего.

Автор: Taras 3.3.2009, 17:01

Цитата(AAANTOXA @ 4.11.2006, 15:06) [snapback]57012[/snapback]
Согласен
он полетит!
а я больше не буду отвечать в этой теме - век зарплаты не видать!

Антоха, Вам еще век за просто так работать.

Автор: AAANTOXA 3.3.2009, 17:02

Цитата(Taras @ 3.3.2009, 17:01) [snapback]359365[/snapback]
Антоха, Вам еще век за просто так работать.

о блин забыл о зароке!!!!!!!!!!!!
а я-то думаю чтож мне не платят уже 2 месяца )))))))

Автор: elexm 3.3.2009, 17:03

Не уходя от темы отметим, что было три вопроса :

Цитата(Bers @ 3.11.2006, 7:20) [snapback]56716[/snapback]
Внимание, вопрос: сможет ли самолет разбежаться по этому полотну и взлететь?"
Ну дык что - Взлетит или не взлетит?
Есть версии?

Если обсуждать вопрос "Взлетит или не взлетит?"
Взлетит!
Законы физики одинаковы для разных летательных аппаратов to AAANTOXA.
1.Очень легкие аппараты ( кордовые, радиоуправляемые...) согласитесь взлетают без разбега.
2.Более тяжелые - ракеты могут взлетать без разбега.
3.Самолеты с минимальным весом и максимальной тягой так же могут взлетать без разбега.
Или, что то не так ?

Автор: AAANTOXA 3.3.2009, 17:17

гы гы вынудил )))
рис 1 . крыло . подъемная сила создается изза разности профилей. воздух вынужден омывать верхнюю часть крыла быстрее чем нижнюю. выше скорость - меньше давление .
рис 2 . самоль ))) движок толкает его вперед. он катится. но под ним транспортер вращает покрытие навстречу. Сила трения в колесах, подшипниках, между колесами и покрытием ... эта сила трения создает систаму самолет-колесо-полотно. Добавление тяги двигателя приводит лишь к увеличению скорости транспортера и все . - обдува крыла нет - подъёмной силы нет .

 

Автор: elexm 3.3.2009, 17:22

А если на втором рисунке поставить ракету (скажем на колесах), а не самолет?
Считаете, что не полетит в горизонтальном направлении ?


Автор: Ernestas 3.3.2009, 17:28

Цитата(Сергей В. @ 3.11.2006, 19:24) [snapback]56926[/snapback]
http://ru.wikipedia.org/wiki/%D0%97%D0%B0%D0%B4%D0%B0%D1%87%D0%B0_%D0%BF%D1%80%D0%BE_%D1%81%D0%B0%D0%BC%D0%BE%D0%BB%D1%91%D1%82


"Задача знаменита тем, что часто публикуется провокаторами в каком-либо из интернет-форумов с целью поднять флейм. Почти все, кто продолжительное время активно пользуются интернет-форумами, хотя бы раз сталкивались с этой задачей и порождённым ею спором. Чтобы убедиться в широком распространении этой задачи, достаточно воспользоваться каким-либо поисковым сервисом, задав поиск по сочетанию слов «самолёт» и «транспортёр».

Условия задачи в описанной выше постановке неполны и некорректны, что создаёт неоднозначность интерпретации. Эта неоднозначность и является одной из основных причин возникающих споров. Участники спора, как правило, подходят к задаче с некоторым набором допущений и начальных условий, отстаивают тот вариант ответа, который считают правильным в рамках собственной модели, и не задумываются о множественности интерпретаций (либо только делают вид, если хотят поддержать провокацию).

При этом имеет место так называемая цепная реакция, когда обилие комментариев к этой задаче создаёт у участников форума интерес к этой ветке, а затем и желание высказать свою точку зрения."

меткое охариктизование всего этого обсуждения.
ИМХО.

Автор: elexm 4.3.2009, 9:26

Молодец AAANTOXA
Инженер мыслит карандашом.
Только рисовать надо так:
Чтобы струя двигателя была направлена ВНИЗ и назад.
Тогда при тяге меньше веса самолета, он будет катиться по транспортеру.
Когда тяга двигателя превысит вес самолета произойдет отрыв колес от транспортера.
После отрыва колес от транспортера.
Что дальше будет с транспортером - неважно, а самолет получит линейную скорость и начнет работать подъемная сила крыла.
Внимание ответ : разбежаться по этому полотну самолет не сможет, а взлететь - сможет.
*При условии, что конструкция двигателя (вектор тяги и мощность) обеспечат отрыв колес от полотна.
Тему можно закрывать ???

 

Автор: Akela 4.3.2009, 9:29

Цитата(elexm @ 3.3.2009, 17:22) [snapback]359383[/snapback]
А если на втором рисунке поставить ракету (скажем на колесах), а не самолет?
Считаете, что не полетит в горизонтальном направлении ?



Вы меня в могилу сведете ! bang.gif

Но прогресс есть - народ уже согласился с тем, что самолет не будет разбегаться. Осталось понять - способен ли он взлететь "с места", как тот же ПТУР, например ?

Отвечаю - нет. На стартующую с места ракету действует одна сила - сила тяги двигателя ( ну еще конечно сопротивление воздуха, трение о направляющие - но они существенно меньше тяги двигателя и ими пренебрегаем).

На наш же самолет с самого момента старта действуют 2 одинаковые по величине, но протовоположно направленные силы - тяга двигателя и тяга транспортера. Результирующая их равно 0, следовательно самолет остается в покое.

( Напоминаю : "Если результирующая действующих на тело сил равна 0, то тело находится в покое либо в состоянии равномерного прямолинейного движения." В нашей задаче начальная скорость =0, следовательно такой и останется).

Никакой неполности и некорректности в условии задачи нет, есть просто много людей, которые в институте не на лекции ходили, а пиво пили на горке. smile.gif

Автор: Akela 4.3.2009, 9:57

Цитата(elexm @ 4.3.2009, 9:26) [snapback]359673[/snapback]
...Когда тяга двигателя превысит вес самолета произойдет отрыв колес от транспортера.
[...*При условии, что конструкция двигателя обеспечат отрыв колес от полотна.
Тему можно закрывать ???


Вы правы, но не совсем. Отрыв колес произойдет когда не тяга двигателей будет больше веса самолета, а проекция вектора тяги на вертикаль.
Таким образом мы получили что ? Правильно, самолет вертикального взлета. Несомненно, такой самолет взлетит с любого транспортера. rolleyes.gif

Автор: инж323 4.3.2009, 13:48

Цитата(Akela @ 4.3.2009, 9:29) [snapback]359675[/snapback]
На наш же самолет с самого момента старта действуют 2 одинаковые по величине, но протовоположно направленные силы - тяга двигателя и тяга транспортера. Результирующая их равно 0, следовательно самолет остается в покое.

Заодно тут стоит упомянуть Ньютона.По его закону сила взаимодействия лошади и телеги одинаковы и разнонаправлены.
Отчего же телега порой тянет лошадь в "свою" сторону? biggrin.gif
Только вот самолет то взаимодействует с воздухом, а колеса так просто прикручены,дабы днищще не корябать. biggrin.gif

Автор: AAANTOXA 4.3.2009, 14:05

Цитата(Akela @ 4.3.2009, 9:57) [snapback]359689[/snapback]
Вы правы, но не совсем. Отрыв колес произойдет когда не тяга двигателей будет больше веса самолета, а проекция вектора тяги на вертикаль.
Таким образом мы получили что ? Правильно, самолет вертикального взлета. Несомненно, такой самолет взлетит с любого транспортера. rolleyes.gif

более того: если проекция вектора тяги на вертикаль будет больше в каком-то диапазоне например навскидку от 1,1 до 3 самоль все равно не взлетит. При этом условии он отрывается от пов-ти , начинает двигаться вперед и вверх но так как обдува клыла достаточного нет то нос (в данном случае реактивный самоль с толкающим движком в хвосте) просто сваливается вниз и самоль снова "падает" на полосу. Потом снова подскакивает...
Для того чтобы он ВЗЛЕТЕЛ необходимо условие : за малое время после отрыва от полосы он должен разогнаться до скорости при которой появится подьемная сила крыла.

Автор: elexm 5.3.2009, 11:35

Цитата(AAANTOXA @ 4.3.2009, 14:05) [snapback]359825[/snapback]
более того: если проекция вектора тяги на вертикаль будет больше в каком-то диапазоне например навскидку от 1,1 до 3 самоль все равно не взлетит. При этом условии он отрывается от пов-ти , начинает двигаться вперед и вверх но так как обдува клыла достаточного нет то нос (в данном случае реактивный самоль с толкающим движком в хвосте) просто сваливается вниз и самоль снова "падает" на полосу. Потом снова подскакивает...

Безконечно это продолжаться не может!
rolleyes.gif rolleyes.gif rolleyes.gif

 

Автор: Giedi Prime 5.3.2009, 23:37

biggrin.gif .

Самолёт может взлететь, если реактивная струя будет вертиклаьно вниз направлена, ну типа как у Харриера. Тогда и транспортёр этот вообще не нужон.

А вобще, тему эту закрыть уже пора.

Автор: Serg Ivanov 14.4.2009, 12:19

Оно имеет систему управления, которая отслеживает и подстраивает скорость движения полотна таким образом, чтобы скорость вращения колес самолета была равна скорости движения полотна. (С)
Что сие означает?
Словоблудие. Как скорость вращательного движения может быть равна скорости поступательного?
А если самоль на лыжах? wink.gif

Автор: инж323 14.4.2009, 14:13

Точно,надо про лыжи вариант развить!
Самолет-гидроплан на лыжах взлетает с поверхности горной реки, скорость которой регулирует дядька, что б самолет стоял на одном месте относительно, деревца на берегу.Взлетит или не взлетит? biggrin.gif Возраст дядьки- 35-40 лет если что.

Автор: проектировщег 14.4.2009, 15:34

а дядька под чем?.. коксом.. героином?.. а мож он плановОй?..

Автор: Young 22.4.2009, 15:12

Кто смотрел дискавери "разрушители легенд" недавно, тот мог увидеть как они это все вживую проделали.

Автор: Машинист 22.4.2009, 15:43

Что ? newconfus.gif Гидроплан под коксом держали, чтоб не улетел ? ))))
Я смотрел их пару раз, веселые ребята, но такого не видел.

Автор: инж323 22.4.2009, 16:58

Цитата(проектировщег @ 14.4.2009, 16:34) [snapback]378242[/snapback]
а дядька под чем?..

А вдруг кому для решения задачи понадобится возраст дядьки?А мы тут сразу, ага, а вот и указали. biggrin.gif

Автор: Майкл 22.4.2009, 19:54

да штоб он развалился уже, аэроплан этот, штоб у него колеса отвалились, а летчик уже пошел домой водку пить, как все нормальные люди.

Автор: Young 22.4.2009, 22:45

Цитата(Машинист @ 22.4.2009, 16:43) [snapback]381149[/snapback]
Что ? newconfus.gif Гидроплан под коксом держали, чтоб не улетел ? ))))
Я смотрел их пару раз, веселые ребята, но такого не видел.

Сначала поставили действующую модель самолета на движущийся в противоположном направлении транспортер - самолетик взлетел. Они на этом не успокоились, нашли мужика с небольшим НАСТОЯЩИМ самолетом, поставили этот самолет на длинный кусок брезента, брезент прицепили к пикапу и провели тот же эксперимент - самолет ехал в одну сторону со скоростью по-моему 40км/ч, пикап тащил брезент из-под самолета в противоположную сторону с такой же скоростью. Самолет взлетел. biggrin.gif

http://rutube.ru/tracks/831433.html?v=21b6c13073683a8ab6144fee5d02d05c - во кстати, нашел серию на рутьюбе.

Автор: Akela 23.4.2009, 8:20

Цитата(Young @ 22.4.2009, 23:45) [snapback]381282[/snapback]
.....- самолет ехал в одну сторону со скоростью по-моему 40км/ч, пикап тащил брезент из-под самолета в противоположную сторону с такой же скоростью. Самолет взлетел.
[



Не зря этого хрена в берете из НАСА выгнали - если у них там все такие инжЫнеры, остается только удивляться, что еще не все Шаттлы разбились...

Покажите мне НАСТОЯЩИЙ самолет со скоростью отрыва 40 км\ч ?!

Эта передача грешит тем же, что и многие другие из этой серии - некорректной постановкой эксперимента. Рассматриваемая задача потому и рассматривается в курсе ТЕОРЕТИЧЕСКОЙ механики, что невозможно на практике соблюсти ее условия. Таковой эксперимент может быть только умозрительным.

Автор: Kult_Ra 23.4.2009, 9:20

"Покажите мне НАСТОЯЩИЙ самолет со скоростью отрыва 40 км\ч ?!"

или
Унесите меня в степь и укройте туманом
Дайте в руки мне гармонь - золотые планки
***
бывает левитация и гравитация?
Бывает!
biggrin.gif
И верблюд летает, но только с кручи
***
Без левитации надо бы выполнить работу - преодолеть гравитацию. clap.gif
Подъемная сила - "ветер" 40 км/час ("V" квадрат на плотность= ветровое давление) и его вектор на площадь (крыло под углом). Тут и масса самолета своё скажет (из бумаги самолетик?) как будет "скользить по слоям воздуха уплотненного".

Но "неверие" большинства не мешает одиноким гениям "прогрессить" ohmy.gif тоже надо учитывать!
НАСА не наша и там народ не чета тутошним. tomato.gif
кто-то, сердешный, уже пошел (отсюда прямо?) домой водку пить (вне дома рискованно?), как все нормальные люди (не ожидая пятницы?)




Автор: Bers 23.4.2009, 10:01

На авиационных форумах, не говоря уже о всяких прочих ресурсах, мега-тема про самолёт уже давно канула в прошлое:-) И только на АВОКе её продолжают терзать пытливые инженерные умы:-)))

Автор: Young 23.4.2009, 21:47

Цитата(Akela @ 23.4.2009, 9:20) [snapback]381336[/snapback]
Не зря этого хрена в берете из НАСА выгнали - если у них там все такие инжЫнеры, остается только удивляться, что еще не все Шаттлы разбились...

Покажите мне НАСТОЯЩИЙ самолет со скоростью отрыва 40 км\ч ?!

Эта передача грешит тем же, что и многие другие из этой серии - некорректной постановкой эксперимента. Рассматриваемая задача потому и рассматривается в курсе ТЕОРЕТИЧЕСКОЙ механики, что невозможно на практике соблюсти ее условия. Таковой эксперимент может быть только умозрительным.

вы слов "по-моему" в моем посту не заметили? tongue.gif Я написал, как запомнил, а ниже ссылку привел этого эксперимента, сами посмотрите и увидите на какой скорости взлетал их самолет. Я не говорю, что они истина в последней инстанции, но хоть на сам эксперимент посмотреть в живую интересно. smile.gif

Автор: инж323 23.4.2009, 21:57

Цитата(Akela @ 23.4.2009, 9:20) [snapback]381336[/snapback]
Не зря этого хрена в берете из НАСА выгнали

Конечно не зря!Его бы и у нас из любого НИИ выгнали.Женщины таких не любят,эт раз.А мужики с ним даже пиво пить не пойдут.
Представьте такого товарица в пивной.Стоит , блин, лысый, потнючий, в усах космами и на этих усах пена пивная.С ним же загребут сразу, полкружки не выпьешь.Ну и как с ним? Значит и мужики его не поддержат.И выгонят его и никто не пожалеет.Ни замуж за него женщинам не сходить, ни мужикам с ним пива не попить.Вот он и ерундой всякой занимается.

Автор: Serg Ivanov 24.4.2009, 10:27

Цитата(Akela @ 23.4.2009, 8:20) [snapback]381336[/snapback]
Покажите мне НАСТОЯЩИЙ самолет со скоростью отрыва 40 км\ч ?!

Смотрите например http://www.oskbes.ru/890.htm#data
+20км/ч встречный ветер smile.gif

Автор: Akela 24.4.2009, 10:55

Цитата(Serg Ivanov @ 24.4.2009, 11:27) [snapback]381791[/snapback]
+20км/ч встречный ветер smile.gif



Ага, а давайте примем ветер 200км/ч .... smile.gif

Автор: AAANTOXA 24.4.2009, 14:05

Цитата(Young @ 22.4.2009, 23:45) [snapback]381282[/snapback]
во кстати, нашел серию на рутьюбе.

фигня. там у них совершенно другие условия . там транспортер движется СО СКОРОСТЬЮ ОТРЫВА самолета а самолю не возбраняется разгоняться быстрее. Получили что и с игрушечным и с настоящим самолем четко видно что они двиигались поступательно относительно неподвижных предметов на земле . тоесть был обдув крыла набегающим потоком воздуха.


Автор: Serg Ivanov 24.4.2009, 15:09

Цитата(Akela @ 24.4.2009, 10:55) [snapback]381812[/snapback]
Ага, а давайте примем ветер 200км/ч .... smile.gif

А зачем? Для Авиатики достаточно 20км/час.
Для других ЛА- вообще не надо. В лом искать.. Есть самоли и с 40км/ч.
Тем более, что в том сюжете тележурналюги скорее всего перепутали при переводе текста км с милями. biggrin.gif

Автор: Giedi Prime 24.4.2009, 15:34

Самолёт бы мог взлететь не двигаясь с места, если на него будет дуть ветер (из какой-нибудь аэротрубы) равный подъёмной силе нак рыле, ну т.е. как тут говорят со скоростью отрыва. А сзати сделать какой-то упор, чтобы самолёт не снесло просто. Вспомните, как в школе с листиками бумаги забавлялись - сзади рукой делаешь упор, а спереди дуешь, листик будет подниматься.
Только в этом случае самолёт сможет взлететь.

Автор: sofo 13.10.2009, 12:59

если принять что правильный ответ это - правильного ответа не существует - то, можно и я добавлю свою бочку дегтя

рассмотрим данную задачу с другой стороны
по условию, колеса самолета крутятся с такой же скоростью что и транспортер, то есть не важно какая скорость ноль или бесконечность, стоит самолет или аннигилируется, важно то что самолет стоит на месте относительно наблюдателя

а вот дальше начинаются проблемы: куда девается тяга создаваемая двигателем?
для половины случаев это можно объяснить
1) если самолет винтовой то можно сказать, что сзади набегает поток воздуха по мощности равный мощности создаваемого потока пропеллером в спокойном воздухе и соответственно тяга равна нулю (как-то так smile.gif )
2) если реактивный - куда-то еще (пока не придумал) bang.gif

в итоге имеем следующий результат:
снижение неопределенности на 50% - винтовой точно не полетит smile.gif tomato.gif

Автор: AAANTOXA 13.10.2009, 15:12

специально ведь зарегился для этого biggrin.gif
-------
вообще это злосная тема, ее надо удалить

Автор: Кляо1 8.12.2009, 16:52

И все таки самолет взлетит, по крайней мере в воздухе зависнет, поскольку относительно наблюдателя он будет стоять на месте, но двигателем создаст поток воздуха вокруг себя и потихоньку оторвется от этого транспортера, а далее - аки птица. А что по этому поводу думают авиаторы?

Автор: Bers 8.12.2009, 16:54

Дело о взлетающем самолёте живёт и процветает:-)))

Автор: Машинист 8.12.2009, 17:07

Ага )

Цитата
И все таки самолет взлетит, по крайней мере в воздухе зависнет, поскольку относительно наблюдателя он будет стоять на месте, но двигателем создаст поток воздуха вокруг себя и потихоньку оторвется от этого транспортера, а далее - аки птица. А что по этому поводу думают авиаторы?

Фига с два он взлетит. Вентилятором поработает, и все.
Ему чтоб от земли оторваться - должна подъемная сила на крыле возникнуть. А для этого надо разогнаться. А транспортер назад везет (то есть на месте удерживает).

Автор: Рекуператор 11.1.2010, 14:08

создали условия что бы колёса крутились ,ну и будут крутиться колёса.............. rolleyes.gif ,а условия для влёта самолёта не создали! с чего вдруг взлёт....... blink.gif

С другой стороны если самолёт для вертикального взлёта сделан,а транспортёр просто для проверки колёс самолёта................тогда взлетит и улетит и при посадке колёса точно будут крутиться ,так как при взлёте была проверка rolleyes.gif Ну просто взлёт такой .....с директивой "перед взлётом особое внимание уделить вращению колёс самолёта "

Цитата(Bers @ 3.11.2006, 7:20) *
Есть такая задачка:

"Самолет (реактивный или винтовой) стоит на взлетной полосе с подвижным покрытием (типа транспортера). Покрытие может двигаться против направления взлета самолета, т.е. ему навстречу. Оно имеет систему управления, которая отслеживает и подстраивает скорость движения полотна таким образом, чтобы скорость вращения колес самолета была равна скорости движения полотна.
Внимание, вопрос: сможет ли самолет разбежаться по этому полотну и взлететь?"

Разбежаться......?как если мы ему(условием) транспортёром не даём разбегаться ?

Цитата(Bers @ 3.11.2006, 7:20) *
Есть такая задачка:


Видел эту задачку на нескольких форумах и практически нигде не пришли к единому мнению. Кстати, на одном форуме вопрос обсуждается уже полгода... Но поскольку у нас форум, так


что для нас пол года !!! laugh.gif Вот 10 лет это да.......... !!!!!!!!!!!!

Автор: HeatServ 11.1.2010, 14:25

Цитата(Рекуператор @ 11.1.2010, 14:03) *
Разбежаться......?как если мы ему(условием) транспортёром не даём разбегаться ?

Не полетит самолёт наш, он ввиду своей геометрии ограничен в подаче воздуха на крыло, даже если транспортёр остановится - самолётик только и начнёт что натужно разбегаться, как это они делают на взлётке, и пока не возникнет подъёмной силы равной его весу, для чего нужно много набегающего на крыло воздуха, так и бегать ему по взлётке. Вертолёт бы взлетел на определённой скорости, у него вектор вертикальной тяги больше.

Да и вообще, если бы он взлетел, то давно бы именно так и взлетали, во всяком случае с авианосцев.

Автор: Vano 11.1.2010, 14:32

http://www.youtube.com/watch?v=0ul_5DtMLhc&fmt=18 опыты на кошках

Автор: Vano 11.1.2010, 14:38

На некоторых форумах лечат прививкой http://lj.rossia.org/users/anton222/685.html
Значение задачи - задача является Интернет-мемом, "запущенным" в сеть 24.07.2003 на форуме avia.ru

Создает эффект строительства Вавилонской башни: каждый начинает отстаивать свою точку зрения, правильное решение в возникающем хаосе найти невозможно. Имеет свойство "заражать" форумы бесконечным обсуждением.
Настоящий "антивирус" для мема "Самолёт на транспортёре" написан по материалам сайта Википедия. Рекомендуется в качестве последней записи с последующей блокировкой форума) Благодарности всем создателям статьи "Самолёт на транспортёре" и участникам обсуждений, а также персонально Tetromino, Trycatch, Ausweis, Ace, Pasteurizer, infovarius и Дяде Фреду.

Автор: AAANTOXA 11.1.2010, 14:42

техника развивается
вот взлетает с пробегом метров 10-15 . движок еще помощнее и взлетит вообще без пробега
http://rutube.ru/tracks/1852605.html?v=e599cf39cb59347d11098ff8491a4b20
но это уже самолет вертикатьного взлёта просто получается а таких и так хватает
просто вертикальный взлет осуществляется традиционным способом а не поворотными соплами и тп

Автор: HeatServ 11.1.2010, 14:54

Цитата(Vano @ 11.1.2010, 14:38) *
[i]Значение задачи - задача является Интернет-мемом, "запущенным" в сеть 24.07.2003 на форуме avia.ru

Создает эффект строительства Вавилонской башни

То есть правильный ответ №5 из голосования?

Автор: toddd 11.1.2010, 16:34

Эти умники, всю прелесть общения погубят.
Скажите, взлетит ли самолет на движущемся конвейере в обратную сторону? bleh.gif

Хвостом вперед

Автор: Рекуператор 11.1.2010, 17:57

Цитата(Vano @ 11.1.2010, 14:32) *
http://www.youtube.com/watch?v=0ul_5DtMLhc&fmt=18 опыты на кошках

видно же что не создали условий!!!!!!!Медленно раскручивать надо!!!!!!!!!

Автор: Kult_Ra 11.1.2010, 18:09

Цитата
Создает эффект строительства Вавилонской башни: каждый начинает отстаивать свою точку зрения, правильное решение в возникающем хаосе найти невозможно. Имеет свойство "заражать" форумы бесконечным обсуждением.

Когда-то "Система" в лице "Берия и К°" всех таких талантливых и прыткоумных отслеживала, да в "шарашку" какую-либо втыкала - двигать "науку" исключительно на пользу "рабочему классу"!

пс.
http://www.patriotica.ru/history/parshev_beria.html, господа коллеги"! Вдруг "шарашка" где-то рядом biggrin.gif

Цитата
Потом Термен жил в Америке, потом сидел в "шарашке". Так вот когда Берия его спросил, сможет ли он сделать атомную бомбу, он заявил, что сможет. А на вопрос о том, что ему для этого нужно, ответил, что "персональную машину с шофёром и полторы тонны стального уголка".

Автор: vit37 11.1.2010, 20:20

Да, подъемной силы нет и полета тоже не будет..... rolleyes.gif

Автор: shanetnavigator 12.1.2010, 14:38

спасибо авторам задачи и отдельно Bers, открывшему тему.
уже больше 5 лет слежу за форумами АВОК (и не только за ними); решил зарегистрироваться только для того, чтобы пригласить участников данного обсуждения сюда:

буду рад видеть всех оппонентами более интересных обсуждений.

з.ы. представляете, сколько времени потрачено на обсуждение задачи без решения!!! а могли бы в это время помочь своим заказчикам...

Автор: Vano 12.1.2010, 15:02

Цитата(shanetnavigator @ 12.1.2010, 14:38) *
з.ы. представляете, сколько времени потрачено на обсуждение задачи без решения!!! а могли бы в это время помочь своим заказчикам...

у тех у кого деньги есть помогаем, вне очереди

Автор: Рекуператор 12.1.2010, 15:45

Цитата(Vano @ 12.1.2010, 15:02) *
у тех у кого деньги есть помогаем, вне очереди

золотые слова! rolleyes.gif

Автор: Рекуператор 12.1.2010, 15:50

Цитата(shanetnavigator @ 12.1.2010, 14:38) *
решил зарегистрироваться только для того, чтобы пригласить участников данного обсуждения сюда:
http://www.relativity.ru/


сломайте себе голову пожалуйста.


Автор: инж323 12.1.2010, 16:18

После того , как Ньютона отоварило здоровенным яблоком, он начал кидать с башни ядро и пушинку.Но самое примечательно, что он смог увидеть, что они упали одновременно.
А про взлетит можно судачить бесконечно, как и про сможет ли Бог создать камень, который не сможет поднять. Или платформу эту под самолет, или такой вакуум, что пушинка и ядро упали одновременно.

Автор: Kult_Ra 12.1.2010, 20:44

Цитата(Vano @ 12.1.2010, 15:02) *
у тех у кого деньги есть помогаем, вне очереди
себе даже мы поможем уж тогда и только тогда, когда деньги будут! huh.gif

Автор: Composter 26.1.2010, 13:17

самолет не взлетит,т.к. нет потока воздуха который бы создавал бы разряжения воздуха над крылом.
но есть верояность что поток воздуха ,создаваемый за счет вращения колес будет очень большим и проявиться экранный эффект http://ru.wikipedia.org/wiki/Экранный_эффект и вот тогда он возможно подымется laugh.gif
но скорее либо колеса сотрутся,либо бензин кончится,либо поток воздуха создаваемый за счет врщения ленты вкупе с поток от колес будет турбулентным и самолет просто опрокинет laugh.gif
вобщем много вариантов smile.gif

Автор: toddd 26.1.2010, 14:43

Цитата(Composter @ 26.1.2010, 12:17) *
вобщем много вариантов smile.gif

Ой, и правда. Варианты (ту-дуньц):
1. Пока самолет то взлетает, то не взлетает, пилот стареет, уходит на пенсию и самолет не взлетает.
2. Со временем пассажиры толстеют настолько, что самолет рад бы, но уже не может.
3. Пробитые маслопроводы уменьшают трение, самолет ускользает от наблюдений.
4. Замученный самолет научился махать крылом как птица, и теперь полет зависит от его желания.
5. Привлеченные бурными баталиями шаманы, готовы за стеклянные бусы оторвать/приземлить что угодно.
6. Появляется такое же ООО, способное дать ответ по номерам и кодам кредиток (плюс советы по Форекс).
7. Сомалийские пираты предлагают испытать самолет на способность плавать.
8. Правительством РФ выделено 2трлн руб. покрышечно-транспортерному комитету.
9. Данфосс начал производить позиционеры для стартующих с транспортера самолетов, с режимом земля/воздух на всякий случай.
10. Открыт госпиталь экстренной помощи с голубыми стенами, удобная парковка, кафе.
11. Свой вариант?

Автор: Сфинкс 26.1.2010, 14:50

Хороший аналог данной задачи: самолёт на НАКЛОННОЙ плоскости с изменяемым углом наклона (совпадение с пушкой случайно).
Изменяя угол наклона, можно найти такой угол, когда самолёт при включенных двигателях, будет неподвижен, стоя на наклонной плоскости.

Автор: HeatServ 26.1.2010, 14:52

Проходили столения и тысячеления, приходили и уходили ледники. Погибали и появлялись цивилизации... В джунглях нового материка распугивая случайных динозавров по движущейся дорожке с диким рёвом неустанно катилась сигарообразная конструкция, на неё смотрели с десяток седобородых старцев почти познавших истину...

Автор: Машинист 26.1.2010, 15:06

Цитата
катилась сигарообразная конструкция...

Обработали после сборки напильником, для улучшения аэродинамики ))))) Теперь он точно никуда не денется !!!

Автор: Seifer 26.1.2010, 15:45

Вот вы некропостеры rolleyes.gif
дали же уже лекарство: самолет улетит с платформы, дальнейшее зависит от местных условий.

или компостер специально тролит на флуд

Автор: Vict 26.1.2010, 16:04

К сожалению, подобные задачки показывают общеобразовательный уровень, который обозначился в результатах опроса sad.gif
Вон, в соседней теме про пушку, даже окончивший физмат считает что не влетит.
Словов нема....

Автор: Seifer 26.1.2010, 16:12

я сразу анекдот вспомнил:
мент останавливает жигули и спрашивает: если я тебе свечу выверну, какая фара гореть перестанет - левая или правая?
ответ: правая
мент: не правильно, штраф 200 баксов.

останавливает опель: если я тебе свечу выверну, какая фара гореть перестанет - левая или правая?
ответ: левая
мент: не правильно, штраф 200 баксов.

останавливает камаз: если я тебе свечу выверну, какая фара гореть перестанет - левая или правая?
вылазит дальнобойщик с монтировкой и молвит: если я тебе сейчас по голове заеду монтировкой, ты на какую ногу хромать будешь?

тут дело не в образовании, в принципе, а в "Канзас Сити Шафл". Платформа и колесо отводят глаза от того, что у самолёта реактивная тяга двигателями. начинают циклиться на этих шасси и всё тут. ловкость рук задачника =)))

анналогично: поднимаетесь вы по эскалатору, а дорожка с ручкой не работает. будете вы дальше подниматься, если начнете держаться за ручку? =)))

Автор: Сфинкс 26.1.2010, 16:15

Мало кто замечает, что самолёт взлетает, лишь нарушив правила задачи. Странно, почему у них реактивный самолёт на луну не улетел до сих пор?

Автор: toddd 26.1.2010, 16:16

Цитата(Машинист @ 26.1.2010, 14:06) *
Обработали после сборки напильником, для улучшения аэродинамики ))))) Теперь он точно никуда не денется !!!

Это же теперь ракета! Раз самолет тут не взлетел, а полет ракеты с ним схож, получается, что если к ракете прикрепить колеса и поставить на этот транспортер - быть миру во всем мире! Нет подъёмной силе - быть добру!

Автор: Vict 26.1.2010, 16:17

Цитата(Seifer @ 26.1.2010, 15:12) *
начинают циклиться на этих шасси и всё тут.
при этом совершенно не понимая, что скорость вращения колес никак не влияет на двиглы самолета.
Не, если ременную передачу к винтам присобачить - то шо то получится остановить biggrin.gif


Цитата(toddd @ 26.1.2010, 15:16) *
Нет подъёмной силе - быть добру!
и жвачке biggrin.gif

Автор: toddd 26.1.2010, 16:18

Ура! Самолотовелосипед!

Автор: Машинист 26.1.2010, 16:24

Цитата
Это же теперь ракета! Раз самолет тут не взлетел, а полет ракеты с ним схож, получается, что если к ракете прикрепить колеса и поставить на этот транспортер - быть миру во всем мире! Нет подъёмной силе - быть добру!

Было б круто, да вот беда... Самолет-то, оказывается, взлетел, назло посредственному уровню образования многочисленных злопыхателей. Так что с ракетой тоже, может случиться бяка. Тангенс некий нужен, отличный от нуля.

Автор: toddd 26.1.2010, 16:25

Самопеды, даже.
"А я сяду
на само-опе-е-ед
и УЕДУ
куда ни-бу-удь" laugh.gif

Автор: Vict 26.1.2010, 16:30

Цитата(toddd @ 26.1.2010, 15:25) *
и УЕДУ
куда ни-бу-удь" laugh.gif
адрес дать? там палаты разные имеются... biggrin.gif

Автор: toddd 26.1.2010, 16:35

Да мне только с транспортера выбраться, бо на самопеде, да без крыльев, да в палаты расписные...
Адрес не нужен - сами найдут! laugh.gif

Автор: Vict 26.1.2010, 16:37

Цитата(toddd @ 26.1.2010, 15:35) *
Адрес не нужен - сами найдут! laugh.gif
clap.gif

Автор: Kult_Ra 26.1.2010, 16:44

Цитата(toddd @ 26.1.2010, 16:35) *
Да мне только с транспортера выбраться, бо на самопеде, да без крыльев, да в палаты расписные...
Адрес не нужен - сами найдут!
Что и "рекомендации" не требуются? blink.gif

Дивная у Вас мечта! Дюже! Нехай сбудется! clap.gif

Автор: Машинист 26.1.2010, 16:51

Дзынь !

Автор: toddd 26.1.2010, 16:53

biggrin.gif Прюветы с самопеда! Здесь прекрасно!

Автор: Сфинкс 28.1.2010, 19:47

А ведь транспортёр можно заменить вращающимся Диском большого радиуса, но самолёт всё одно взлетит лишь нарушая правила задачи.

Автор: Seifer 28.1.2010, 19:59

Не решение нарушает правила задачи, а задача составена с нереальным условием: не существует такой автоматики, которая скомпенсировала бы вращение шасси.
хватит темку апать biggrin.gif пускай уже загнется в низах тем

Автор: dValeriy 28.1.2010, 20:15

А мне интересно отношение некоторых форумчан к решению этих задач. Подход - не поприкалываться и всё же решить - что, по-моему, было бы абсолютно нормально, а - само правильное решение неважно, главное - самоутвердиться(да хоть бы за счёт наезда на собеседников!).
А самолёт не взлетит - колёса у него заблокированы свободнокрутящимся только в обратную сторону транспортёром. Реакция колёс на пытающийся двинуться корпус самолёта(неважно какой двигатель, важно - тяга двигателя менее веса самолёта х коэффициент трения х площадь контакта колёс) - начать крутится в сторону одын, ответная реакция транспортёра - туды крутиться не могу! Вот оно и заблокировано. А когда тяга двига больше - самолёт шасси себе поотрывает, но взлетит! Но это уже и не самолёт а, скорей, ракета.

Автор: Seifer 28.1.2010, 20:23

http://lj.rossia.org/users/anton222/685.html

Другими словами - не существует такой скорости движения полотна, чтобы самолет остановился относительно земли.

Типичные роли обсуждающих на форуме
''Наивные'''
Подвижное полотно взлётной полосы уводит решающего задачу в сторону, порождая у него многочисленные бытовые ассоциации с движущейся беговой дорожкой-тренажером, автомобилем и т. п. и, в итоге, к неправильному решению и/или ответу, кажущемуся ему вполне очевидным. Полная уверенность в своей мнимой правоте позволяет легко вовлечь в эмоциональное обсуждение. Достаточное количество людей, вовлеченных в обсуждение, является решающим для распространения "вируса" - это его питательная среда.

'''Полупрофессионалы'''
Эта категория решающих легко отвлекается на несущественные неточности в формулировке задачи, излишние для получения правильного ответа данные из условия, мелкие и ненужные детали, "неполноту", "некорректность", "противоречивость", "невыполнимость" и т. д. и т. п. Типичный пример - подробное обсуждение вопроса о скорости вращения колес. Поскольку эта скорость может быть любой, этот вопрос для успешного решения задачи вообще не важен. Однако именно ему уделяется центральное место, до самого решения дело так и не доходит, а элементарная задача начинает выглядеть как сложная научная проблема.

'''Изобретатели вечного двигателя'''
Неостановимые изобретатели вечного двигателя, часто из других областей, не имеющих отношения к физике - философы, гуманитарии.

'''Недобросовестные'''
Намеренно разжигающие флейм первоначальным вбросом задачи на форум, последующими ненужными уточнениями, бесконечными вопросами якобы ”для пользы дела”, комментариями, использованием слабых мест и особенностей других решающих (см. выше), эмоциональными примерами из жизненного опыта и т. п. приёмами.

"Физики"
Профессиональные физики очень быстро понимают, что они лишние на этом "празднике жизни" и быстро покидают обсуждение. Ведь все равно толпа затопчет любое компетентное мнение.

"Модераторы"
Закрывают ветку форума, если тема его дискредитирует. Или наоборот - поддерживают, если хотят повысить посещаемость ресурса.

PS^ вы или полупрофессирнал, либо недобросовестный spam.gif

Автор: dValeriy 28.1.2010, 20:37

Да я абсолютно серьёзно отвечаю - колёса блокируются! Самолёт стоит на тормозах! Ничего не крутится - ни транспортёр, ни колёса! Они блокируют любые взаимные попытки движения! При этом не важно как бы колёса могли раскрутитья! Мотор тянет - вытянуть не может!

Автор: Seifer 28.1.2010, 20:43

Цитата
Да я абсолютно серьёзно отвечаю - колёса блокируются! Самолёт стоит на тормозах!

laugh.gif laugh.gif laugh.gif учите матчасть. У самолетов нет тормозов, он тормозит реверсом двигателя. Тяга двигателя малая говорите? да он этой тягой разгоняется по взялетке, а не колёсами, и ей же взлетает. Вобщем, не ленитесь, тыркните ссыль и внимательно почитайте.

ИМХО

Автор: Сфинкс 28.1.2010, 20:54

А я сегодня попробовал поставить дорогостоящий эксперимент с самолётом на диске заместо полотна. Но забыл убрать с того же диска пушку из соседней темы, и они столкнулись. Придётся брать кредит чтобы расплатиться за порчу дорогостоящего оборудования.

Автор: Vict 28.1.2010, 20:54

Цитата(dValeriy @ 28.1.2010, 19:37) *
Да я абсолютно серьёзно отвечаю - колёса блокируются! Самолёт стоит на тормозах! Ничего не крутится - ни транспортёр, ни колёса! Они блокируют любые взаимные попытки движения!
А если самоль на лыжах?
Да я абсолютно серьёзно спрашиваю!

biggrin.gif

Цитата(dValeriy @ 28.1.2010, 19:15) *
А мне интересно отношение некоторых форумчан к решению этих задач. Подход - не поприкалываться и всё же решить - что, по-моему, было бы абсолютно нормально, а - само правильное решение неважно, главное - самоутвердиться(да хоть бы за счёт наезда на собеседников!).
А мне интересно, как это у двух выпускников физмата, на двух задачах два противоположные решения. У одного снаряд попадет в центр и самолет не взлетит, у другого в центр снаряд не попадет и самолет взлетит? Или главное самоутвердиться, а само правильное решение неважно? rolleyes.gif

Автор: Kult_Ra 28.1.2010, 21:15

Цитата
А если самоль на лыжах?
да или "с воды" вдруг взлетать собрался?

Автор: Seifer 28.1.2010, 21:17

Цитата
А мне интересно, как это у двух выпускников физмата, на двух задачах два противоположные решения. У одного снаряд попадет в центр и самолет не взлетит, у другого в центр снаряд не попадет и самолет взлетит? Или главное самоутвердиться, а само правильное решение неважно?

Да это банальный http://lurkmore.ru/Тролль =)

Автор: Kult_Ra 29.1.2010, 10:17

Цитата(Seifer @ 28.1.2010, 21:17) *
Да это банальный http://lurkmore.ru/%D0%A2%D1%80%D0%BE%D0%BB%D0%BB%D1%8C =)
Страшное дело читать такие ссылки! Ужос! blink.gif Все равно что "Справочник фельдшера" mad.gif
Сразу в себе кучу болячек найдёшь.

"То рубашка коротка, то все хозяйтво видно". От себя, зараза, не убежишь, а от взора окружащих тем более! Всяк норовит тебе "этикетку" на экране твоём же приклеить - не редко такое бывает!
Но каждый приходит сюда с хорошим мнением о себе! Выходит с таким же. И не важно, что экран кажет.

Seifer - не берите в голову всякие мелочи! Вы есть и это самое Вам главное!
Не зря говорят:
"чужая душа потёмки и нет у такого фонарика, что её светом из вне озарить"
Придёт её время и она сама из нутри озарит тебя и меня!
Seifer! Будем ждать света её?
Или будем жизнь свою продолжать как и всегда своим теплом, без тепла чужой Души!
clap.gif

Автор: Seifer 29.1.2010, 10:28

Цитата
Страшное дело читать такие ссылки!

Ну можно другими словами объяснить: люди пишут абы что, возможно, заведомо ложное, лишь бы продолжить бесполезный флейм и флуд.
Наверняка, знают уже решение задачи и всё равно пытаются апать занекренную темку. Авось, кто-нить эмоционировать со слюной начнёт biggrin.gif Вот и вся суть троллинга.
а чтоб этого не случилось, есть одно лекарство на картинке выше: Dont feed the trolls
Не давайте Тролям пищи для их флуда. Кто внимательно почитал ссылку с лекарством итак всё поняли.
Остальной флуд - просто флуд spam.gif

Цитата
Все равно что "Справочник фельдшера"

Статья-детектор всё-таки wink.gif biggrin.gif

Автор: Сфинкс 29.1.2010, 13:57

До взлетистов никак не может дойти, что даже реактивный самолёт с включенным двигателем может двигаться вспять на транспортёре.

Автор: Kult_Ra 29.1.2010, 15:12

Цитата(Seifer)
Авось, кто-нить эмоционировать со слюной начнёт biggrin.gif Вот и вся суть троллинга.

Ааааа! Вот она, радость-козявка, где прячется! Под такое мудрёное иностранное слово упаковалось!
А по русски вроде так:
Кто на портрет Ломоносова "глаз положил", кто "что б как не ослепнуть тренируется", кто сплетничает, но есть такой, разновид радости и как интернетспорта. Эйфория! ohmy.gif

Автор: YourDeath 17.7.2010, 0:30

Самолёт и не должен был взлетать,он должен был лишь всего оторваться от поверхности платформы,смысл задачи,да и вообще задача была несколько раз пересформулирована.надо было всего лишь сделать так что бы самолёт оторвался от поверхности,но если пытатся на нём взлетить то ничего не получится,следовательно нужно просто мыслить логически.
моё решение такого,нужно платформу на которой стоит самолёт,скинуть с высоты птичьего полёта так сказать, за время падения самолёт отделится от платформы,всё,задача которую решали годами,решена,или я что то сделал не так?

Автор: HeatServ 17.7.2010, 0:32

Какого же хрена тема до сих ясных пор не закрыта?

Автор: Машинист 17.7.2010, 0:55

Взлетит - когда лопатой подбросят ))))))))

Автор: HeatServ 17.7.2010, 1:23

Тема посвящена светлой памяти модератора прошлого созыва. Берса. Честно говоря иногда перечитываю годовалой даже давности темы с участием люда крайне уместного и думаю: "А форум взлетит или так и дальше будет... Падать..."

Автор: Машинист 17.7.2010, 1:57

Цитата(HeatServ @ 17.7.2010, 2:23) *
Тема посвящена светлой памяти модератора прошлого созыва. Берса. Честно говоря иногда перечитываю годовалой даже давности темы с участием люда крайне уместного и думаю: "А форум взлетит или так и дальше будет... Падать..."

Да конечно взлетит, надо только это... лопатой.. ну сам понимаешь. Активнее лопатой!
Гребсти надо, Серв. За Берса, за остальных.

Автор: Olgerd 17.7.2010, 9:04

Не хватило терпения прочитать все посты, но с моей точки зрения все достаточно просто... Вопрос "взлетит или не взлетит" зависит от силы трения на колесах. Если она будет выше тяги двигателя, то естественно не полетит. К слову хочу заметить, что реактивному самолету для того, чтобы начать движение, наличие воздуха - совершенно не обязательное условие. Таким образом можно подытожить, что не полететь самолет может при совершенно невыполнимых условиях, а именно: абсолютной неразрушаемости шасси и возможности системы управления подстраивать скорость движения полотна таким образом, чтобы скорость вращения колес самолета была равна скорости движения полотна даже на экстремальных скоростях.

Автор: YourDeath 17.7.2010, 13:44

Автор задачки StealthMan утверждал, что разрушительное воздействие на мозг было опробовано еще в 1997 году на курсантах Ленинградского училища связи (ЛВВИУС/СПВВИУС).

Вот оригинальный текст задачи:
Самолет (реактивный или винтовой) стоит на взлетной полосе с подвижным покрытием (типа транспортера). Покрытие может двигаться против направления взлета самолета. Оно имеет систему управления, которая отслеживает и подстраивает скорость движения полотна таким образом, чтобы скорость вращения колес самолета была равна скорости движения полотна. Вопрос: сможет ли самолет взлететь в таких условиях?

Вопрос в том сможет он взлететь или нет?взлётом считается отделение от поверхности на которой он стоит,если считать как вы физики,то он никогда не взлетит,а если подумать логически,физики лишены логического мышления в основном, то взлетит,и это неоспоримый факт,он взлетел,но не факт что полетел biggrin.gif Задачу эту я решил менее чем за час,а вы с 2006 года тут пишите и ни одного ответа близкого к истине.или может я не прав где то?поправьте если я не прав.только не так:"нет он не взлетит и точка!" с объяснением...

Автор: Сфинкс 17.7.2010, 14:55

Аналоги движущейся ленты: диск, поезд, наклонная плоскость, притяжение планет.
Можете ли представить себе тот же самолёт, движущийся задом наперёд?
Я могу представить это, и у меня тот самолёт может взлететь лишь нарушив условия задачи.

Автор: YourDeath 17.7.2010, 15:11

лол =О и как он взлетит?если он летит задом наперёд,то и лента движется наоборот =-=

Автор: HeatServ 17.7.2010, 15:13

Цитата(YourDeath @ 17.7.2010, 14:44) *
Задачу эту я решил менее чем за час,а вы с 2006 года тут пишите и ни одного ответа близкого к истине

smile.gif Порадовали, жгите ещё.

Автор: YourDeath 17.7.2010, 15:34

Цитата(HeatServ @ 17.7.2010, 16:13) *
smile.gif Порадовали, жгите ещё.

а что хочешь сказать это неверное решение?=-= он взлетел?взлетел =-= вот и всё ^^ не нравится моё предположение?выскажи своё,я тоже посмеюсь biggrin.gif

Автор: HeatServ 17.7.2010, 15:53

Цитата(YourDeath @ 17.7.2010, 16:34) *
а что хочешь сказать это неверное решение?=-= он взлетел?взлетел =-= вот и всё ^^ не нравится моё предположение?выскажи своё,я тоже посмеюсь biggrin.gif

Да я видите ли физик, а мы в своей серой массе "лишены логики", поэтому воздержусь. Ну можно добавить, что решений у задачи слишком много, в зависимости от того, что взять за исходные условия, поэтому и говорить не о чем.

Автор: YourDeath 17.7.2010, 16:15

оригинальный текст задачи я скинул,я тоже не физик,скажу больше,физика у меня на уровне медика,задача решается логически,за час,не знаю как вы но я за час пришёл к выводу что он взлетит,и не просто от балды, а с объяснением почему именно так.
Перед тем как высмеивать кого то с "неправильным решением" выложи своё,и откуда знаешь?может именно моё решение верно?=-=

Цитата(HeatServ @ 17.7.2010, 16:53) *
говорить не о чем.

не говори,никто не просит)

Автор: HeatServ 17.7.2010, 16:22

Цитата(YourDeath @ 17.7.2010, 17:15) *
оригинальный текст задачи я скинул,я тоже не физик,скажу больше,физика у меня на уровне медика,задача решается логически,за час,не знаю как вы но я за час пришёл к выводу что он взлетит,и не просто от балды, а с объяснением почему именно так.
Перед тем как высмеивать кого то с "неправильным решением" выложи своё,и откуда знаешь?может именно моё решение верно?=-=

Конечно Ваше решение верно, одно из бесконечного количества верных. На физмате подобных задач нарешался, немного измени условие - всё, новое решение или некоторое количество решений. Задача в предложенном виде не имеет решения. Точнее имеет, и много решений, и все верные.
Цитата(YourDeath @ 17.7.2010, 17:15) *
не говори,никто не просит)

Мы давно на "ты" перешли?

Автор: Сфинкс 17.7.2010, 17:56

Одна из аналогий взлетистов: чел на роликах привязанный к стене и по своей верёвочке ползущий к стенке.
Однако, этот чел может уползти за пределы Солнечной системы, а самолёт не может...

Автор: BROMBA 18.7.2010, 2:45

http://lurkmore.ru/Самолет

Автор: Olgerd 18.7.2010, 8:47

Я тоже погуглил... Все так как я и представлял. Наиболее полно описано здесь http://sites.google.com/site/zadachaprosamolet/home. Но с припиской внизу, что теоретически все же он может не взлететь smile.gif)))

Автор: BROMBA 19.7.2010, 21:49

Кто знает, чем запрессовывают подшипники колёс АН-2 "Кукурузник"?

Автор: инж323 19.7.2010, 22:02

Цитата(YourDeath @ 17.7.2010, 14:44) *
Задачу эту я решил менее чем за час,а вы с 2006 года тут пишите и ни одного ответа близкого к истине.или может я не прав где то?поправьте если я не прав.только не так:"нет он не взлетит и точка!" с объяснением...

Это вот это вот ответ?- "он взлетел,но не факт что полетел "
А решение где? Объяснения ваши не нужны. нужно решение.

Автор: HeatServ 19.7.2010, 22:09

Цитата(BROMBA @ 19.7.2010, 22:49) *
Кто знает, чем запрессовывают подшипники колёс АН-2 "Кукурузник"?

Да кто ж тут такое знает? А чем? Миллион раз летал на кукурузнике и ни разу такой мысли не возникло.

Сегодня смотрел ролик "Разрушителей мифов" про взлетит или не взлетит, взлетели, но с явным нарушением условия задачи, у них самолёт относительно земли двигался.

Автор: BROMBA 19.7.2010, 22:42

Цитата(HeatServ @ 19.7.2010, 22:09) *
Да кто ж тут такое знает? А чем? Миллион раз летал на кукурузнике и ни разу такой мысли не возникло.

Сегодня смотрел ролик "Разрушителей мифов" про взлетит или не взлетит, взлетели, но с явным нарушением условия задачи, у них самолёт относительно земли двигался.


Да дело в том, что годика 3 назад вымутили у аэродромных механиков баночку этой самой смазки...
Ни за что не угадаете, для чего!!! Для смазки вентиляторов кулеров на процессорах и видеокартах, о как!!! Оказалось - именно то, что дохтур прописал...

теперь по поводу задачи: мало, кто обратил внимание на интересный факт - а почему скорость взлета, и особенно, посадки, не дожна превышать определенной величины для каждой модели самолёта?
А что будет, если в болгарку заместо абразивного диска закрутить CD/DVD?
(Спойлер: CD разлетается веером осколков, так-то! Хотя, странно, частота вращения шпинделя привода оптических дисков равна 10000 об/мин, у болгарки - вроде б то же самое значицца на шильдике)
Так вот, при какой частоте вращения разлетится резина/обод колеса "кукурузника"?


PS: ну, ладно, может кто-нить ишшо и дотумкал в болгарку CD закрутить, а интересно, кто-нить совал болгарошный диск в CD-привод?!?!? Бу-го-га!!!

Автор: инж323 19.7.2010, 22:47

Да лан. Самолет взлетает при совсем не таких скоростях вращения колес. Для разлетания далеких.

Автор: BROMBA 19.7.2010, 22:57

Цитата(инж323 @ 19.7.2010, 22:47) *
Да лан. Самолет взлетает при совсем не таких скоростях вращения колес. Для разлетания далеких.

Взыв баллона колеса при посадке - явление сааавсем не редкое... при касании полосы частота вращения рывком возрастает от 0 до (ну, дальше сами, гы-гы..), со всеми выстреливающими, да + рывок тангенциальнонаправленной противодействующей инерции силы в пятне контакта + усилие удара шасии о полосу, ну и так далее...
Понятно, что запас прочности колес по идее должен быть % эдак на 300 выше расчетного, но всё таки...

Теперь - по поводу задачи: следует уточнить дефиниции понтяий "двигатель", и "движитель", тогда наступит прозрение, озарение, и решение прийдет само, аки кирпич, упавший с крыши прямо на каску...

Автор: BROMBA 19.7.2010, 23:08

Цитата(HeatServ @ 19.7.2010, 22:09) *
Миллион раз летал на кукурузнике и ни разу такой мысли не возникло.

Взлет кукурузника длицца примерно столько же, как среднестатистический половой акт; на миллион взлетов без перекуров, сна, еды, полетов, посадок и фильмов ужосов, придёцца потратить примерно 6 лет жизни ...

Автор: инж323 19.7.2010, 23:40

Цитата(BROMBA @ 19.7.2010, 23:57) *
Взыв баллона колеса при посадке

И это - " Самолет взлетает при совсем не таких скоростях вращения колес."
Есть ли различие?
И потом. При посадке все ж видится покрышка испытывает ударную нагрузку. Само то колесо просто от ) до скорости при посадке начинает раскручиваться, хоть момент велик,но .. не совсем уж и лежит он в разрушительной части шкалы. И далеко от неё.

Во время войны изобрели для пушек колеса неубиваемые, те, что для гаубиц. Их заполняли неким гусметиком. При попадании пули или осколка в колесо появлялся типа нарыва,но колесо было пригодно к использованию сколь угодно после этого.

Автор: HeatServ 19.7.2010, 23:54

Цитата(BROMBA @ 20.7.2010, 0:08) *
Взлет кукурузника длицца примерно столько же, как среднестатистический половой акт; на миллион взлетов без перекуров, сна, еды, полетов, посадок и фильмов ужосов, придёцца потратить примерно 6 лет жизни ...

smile.gif В своё время одноклассника так же подловил, он утверждал, что посмотрел миллион (или чуть меньше?) фильмов. Я перевёл эти фильмы в года.

По задаче: автору нужно ставить памятник вообще или сажать на кол.
В задаче 3 логические бомбы.
1. Упомянутый механизм на земле невозможно создать, т.е. самолёт будет тудыма-сюдыма рыпаться относительно земли.
2. Механизм вообще не при делах, поскольку в идаеле (самолёт неподвижен относительно земли) это вообще блокировка колёс.
3. Кажущаяся простота задачи, задача уходит в такую крутую аэродинамику при отрыве колёс от полотна, что туда даже ползти страшно.

Наверное бы было очень неплохо чтобы самолёты так взлетали, аэродромы не нужны, но движки нужны просто нереальные.

Автор: BROMBA 20.7.2010, 0:04

Цитата(HeatServ @ 19.7.2010, 23:54) *
smile.gif В своё время одноклассника так же подловил, он утверждал, что посмотрел миллион (или чуть меньше?) фильмов. Я перевёл эти фильмы в года.

Хммм... в порядке флуда (аааа, один чёрт, тема флудовая): узнакомого сисодмина в коллекции 7000 DVD с фильмами + 6 терабайтных винтов с ними жэ... сколько примерно лет потребуецца, шоб просмотреть это всё?
(вопрос "а нафига?!?!?" пока не рассматриваем)
По задаче - танунафиг, задачка сама по себе проста, как палка и веревка; подобная хрень, помницца, мелькала в книжках типа "познавательная механика", просто на первый взгляд она содержит взаимоисключающие параграфы...
Не стоит заморачивацца, один хрен, любое из возможных решений доказать (вернее, зставить в него поверить) невозможно, ибо такова люццкая психология...

Лучче ответьте на такой вопрос: "Можно ли присесть (задницей) на свежезалитую (жидкой сталью) литейную форму?" (очень этот вопрос полюбляем задавать юным металлургам на защитах дипломов, хе-хе...)
Спойлер: Юная девченка-литейщица снесла башню всему ГЭКу - "Можно, если не попасть на стояк!"... ржали все... полчаса... а ить ответ правильный...

Автор: Илья М 23.7.2010, 8:02

Эта задача кажется сложной только дилетантам- обывателям.

В случае движения полотна в обратную сторону со скоростью самолёта возникает всего лишь дополнительное удвоенное сопротивление в подшипниках ступиц, и удвоенное сопротивление качению колеса.

Эти две составляющие ничтожно малы в сравнении с лобовым сопротивлением самолёта в полёте!!!
Так что самолёт взлетит без всяких проблем.

В качестве более сложного взлёта могу напомнить взлёт с воды, где сопротивление воды растёт квдратично от скорости скольжения, но самолёт таки взлетает!!


Прочность колеса при посадке тоже нужна вовсе не бесконечная, а ударные нагрузки при раскручивании в момент касания смазаны во времени из-за пробуксовки колёс по бетону ВПП.
Дым из под колёс при посадке- это и есть следствие пробуксовки, ну а буксование колёс и на автомобиле можно легко организовать, если это Мустанг с АКПП и 6-ти литровым движком....колёса при этом не разрушаются мгновенно, а постепенно истираются, оставляя чёрные полосы резины на асфальте.

Автор: alem 23.7.2010, 8:42

Цитата(Илья М @ 23.7.2010, 9:02) *
Так что самолёт взлетит без всяких проблем.


Мощно сказано!

Автор: HeatServ 23.7.2010, 19:25

Цитата(Илья М @ 23.7.2010, 9:02) *
Эта задача кажется сложной только дилетантам- обывателям.

Чорт, я так и думал, что однажды придёт профессионал и распедалит всю проблему.
Цитата(Илья М @ 23.7.2010, 9:02) *
Так что самолёт взлетит без всяких проблем.

Всё, ничего лишнего, тема закрыта. Учитесь, господа!

Автор: LordN 23.7.2010, 19:28

Цитата
Всё, ничего лишнего, тема закрыта. Учитесь, господа!
поздно. самлеты на транспортерах не летают.

Автор: HeatServ 23.7.2010, 19:32

Цитата(LordN @ 23.7.2010, 20:28) *
поздно. самлеты на транспортерах не летают.

Лорд, Вам же профи сказал, что полетит и без проблем, значит взлетают. А вот почему в кажном аеропорту таких транспортёров нет - ну так ведь бетон нужно куда-то девать, вот и девают - аэродромы строят, ничего, вот кризис поприударит по бетонным заводам, и тогда - истинно говорю, поставят в кажном захолустье, даже на авианосцах поставят транспортёрные ленты, неча пелотов взлётными перегрузками мучать.

Автор: LordN 23.7.2010, 20:15

на авианосцах може и поставют, там всякого понаставлено, одним всяким больше - ништо, перебьюцца, а вот в портах таки нет. не будут.

Автор: ЁЖик 23.7.2010, 20:26

Цитата(BROMBA @ 19.7.2010, 23:42) *
А что будет, если в болгарку заместо абразивного диска закрутить CD/DVD?
(Спойлер: CD разлетается веером осколков, так-то! Хотя, странно, частота вращения шпинделя привода оптических дисков равна 10000 об/мин, у болгарки - вроде б то же самое значицца на шильдике)
Так вот, при какой частоте вращения разлетится резина/обод колеса "кукурузника"?!!!

Ашипка

Скорость вращения 125й болгарки = 11 000, а не 6 000. 6 000, это у 230-й.
Важно, штоп скорость резания была не ниже 35м/сек. Выше -можно (до определённой поры). Ниже - нет. Как в шлифовке.
Но эт-ж, какой CDюк/DVDюк нужен?
Тут важно смотреть на шильдик именно той болгарки, в которую CD-юк заряжается. Тогда... и музыка... и картинки самоподвижныя...
blink.gif

Автор: timmy 23.7.2010, 20:31

Цитата(BROMBA @ 20.7.2010, 1:04) *
Лучче ответьте на такой вопрос: "Можно ли присесть (задницей) на свежезалитую (жидкой сталью) литейную форму?" (очень этот вопрос полюбляем задавать юным металлургам на защитах дипломов, хе-хе...)

Упасть можно, зашибиться можно, искупаться можно (в Северстали был такой эпизод), а вот чтоб присесть... Жарковато по-моему будет. Я б не смог.

Автор: HeatServ 23.7.2010, 20:55

"Вниманию пассажиров вылетающих рейсом 777 ушедших позавчера в кафе, ваш самолёт вот-вот взлетит с седьмого транспортёра."

Автор: инж323 23.7.2010, 21:59

Цитата(timmy @ 23.7.2010, 21:31) *
Упасть можно, зашибиться можно, искупаться можно (в Северстали был такой эпизод), а вот чтоб присесть... Жарковато по-моему будет. Я б не смог.

Что то с Т на улице сегодня не в порядке похоже.
А вы пар в утюге не пробовали пускать, что б ощутить, что утюг приподнимается? Так вот выходящие продукты сгорания сделают некую подушку , которая оттолкнет "задницу".

Автор: Vict 23.7.2010, 22:18

Цитата(timmy @ 23.7.2010, 20:31) *
а вот чтоб присесть... Жарковато по-моему будет. Я б не смог.
Вы просто не пробовали biggrin.gif
Если песчанная опока(форма) - пофиг, можно и задницей(кратковременно)...
если стальная - фиг сядете, бо неудобно biggrin.gif

Автор: BROMBA 23.7.2010, 22:54

Цитата(timmy @ 23.7.2010, 20:31) *
Упасть можно, зашибиться можно, искупаться можно (в Северстали был такой эпизод), а вот чтоб присесть... Жарковато по-моему будет. Я б не смог.

Два! И вон из класса!!!
Цитата
Что то с Т на улице сегодня не в порядке похоже.
А вы пар в утюге не пробовали пускать, что б ощутить, что утюг приподнимается? Так вот выходящие продукты сгорания сделают некую подушку , которая оттолкнет "задницу".
Два!! Списталисты, блин...
Цитата
Если песчанная опока(форма) - пофиг, можно и задницей(кратковременно)...
если стальная - фиг сядете, бо неудобно
А вот эта - пядь!!! Всё правильно, всё сходицца! Теплопроводность песчано-глинистй смеси ничтожна, и за 30...40 сек. контрлад формы нагреецца примерно на 2...5 градусов за счет газопроницаемости, не более того... (ну, ежли смесь не очень сырая, и водородных хлопочков не будет)... Так шо девка права на стопиццот промилле - можна!!! Если не сесть жопой на стояк, ибо там - затвердевающая сталь, а стояк перемерзает перед прибылями, так-то!!!!!!

Бе-бе-бе!!! (дразницца)

Автор: инж323 23.7.2010, 23:18

Если "ну, ежли смесь не очень сырая, и водородных хлопочков не будет", ежели " и за 30...40 сек."
Еще сюда про бабушку, которая могла б быть дедушкой.

Автор: BROMBA 23.7.2010, 23:50

Цитата(инж323 @ 23.7.2010, 23:18) *
Если "ну, ежли смесь не очень сырая, и водородных хлопочков не будет", ежели " и за 30...40 сек."
Еще сюда про бабушку, которая могла б быть дедушкой.

"Проверено электроникой"(с), мы гарантируем это...
Кстате, вопрос на засыпку: а, собссна, откуда "водородные хлопочки" выбивают, ась?

Автор: HeatServ 24.7.2010, 0:20

Да вполне можно поверить девушке-литейщице, интересный подобный эффект достигается при снятии КИПЯЩЕГО чайника (дно плоское) с костра и кратковременным становлением его на ладонь, вполне терпимо.

Автор: timmy 24.7.2010, 0:33

Инж323
Я ж написал - я б не смог. Я даже больше могу написать - и нафиг мине энтот екстрим не далси. А насколько обоснованно он мине не далси, мне пофиг.
Vict
Пасибки, я как-нить без прыжков по опокам проживу.

Цитата(BROMBA @ 23.7.2010, 23:54) *
Бе-бе-бе!!! (дразницца)

Вот вы мне Бромба и объясните, успею ли я за эти самые 30-40 секунд до этой вашей формы дойти, а если успею, то успею ли присесть и более того - не пострадать от этого и не вылететь с работы. Ежли хоть раз ответом "нет" буит, то и ответ мой останется тем же - я б не смог. И ваще - я в отделе за ТБ отвечаю, так что мине хулюганить не положено.

Автор: timmy 24.7.2010, 0:39

Цитата(HeatServ @ 24.7.2010, 1:20) *
Да вполне можно поверить девушке-литейщице, интересный подобный эффект достигается при снятии КИПЯЩЕГО чайника (дно плоское) с костра и кратковременным становлением его на ладонь, вполне терпимо.

Да поверить то можно, выполнять неохота. Чуть стормозил и капец руке. А если совсем не руке? Не надо нам такого щастя.

Автор: BROMBA 24.7.2010, 1:08

Цитата(timmy @ 24.7.2010, 0:33) *
Вот вы мне Бромба и объясните, успею ли я за эти самые 30-40 секунд до этой вашей формы дойти, а если успею, то успею ли присесть и более того - не пострадать от этого и не вылететь с работы. Ежли хоть раз ответом "нет" буит, то и ответ мой останется тем же - я б не смог. И ваще - я в отделе за ТБ отвечаю, так что мине хулюганить не положено.

Та не вопрос: Вы-разлившик, на плацу стоят набранные за смену формы, много форм, крановщица Вам только что подвезла пятнадцатитонный ковш с металлом, и Вы, покручивая подъем стопора, разливаете это Ваше всё соласно металлоёмкости... Ну и кто мешает после опорожнения ковша устало присесть на последнюю залитую форму? Да никто... Крановщица увезла опустевший ковш, делать пока нечего... Штаны войлочные, пинджак, шляпа, щиток на лице, ботинки со стальной прокладкой в носке на термостойкой подошве... почти марсианин, кто упрекнет? Да никто, даже мастер, даже начальник участка, даже начальник смены, ибо всем <censored>....

Кстате, в 7-м цехе, когда там еще были живы вагранки (чуть позже перешли на индукционки), там был уникальный вагранщик... он трусов не носил принципиально; а войлочные штаны были всегда лопнувшие по шву... так вот, лётку в копильнике он выбивал особым образом (весь цех прибегал позырить и поржать) - садился задницей на выпускной желоб, яйца через прореху в штанах выпадали в желоб, а он в это время штангой выбивал пробку из лётки; когда струйка жидкго чугуна из копильника начинала течь по желобу в ковш, он приподнимался, мозолистой левой рукой подбирал из желоба свои яйца, и махнув ногой над заполненным желобом, удалялся за свежей пробкой... цех руколескал...
Кстате, он таки мог перебить голой рукой струю жидкого чугуна из желоба... не за бутылку, просто так, из врожденного артистизма...
Вопрос: как это можно сделать? И почему нельзя перебить рукой струю жидкой бронзы?

Автор: timmy 24.7.2010, 2:11

Во-во. Им то да, а мне - нет. Мне дороже обойдется. А как перебить - ну вот чего не знаю, того не знаю. Мне таких фокусов металлурги еще не показывали. Могу только предположить, что чугун к коже не липнет и ожог если и будет, то не большой. А как сделать? Хм. Маслом чтоль обмазаться? Больше никаких идей в башку в три часа ночи не заползает...

Автор: LordN 24.7.2010, 5:24

Цитата
почему нельзя перебить рукой струю жидкой бронзы?
поверхностное натяжение разное, всмысле прилипает к коже.

Автор: HeatServ 24.7.2010, 9:32

Цитата(LordN @ 24.7.2010, 6:24) *
поверхностное натяжение разное, всмысле прилипает к коже.

Я бы сказал смачивается. Наверное это жёстко - смочить руку жидкой бронзой.

Автор: LordN 24.7.2010, 15:32

смачивает, да. это из-за олова, оно действует как мыло в этом сплаве.

Автор: timmy 24.7.2010, 22:26

Цитата(LordN @ 24.7.2010, 16:32) *
смачивает, да. это из-за олова, оно действует как мыло в этом сплаве.

Ага. Значит не зря я калды пересдавал. Помню кой-чего.

Автор: BROMBA 12.8.2010, 22:38

Цитата(LordN @ 24.7.2010, 15:32) *
смачивает, да. это из-за олова, оно действует как мыло в этом сплаве.

Хи-хи... Это бронзы ОЦС... а как по поводу БрАЖ, БрКМц, БрАМц, они олова не содержат...
А руку отожгут на "раз-два-три"...

Автор: BROMBA 12.8.2010, 23:19

Это как-то раз один из нас отжег, так отжег:
Присралось BRomba как-то отлить нестандартный радиатор для какого-то самопального устройства; и тут на хвост упал коллега, ему тоже внезапно понадобилось и такой жэ; ну, как говорицца, "говно-вопрос", верстак, смесюка, трамбовка, две формы, в одной литники BRomba прорезал как для стали, отливка в нижней полуформе, иголками вниз, вторую форму коллега прорезал сам - цветной сплав, ну, типо технолоия как для чугуна, иглы вверх, отливка вверху (а душники не наколол, хе-хе)...
Короче, печь СШОЛ, силумин, перегретый до 820*, BRomba захватами вынимает из печи тигель (печь выключает, и оставляет открытой, шоб побыстрее остыла), перехватывает клещами, и заливает первой свою форму; всё пучком, металл вышел через выпора, процесс пошел нормально; перходит ко второй форме, заливает быстро и уверенно... и тут... кааак ебарахнуло!!! Форма сырая, полости вверху, литники уже перехватило, и всё такое - верхняя полуформа подлетает на полметра, и 2 кило жидкого силумина веером разлетаюцца в разные стороны в радиусе трех метров... ну, BRomba - старый разливщик, и загодя напялил поверх джынсов войлочные штаны, потому как знал, чем чревато... а вот у коллеги и учебного мастера (только устроился на работу, недели не прошло, о шуточках от BRomba ни ухом ни рылом, хи-хи...) - пасхальные брюки были в демисезонном исполнении, стали в летнем - "в дырдочку"... пока они оба тушили штаны и громко горевали, BRomba уже всё убрал, и уволок формы на выбивку, а в остывшую уже печь в обычной консервной банке из-под сгущенки загрузил куски и обломки сплава Вуда, чтобы потом разлить на слитки... в печи примерно 65...70*, чего вполне для Вуда достаточно... Тут подходит офигевший учебный мастер и смотрит... и тут BRomba внезапно решил повыёживацца... закатывает рукав, голой рукой лезет в шахту печи, (глаза у мастера стали как два полтинника), вынимает банку с жидким металлом, (глаза стали как 2 монеты по 1 юаню), наливает металл в левую ладошку (на штанах медленно начало расползацца желтое пятно), болтает металл в ладошке, говорит "ну, зашибись", сливает металл обратно в банку, и спокойно идет разливать металл по чашкам от установки для измерения влажности смеси... охреневший окончательно мастер вылетает из лаботатории с невнятным жывотным воплем смертельно раненого мозга с глазами на лбу, отваленной челюстью и пятном н штанах...
Ржали все... долго, от души, с чувством, и обстоятельно... до боли в межреберных мышцах, до кашля и хрипоты, коллега ради этого даже не затаил обиду за пропаленные штаны от костюма...
(Для справки - темпераура плавления силумина - 720...740*, перегрев перед заливкой - 820*; температура плавления сплава Вуда - 57*, перегрев перед разливкой на слитки - 60*... Хи-хи-хи... а жидкий силумин у ликвидуса и жидкий Вуд в ладошке вееесьма друг на друга похожи (для тех, кто не понимает), хи-хи-хи...)

Автор: areneal 16.8.2010, 10:44

Взлетит.
Объясню.
Самолет, в отличие от автомобиля движется не с помощью мощности приведенной к валу (оси) колеса, проще говоря - движется он не потому-то крутит колесами.
Самолет отталкивается, применим здесь данный термин, от воздуха с помощью винта или реактивного двигателя (не придираться, в данном случае он не отталкивается, знаю smile.gif). Если принять нулевое сопротивление качения колес, допустимость числа оборотов колеса близкому к бесконечности и если существует хоть какое-то минимальное время реагирования механизма дорожки на увеличение скорости самолета (запаздывание реакции) - он взлетит!
)))

Автор: areneal 16.8.2010, 10:52

Цитата(zr84 @ 4.11.2006, 16:25) *
Теоритически может взлететь. Нужно сделать численные оценки.
Вот почему:
В полосе которая движется, в силу вязкости воздуха образуется пограничный слой
воздуха с градиентом скоростей. Поэтому на верхней и нижней плоскостях крыла возникнет разность давления(в силу профиля крыла)- которая и составляет подъемную силу.
Вопрос в том, с какой скоростью должена двигаться полоса, чтобы обеспечить необходимую
разность давления для полета?


Ключевая фраза разность давления - только в данном случае подъемная сила становится "опускающей"...

Автор: Сфинкс 18.8.2010, 8:29

Вновь обращаю внимание на 3 варианта самолёта с включ. двигателями на движ. ленте относ. земли:
1) Самолёт движется взад.
2) Самолёт стоит на месте.
3) Самолёт движется вперёд.
Задаче соотв. случай 2, другие нарушают условия задачи.
Самолёт НЕ взлетит.

Автор: Илья М 18.8.2010, 15:37

Цитата(Сфинкс @ 18.8.2010, 9:29) *
Вновь обращаю внимание на 3 варианта самолёта с включ. двигателями на движ. ленте относ. земли:
1) Самолёт движется взад.
2) Самолёт стоит на месте.
3) Самолёт движется вперёд.
Задаче соотв. случай 2, другие нарушают условия задачи.
Самолёт НЕ взлетит.

Боюсь, что подобное изменение (интерпритация) исходных условий задачи несколько странно звучит.
Если бы так ставилась задача, то и надо было говорить о неподвижном относительно земли самолёте, под которым зачем-то ползёт транспортёр.
Ну, а неподвижные САМОЛЁТЫ не летают по определению!
Или это была задачка для юристов на казуистичность формулировки?....)))

Автор: alem 18.8.2010, 15:43

Цитата(Илья М @ 18.8.2010, 16:37) *
Или это была задачка для юристов на казуистичность формулировки?....)))


Это, похоже, тест на способность правильно понять неправильно формулируемое задание. В жизни встречается часто. Например, под проверкой эффективности вентиляции обычно понимают проверку на соответствие нормативной кратности.


Автор: Kult_Ra 18.8.2010, 15:56

Не зря пришли к соглашению - правильно сформированный вопрос содержит ответ.
Или "если в программу по расчёту системы вентиляции поданы заведомо неточные данные" соответствие нормативной кратности не жди! sad.gif

Автор: Сфинкс 18.8.2010, 18:53

Вновь внимательно прочитав условие задачи обнаружил, что скорость движения колёс будет равна скорости движения полотна лишь в моём случае 2, при котором самолёт никуда не полетит.

Автор: HeatServ 18.8.2010, 20:48

Скорость ветра увеличивается каждый час на 100 км/ч. Дерево абсолютно прочное (высота в точке прицепления H), трос абсолютно прочный (длина L), АН-2 абсолютно прочный (кто бы сомневался?). Назовите максимальную высоту на которую поднимется АН-2.

 

Автор: Машинист 19.8.2010, 14:51

Цитата(HeatServ @ 18.8.2010, 21:48) *
Скорость ветра увеличивается каждый час на 100 км/ч. Дерево абсолютно прочное (высота в точке прицепления H), трос абсолютно прочный (длина L), АН-2 абсолютно прочный (кто бы сомневался?). Назовите максимальную высоту на которую поднимется АН-2.

От передних колесиков до земли - длина троса rolleyes.gif
От задних - длина троса плюс высота табуретки

Автор: Илья М 20.8.2010, 7:01

ВЫсота отрыва в этом случае равна нулю, так как положение самолёта сооответсвует горизонтальному полёту.
Правда и на грунт он уже опираться не будет.

Автор: HeatServ 20.8.2010, 14:01

Цитата(Илья М @ 20.8.2010, 8:01) *
ВЫсота отрыва в этом случае равна нулю, так как положение самолёта сооответсвует горизонтальному полёту.
Правда и на грунт он уже опираться не будет.

То есть подъёмная сила крыла не зависит от скорости набегающего потока?

Автор: Илья М 20.8.2010, 14:33

Цитата(HeatServ @ 20.8.2010, 15:01) *
То есть подъёмная сила крыла не зависит от скорости набегающего потока?

Самолёт вообще-то может лететь горизонтально в широком диапазоне скоростей, правда меняя при этом углы атаки крыла, положение закрылков и тягу двигателя.

В этом случае нам опять подсунули некоректные условия.
Так что мой ответ ТОЖЕ верен...)))

Автор: Kult_Ra 20.8.2010, 15:34

Если "вектор ветрового давления направления не меняет" и про табуретку не сказано что она "прочно абсолютно закреплена", то её сдует, зараза, и только "попа самолёта" опустится на высоту табуретки. blink.gif

Автор: kornkorn 20.8.2010, 15:34

А вот здесь ответ однозначный:

Как передает Life News, по словам источника в представительстве завода Citroen во Франции, «данные исследований электронных блоков контроля систем автомобиля говорят о том, что в момент ДТП Citroen ехал со скоростью 75 км/ч, а Mercedes со скоростью 35 км/ч». При этом источник призвал принять во внимание тот факт, что реальная скорость малолитражки была намного выше.
http://www.rosbalt.ru/2010/08/20/764109.html

Автор: Машинист 20.8.2010, 23:50

Цитата(HeatServ @ 20.8.2010, 15:01) *
То есть подъёмная сила крыла не зависит от скорости набегающего потока?

В том-то и дело, что зависит. До момента срыва потока. Еще один Х-фактор.

Автор: BROMBA 21.8.2010, 1:48

Цитата(HeatServ @ 18.8.2010, 20:48) *
Скорость ветра увеличивается каждый час на 100 км/ч. Дерево абсолютно прочное (высота в точке прицепления H), трос абсолютно прочный (длина L), АН-2 абсолютно прочный (кто бы сомневался?). Назовите максимальную высоту на которую поднимется АН-2.



Цитата(Машинист @ 20.8.2010, 23:50) *
В том-то и дело, что зависит. До момента срыва потока. Еще один Х-фактор.

Стоп-стоп-стоп, ребята/девчата... этот случай нетипичный... эффект экраноплана... в этом случае несущие плоскости расположены слишком низко, и до срабатывания мифического срыва потока вовсю будет работать эффект экрана...

Автор: HeatServ 22.8.2010, 11:59

Ну, вот, мнения разделились и ушли высоко в теорию крыла.
Ваши варианты с теми же условиями при привязыванию к дереву болида Формула-1, установленного на подпружиненный стол.


 

Автор: Kult_Ra 22.8.2010, 12:21

Цитата
Ну, вот, мнения разделились и ушли высоко в теорию крыла.
Почему Вы, HeatServ, моё проигнорировали - ничего не произойдёт, крыло не причём, но табуретку из-под сдует нафиг, blink.gif раз "вектор ветрового давления направления не меняет".
Протестую! tomato.gif

Автор: Сергей В. 22.8.2010, 12:21

Цитата
болида Формула-1,

Хи-хи!
Не взлетит. У болидов обратная аэродинамика, предназначеннная для прижимания машины к земле.

Автор: HeatServ 22.8.2010, 20:17

Цитата(Сергей В. @ 22.8.2010, 13:21) *
Не взлетит. У болидов обратная аэродинамика, предназначеннная для прижимания машины к земле.

А это значит, то, что...

Автор: Машинист 22.8.2010, 21:32

Пора-по-пиву-пора/// ©

Автор: HeatServ 22.8.2010, 22:20

Цитата(Машинист @ 22.8.2010, 22:32) *
Пора-по-пиву-пора/// ©

Водки, водки, друг мой, не марайтесь пивом.

Автор: BROMBA 23.8.2010, 1:16

Кстате, сёдня, после пива вкуснога, вот такая мысля возникла:
Не, ну а кто мешает, дабы окончателно спентвить бальдуы и приклимить бадубины, произвести "пробу крестом", каг грили иеизуиты???
Для этого всего-то надо:
- мотодельтаплан, одна штука (не та хрень, с моторчиком на крыле, а другая, потяжелее, с рамой и колёсиками на раме)...
- отвальный транспортёр карьерного роторного/шагающего/цепного экскаватора с тиристорником на регулировке скорости ленты.
Всё..
Мы не можем произвести пробу крестом... не потому, что нет дельтаплана, он есть; не потому, что нет танспортера, он есть; а потому, что никак невозможно протащить этот сраный дельтаплан на территорию завода, где на участке шлакопереработки эта срань стоит - режим там полютее Освенцима...
Если сможете реализовать эту идею, отпишитесь пожалуйста...
Заместо нищебродского дельтаплана можно использовать мотопароплан...

Автор: BROMBA 24.8.2010, 11:40

Уффф... сегодня уже попустило, перечитали то, что понаписывали, и стало удивительно - а ведь в идее с мотодельтапланом что-то рациональное есть...
Вот смотрите, к примеру:



трайк российского производства весит вместе с мотором 25...65 кг; украинское крыло С-5 - 28...36 кг ...
То есть, дотащить летательный аппарат тяжелее воздуха до ближайшего транспортёра можно и в одиночку...
Скорость отрыва - 45км/ч... велосипед и то шустрее...
Статическая тяга - 45-70кг... то есть, для решающего эксперимента подходит...
Разбег - 35-60м - то, шо дохтур прописал, на случай, если таки взлетит... найти транспортер длиной 100 м - не проблема.
Колея - 1300 - вот это уже не совсем хорошо... но поправимо (смотрим фото, любуемся задней подвеской, и прикидываем, чем лучче пилить - ножовкой или болгаркой)...
Транспортер... да любой, с управляемым приводом, шириной ленты от 1м, и ширше, длиной до 100 м...
Шахтные отвальные, экскаваторные отвальные, и так далее, стоящие на открытом пространстве...
Ну, и при отсутствии автоматической системы управления скоростью, сопряженной с колёсиками трайка - промышленный робот пятого поколения в ватнике и с сиськами, осуществляющий принцип визуально-интуитивно управления приводом транспртера посредством органа ручного управления скоростью на передней панели преобразователя...
Вроде всё...
Да, толпа зевак, пилот (одна штука + парочка запасных), радио, телевидение, журнализды и журноламеры в ассортименте...

Реально? А то!!! Выполнимо??? Лехххко!
Ну, так почему до сих пор не сделано?

Автор: Илья М 24.8.2010, 11:52

Потому это до сих пор не сделано, что сама по себе задача глупостью попахивает..... да и правильный ответ ясен и без дорогостоящих экспериментов любому трезво мыслящему инженеру-механику (мне во всяком случае)...)))

Автор: dsemn 24.8.2010, 15:37

Цитата
Мне думается не взлетит, так как подъемная сила на крыле образуется за счет набегания потока воздуха при разбеге самолета, а не при его неподвижном состоянии и включенных двигателях. Скорость набегания потока при включенных двигателях и неподвижном состоянии самолета будет недостаточна для создания нужной подъемной силы, и самолет останется на дорожке.

Я не знаю точно, но по моему на винтовых самолетах значение имеет не столько поток воздуха при разбеге самолета, сколько его поток от винтов. НО вообще мое мнение - не взлетит

Автор: Сергей В. 24.8.2010, 18:47

Цитата(BROMBA @ 24.8.2010, 11:40) *
... Реально? А то!!! Выполнимо??? Лехххко!
Ну, так почему до сих пор не сделано?

Да, говорят, сделали. "Разрушители легенд". Сам эту передачу не видел, но мне пересказывал источник, достойный доверия biggrin.gif - непьющий.
Там вначале была радиоуправляемая модель, а потом легкий одномоторный самолет и дли-и-и-инная полоса брезента, которую тащили в обратную сторону.
Самолет взлетел bleh.gif , естественно совершив положенный ему разбег wink.gif

Автор: HeatServ 24.8.2010, 19:18

Цитата(Сергей В. @ 24.8.2010, 19:47) *
Самолет взлетел bleh.gif , естественно совершив положенный ему разбег wink.gif

Вот именно, разбег, в реальности собрать ленту-бегунок которая поддерживает линейную скорость самолёта относительно земли равной нулю - невозможно. Это условие идеализировано. А у разрушителей он взлетел, потому что немного разогнался на самом деле относительно земли. Вообще - взлетит самолёт при иходных условиях задачи, потому что мощность условиями не лимитирована.

Автор: Сфинкс 24.8.2010, 19:44

Лента бегунок чудесно станет возможной, если самолёт сам будет поддерживать соотв. скорость.
Так что самолёт с вкл двигателем едущий по ленте взад реальность,
а взлетит лишь нарушив условие задачи.

Автор: toddd 24.8.2010, 20:14

Самолет без колес, фюзеляж обмазан салом с чесноком. Включаем двигло, мощное двигло.
Что произойдет (все крепкое, ничего не отваливается)?
Как можно скорость фюзеляжа уравнять скоростью транспортера?
Для особой продвинутости, представим ленту транспортера не лентой, точнее очень короткой лентой - размером пятна контакта колеса(которое сняли). Т.е. представим в виде колеса, привинченного к земле.
Имеем самолет без колес, стоящий на колесе прикрученному к земле. Абсурд? biggrin.gif

Автор: HeatServ 24.8.2010, 20:36

Цитата(toddd @ 24.8.2010, 21:14) *
Для особой продвинутости, представим ленту транспортера не лентой, точнее очень короткой лентой - размером пятна контакта колеса(которое сняли). Т.е. представим в виде колеса, привинченного к земле.
Имеем самолет без колес, стоящий на колесе прикрученному к земле. Абсурд? biggrin.gif

Почему абсурд, взлетит самолёт, двигло-то ядрёное, болты посрывает, попрыгает и взлетит.
Цитата(toddd @ 24.8.2010, 21:14) *
Самолет без колес, фюзеляж обмазан салом с чесноком.

Горилку пилоту, литер.
Цитата(toddd @ 24.8.2010, 21:14) *
Как можно скорость фюзеляжа уравнять скоростью транспортера?

Никак.

Автор: toddd 24.8.2010, 21:06

Цитата(HeatServ @ 24.8.2010, 20:36) *
Почему абсурд, взлетит самолёт, двигло-то ядрёное, болты посрывает, попрыгает и взлетит.

Немного невнятно я написАл.
Самолет без колес опирается фюзеляжем на колесо, которое прикручено к земле и имеет привод, настроенный на вращение ему навстречу. В общем, фигли мудрить - улетел он, и срывать ниче не нужно. biggrin.gif
Пилот при горилке, само собой, браво, HeatServ clap.gif

Автор: Сфинкс 24.8.2010, 22:09

Представим самолёт на наклонной плоскости. Горизонтально развевается и взлетает легко. Вертикально не разбегается и не взлетает никак.
Под конкретным углом наклона висит на месте.
Так что самолёт катящийся по барабану не абсурд: выкл. двигло, укатится взад

Автор: toddd 24.8.2010, 22:22

Цитата(Сфинкс @ 24.8.2010, 22:09) *
Так что самолёт катящийся по барабану не абсурд: выкл. двигло, укатится взад

Круто! Давайте еще проще.
Мальчик Коля тащит санки за веревочку, ножками топая по земле. Санки лыжами стоЯт на роликах, вращающихся навстречу. Возможно ли роликами уравнять скорость санок? Стащит ли Коля санки с роликов? smile.gif

Автор: BROMBA 24.8.2010, 23:29

Цитата(Илья М @ 24.8.2010, 11:52) *
Потому это до сих пор не сделано, что сама по себе задача глупостью попахивает..... да и правильный ответ ясен и без дорогостоящих экспериментов любому трезво мыслящему инженеру-механику (мне во всяком случае)...)))

Ну что значит "дорогостоящими"? Когда хочецца приколоцца, отжечь нипадеццки, и оттянуцца в полный рост, разве цена вопроса имеет значение? Разве взлететь на такой вот тарахтелке с транспортера не интересно? Просто так, ради прикола...

Автор: инж323 25.8.2010, 1:59

Похоже проблемы прочности колеса поднятые Салтыковым -Щедриным до сих пор терзают пытливые умы. Уже и Рязань в тоске по тому колесу.
Может конкурс какой объявить- На право называться городом куда таки доехало то самое колесо?Или город в котором взлетел тот самый самолет? или еще как?

Автор: Сфинкс 25.8.2010, 5:07

Приведённый пример про Колю и санки НЕУДОВЛ, так как при вЫключении двигателя Коли, санки остаются на месте, привязанными к Коле. Но самолёт с вЫкл двигателями уедет взад.
Есть другая аналогия ЗА вас, она связана с *****е**ем.

Автор: Kult_Ra 25.8.2010, 8:07

Цитата
Стащит ли Коля санки с роликов?
Неа. Ролик мальчика заташит в сани!

Уронила в речку мячик. Наша Маша. Тише, реченька, не плач! Не утонет под Рязанью в Маше мяч.

Автор: Сергей В. 25.8.2010, 10:13

Цитата(инж323 @ 25.8.2010, 1:59) *
Похоже проблемы прочности колеса поднятые Салтыковым -Щедриным ...

wink.gif А по-моему Гоголем.
Так что памятник ставить в Миргороде biggrin.gif

Автор: Илья М 25.8.2010, 10:59

Цитата
Покрытие может двигаться против направления взлета самолета, т.е. ему навстречу. Оно имеет систему управления, которая отслеживает и подстраивает скорость движения полотна таким образом, чтобы скорость вращения колес самолета была равна скорости движения полотна.


Формально следуя тексту лента должна начать двигаться в момент начала движения самолёта.
Вопрос только куда?
Скорость вращения колеса= скорости движения полотна...Вопрос: как угловую скорость приравнять к линейной?

Ести считать скорость движения оси колеса за линейную скорость, то транспортёр начнёт двигаться против направления взлёта, а колесо начнёт крутится с удвоенной скоростью, что не сильно помешает взлёту.
Если считать за линейную скорость вращения скорость вращения окружных точек колеса относительно самого самолёта, то транспортёр поедет вслед самолёту с половинной скорости самолёта, что только поможет взлёту.
Если считать скоростью колеса скорость оси относительно земли, то транспортёр поедет в обратную от самолёта сторону со скоростью самого самолёта относительно земли.

В любом случае что-то куда-то покатится, а самолёт в любом случае взлетит.

Автор: dsemn 25.8.2010, 11:19

Как я понял, вся задачка сводится к одному вопросу: По какой причине взлетают самолеты? Причина: разность давления воздуха над крылом и под ним. Благодаря хитрой конфигурации и наличию угла атакиу крыла в процессе разгона самолета и обдува фюзеляжа и крыльев винтами, давление над крылом становится меньше, чем под ним. Эта разность стремится компенсировать себя, и какбы "всасывает" самолет. Самолет поднимается вверх. Ответ - не вздетит

Автор: Nurasha 25.8.2010, 11:27

согласна, не полетит! По-сути он ведь на месте стоит!? А для чего ваще такую штуку дклать? как бы Укорачивать взлетную полосу?

Автор: Сергей В. 25.8.2010, 16:11

Цитата(Nurasha @ 25.8.2010, 11:27) *
По-сути он ведь на месте стоит!?

wink.gif С чего Вы взяли?

Автор: HeatServ 25.8.2010, 21:26

Цитата(Nurasha @ 25.8.2010, 12:27) *
По-сути он ведь на месте стоит!?

Именно. Крутятся только колёса, которые на взлётные характеристики не влияют.
Цитата(Nurasha @ 25.8.2010, 12:27) *
А для чего ваще такую штуку дклать? как бы Укорачивать взлетную полосу?

Давно бы уже сделали, и в принципе реально, но перегрузки будут дикие.

Автор: Сергей В. 25.8.2010, 22:21

Цитата(HeatServ @ 25.8.2010, 21:26) *
... Давно бы уже сделали, и в принципе реально, но перегрузки будут дикие.

Как отметили предыдущие ораторы: пол-литра, сало и чеснок на одного пилота - это действительно сильные прегрузки tongue.gif

Автор: Nurasha 26.8.2010, 7:17

Цитата
С чего Вы взяли?

Я так поняла, что по условиям взлетная полоса навстречу самолету и с той же скоростью движется!!! Это ж просто беговая дорожка для самолета!!!!!!!!!!!!!! Вот когда по бег. дорожке бежишь - ветер в лицо не дует! rolleyes.gif bleh.gif т.е. нет встречных потоков, которые бы позволили нам(если мы были самолет или дельтоплан или что-то еще) взлететь! не правильно рассуждаю?

Автор: Kult_Ra 26.8.2010, 7:28

Самолёт полетит враз, если остановить вдруг Земли вращенье! Как верблюд с кручи. А вот куда пушка тогда полетит? ohmy.gif

Автор: Илья М 26.8.2010, 11:04

Цитата(Nurasha @ 26.8.2010, 8:17) *
Я так поняла, что по условиям взлетная полоса навстречу самолету и с той же скоростью движется!!! Это ж просто беговая дорожка для самолета!!!!!!!!!!!!!! Вот когда по бег. дорожке бежишь - ветер в лицо не дует! rolleyes.gif bleh.gif т.е. нет встречных потоков, которые бы позволили нам(если мы были самолет или дельтоплан или что-то еще) взлететь! не правильно рассуждаю?

Нет вы НЕПРАВИЛЬНО рассуждаете!
В задаче никто не ограничивает самолёт в его действиях, а лишь добавляют движущееся куда-то полотно полосы под колёсами.
То есть тягу двигателя самолёта никто не убирал, а вращающиеся без тормозов колёса затормозить самолёт нев состоянии вне зависимости от скорости и направления их вращения.
Получается, что самолёт взлетает, вообще при любом направлении движения полотна полосы.

Автор: Nurasha 26.8.2010, 11:25

ИльяМ написала я конечно с ошибкой, это сори! :-) Вот условие задачи:

Цитата
Оно имеет систему управления, которая отслеживает и подстраивает скорость движения полотна таким образом, чтобы скорость вращения колес самолета была равна скорости движения полотна.
именно одинаковая скорость. Но даже если и разная! 1. Если скорость больше у дорожки, самолет назад отъедет;
2. Скорость меньше у дорожки - самолету созданы вдвойне сложные условия, чтобы сдвинуться с мета ему надо преодолеть сопротивление дорожки! А чтобы взлететь, ему надо сдвинуться!
3. Скорости равны, все усилия самолета затрачиваются на то, чтобы оставаться на месте!
А что есть полет???? Возникновение ПОТОКА воздуха над и под крылом, для создания меньшего давления над крылом и большего под крылом!! Ключевое слово поток! Т.е. Если при всех этих условия задачи подует ТАКОЙ ветрище, что сможет поднять самолет, то да, он полетит!

вот еще! Относительно дорожки скорость самолета велика, о да! А относительно Земли - 0! При отсутвии скорости как взлетим?

Автор: Илья М 26.8.2010, 11:42

Cударыня, вы вероятно плохо понимаете суть явлений, происходящих при взлёте самолёта.
По идеи влияние колёс вообще отсутствует, так как их легко заменяет воздушна подушка с нулевым трением покоя.

Если самолёт стоит, то и колёса стоят, а если стоят колёса, то и полотно никуда не движется.
Как только двигатель самолёта опёрся на ВОЗДУХ, то в этот момент начинается разгон, НО колёса в разгонне НЕ участвуют!!!..не считая мизерного сопротивления на трение в шариковых подшипниках ступиц, которым в этом случае можно пренебреч...

Автор: даниил 26.8.2010, 11:56

Цитата
А что есть полет???? Возникновение ПОТОКА воздуха над и под крылом


вертолеты вон вопще без крыльев летают!!!
то есть они ими не только не машут, а их(крыльев) у них вопще нету!!!
как так?

Автор: Илья М 26.8.2010, 12:02


Кстати, откуда в задаче взялся сильный встречный ветер?
Транспортёр под колёсами его точно создать никак не может!
Ведь нам никто не сказал, что этот транспортёр шире самих колёс!...а для решения задачи нужна только опора колёс на транспортёр, а делать транспортёр шириной со всю ВПП, да ещё с ветряными лопатками для большего ветросопротивления нас никто не обязывал по условиям задачи.

Автор: Nurasha 26.8.2010, 12:03

unsure.gif
хорошо! я ничего не понимаю! И вертолет тут не виноват!
более не вмешиваюсь!
(а тут бы исчезнуть как Чеширскому коту!)

Автор: Илья М 26.8.2010, 12:15

Вот именно за вот такую неадекватную и крайне эмоциональную реакцию на обычные возражения по существу обсуждаемого вопроса и не любят женщин в технике...(((

Если расписать условия задачи в векторной форме на бумаге, то окажется, что вообще никакой проблемы нет!
А вот пустопорожнее жонглирование словесными страшилками и приводит к возникновению ситуации, что самолёт куда-то уедет задом наперёд...)))

Автор: Nurasha 26.8.2010, 12:18

ИльяМ, мне честно хоца поспорить! Но словами лучше получается, а писать, для меня трудно! и оч. деревянно получается! В сравнении с вашими летающими, звучащими фразами, я как щенок писклявый! Вот и удаляюсь. Но читать не перестану! Буду тут, на рабочем месте возмущаться! Удачи!

Автор: Илья М 26.8.2010, 13:13

Всё же надо поставить точку в этом обсуждении.
Вот исходная фраза, которая и создаёт всеобщее непонимание.

Цитата
Оно имеет систему управления, которая отслеживает и подстраивает скорость движения полотна таким образом, чтобы скорость вращения колес самолета была равна скорости движения полотна.

Тезис 1. СКОРОСТЬ САМОЛЁТА ЗДЕСЬ НИКЕМ НЕ ОПРЕДЕЛЕНА и НИ С ЧЕМ НЕ СВЯЗАНА!
Тезис 2. Колесо ВРАЩАЕТСЯ, а это значит, что дорожка движется в некотором направлении относительно оси колеса, при этом скорость в пятне контакта колеса с дорожкой у точки колеса в контакте с полотном и точки полотна равны (колесо по полотну не проскальзывает).
Тезис 3. Изначально самолёт стоял неподвижно, то есть скорости всех его элементов были нулевыми, откуда следует, что и транспортёрная лента изначально была неподвижна.
именно одинаковая скорость.

Теперь разберём получившуюся картину.
В начальный момент колесо неподвижно.
В момент начала движения самолёта колесо начинает катиться в перёд...НО при этом точка в контакте с грунтом остаётся НЕПОДВИЖНОЙ ВСЕГДА!!!
Таким образом, скорость полотна должна корелировать себя со скоростью ОСИ КОЛЕСА, которая равна скорости самого самолёта!

Далее фигурирует СКОРОСТЬ ПОЛОТНА, которая имеет смысл только относительно неподвижной земли.
А равна эта СКОРОСТЬ ПОЛОТНА (м/с) скорости ВРАЩЕНИЯ (радиан/с) колеса!!!...тут и зарыт корень этой заковыристой задачки.

НЕВОЗМОЖНО ПРИРАВНЯТЬ ЕДИНИЦЫ РАЗНЫХ ФИЗИЧЕСКИХ РАЗМЕРНОСТЕЙ!!!

Как только мы вскроем это противоречие, задачка станет простой и понятной.

Вариант первый: Мы меряем скорость оси колеса относительно земли и приравниваем её к скорости движения полотна относительно земли.
В этом случае полотно может двигаться как вместе с самолётом (колесо вообще не вращается), либо в противоположную сторону от самолёта (колесо вращается с удвоенной частотой).
В обоих вариантах самолёт взлетит, хотя во втором случае это потребует чуть большей тяги от мотора.

Вариант второй: Мы меряем характерную частоту вращения колеса, считая скоростью вращения колеса скорость точек колеса на беговой дорожке шины относительно самолёта. При этом скорость полотна мы меряем относительно земли.
Получается, что скорость движения точки колеса равна скорости движения полотна, НО измерены эти скорости относительно РАЗНЫХ СИСТЕМ КООДИНАТ!!! (одна относительно земли, а другая относительно самолёта).
В этом случае решений опять ДВА:
1. Полотно движется относительно самолёта в ту же сторону относительно земли, но с половиной скорости самолёта. Колесо вращается также с половинной характерной частотой.
В этой ситуации самолёт взлетает легче, чем при взлёте с неподвижной ВПП.
2. Полотно движется в любую сторону относительно самолёта, НО при этом самолёт неподвижен относительно земли.
В этом случае НЕПОДВИЖНЫЙ самолёт никуда не взлетает, а ситуация похожа на испытания автомобильных тормозов на вращающихся барабанах пункта тех.осмотра.

Вывод:
Задача была для юриста, а не для инженера.
Такие задачи просто выбраковываются из-за некоректности условий, так как в некоректной задаче получается вообще практически любой ответ!!!!

Автор: Сфинкс 26.8.2010, 14:05

А по-моему, перевод из рад/сек в руб/сек... ой, я хотел сказать в м/сек, такой перевод изучают в теоретической механике.
Спокойно замените движ полотно на множество колёс, того же диаметра что и колесо самолёта (и ещё есть варианты), авось полегчает.

Автор: Илья М 26.8.2010, 14:31

В школьной физике угловые и линейные скорости тоже изучают....)))

Ну, а как только я полезу с этим переводом в условие задачи, то задача сразу станет совсем другой, а именно: простой и понятной, то есть без всякой интриги...(((

Автор: Usach 27.8.2010, 8:31

Ответ в вопросе: А на хрена тогда вертолёт изобрели?

Автор: Usach 27.8.2010, 8:36

Цитата(Nurasha @ 26.8.2010, 15:25) *
ИльяМ написала я конечно с ошибкой, это сори! :-) Вот условие задачи:
именно одинаковая скорость. Но даже если и разная! 1. Если скорость больше у дорожки, самолет назад отъедет;
2. Скорость меньше у дорожки - самолету созданы вдвойне сложные условия, чтобы сдвинуться с мета ему надо преодолеть сопротивление дорожки! А чтобы взлететь, ему надо сдвинуться!
3. Скорости равны, все усилия самолета затрачиваются на то, чтобы оставаться на месте!
А что есть полет???? Возникновение ПОТОКА воздуха над и под крылом, для создания меньшего давления над крылом и большего под крылом!! Ключевое слово поток! Т.е. Если при всех этих условия задачи подует ТАКОЙ ветрище, что сможет поднять самолет, то да, он полетит!

вот еще! Относительно дорожки скорость самолета велика, о да! А относительно Земли - 0! При отсутвии скорости как взлетим?


Ага! И где это видано, чтоб самолет взлетал при сильном встречном или боковом ветре? Это аппарат тяжелее воздуха, а не стратостат. И ещё самолёт для взлёта должен двигаться тока "вперёд", такова конструкция крыла.

Автор: vovganmgk 16.9.2010, 12:12

самолет сможет оторваться от поверхности на очень малый промежуток времени и на столь малое растоняие что фактические взлетом это считать нельзя.
ОТВЕТ: не взлетит

P.S. надо данную задачку подбросить в передачу "Мозголомы" они на практике поставять эксперемент и дадут ответ rolleyes.gif

Автор: S.Pavel 16.9.2010, 12:21

Млин, проголосовал, а потом подумал ))))

Не взлетит. Физика и аэродинамика ни причем - чтоб взлететь, надо двигаться относительно воздуха, а двигаться относительно воздуха не позволяет система управления дорожкой "... подстраивает скорость дорожки так, чтобы скорость вращения колес компенсировалась..." т.е. самолет ВСЕГДА неподвижен относительно воздуха(или земли).

Автор: Илья М 30.9.2010, 10:59

Цитата(S.Pavel @ 16.9.2010, 13:21) *
Млин, проголосовал, а потом подумал ))))

Не взлетит. Физика и аэродинамика ни причем - чтоб взлететь, надо двигаться относительно воздуха, а двигаться относительно воздуха не позволяет система управления дорожкой "... подстраивает скорость дорожки так, чтобы скорость вращения колес компенсировалась..." т.е. самолет ВСЕГДА неподвижен относительно воздуха(или земли).


Задача ставится о взлёте самолёта по движущейся на встречу дорожке, а не о системе управления движением дорожки по некоему алгоритму.
Таким образом самолёту наплевать на скорость вращения его колёс после того как его начали тянуть его винты или турбины!

Если поставить самолёт на лыжи, чтобы больше ничего не вращалось, то взлетит ли самолёт по движущейся на встречу ленте?

Автор: LordN 30.9.2010, 11:48

Цитата
Если поставить самолёт на лыжи, чтобы больше ничего не вращалось, то взлетит ли самолёт по движущейся на встречу ленте?
а еще лучше прибить лыжи. гвоздями. или шурупами. и не к ленте, а прямо к ВПП. и посмотреть - а полетит ли самолет? земля-то ведь тоже крутится! а если полетит - то что окажется крепче - стойки шасси, шурупы или бетон? а може и земля полетит вослед самолёту? и тогда - во прикол, можно купить самолет, прибить гвоздями к земле и летай куда хочу не опасаясь разбится.

Автор: Илья М 30.9.2010, 12:14

Цитата(LordN @ 30.9.2010, 12:48) *
а еще лучше прибить лыжи. гвоздями. или шурупами. и не к ленте, а прямо к ВПП. и посмотреть - а полетит ли самолет? земля-то ведь тоже крутится! а если полетит - то что окажется крепче - стойки шасси, шурупы или бетон? а може и земля полетит вослед самолёту? и тогда - во прикол, можно купить самолет, прибить гвоздями к земле и летай куда хочу не опасаясь разбится.

А если МОДЕРАТОР несёт околесицу, то можно ли ему в этом сделать замечание, или это противоречит правилу форума?

Автор: LevaK 30.9.2010, 12:22

Взлетит.
Самолету НИЧТО не мешает разогнаться относительно земли и воздуха.
А колеса пусть себе крутятся как хотят.

Автор: Илья М 30.9.2010, 12:31

Цитата(LevaK @ 30.9.2010, 13:22) *
Взлетит.
Самолету НИЧТО не мешает разогнаться относительно земли и воздуха.
А колеса пусть себе крутятся как хотят.


НУ, ЕСТЬ ЗДЕСЬ ВСЁ-ТАКИ ТРЕЗВО МЫСЛЯЩИЕ ЛЮДИ!!!

Автор: CNFHSQ 30.9.2010, 13:00

Вот уж не думал что обсуждаю всерьез. Задачка для 7 класса. Подъемная сила появляется только если крылья самолета перемещаются относительно воздуха вперед или даже назад, а чем это достигается не важно, можно вентилятором, двигателем, лентой, лашадью и т.д.

Автор: LordN 30.9.2010, 13:20

вот и я говорю - земля же крутится - значит самолет летает.

Автор: Сандугач 7.10.2010, 9:27

За качество снимка очень извиняюсь, но вот практическая задачка из крайней поездки. Надо оценить скорость течения. Высота бакена 1м. Поплавки- брёвна отпиленные пилой "дружба-2"

Автор: Сандугач 8.10.2010, 10:33

Вот те на.. А Бакен то оказывается и не прикрепился прошлый раз.. Вот он.

 

Автор: LordN 8.10.2010, 13:40

Цитата
.. Вот он.
легче не стало. и так не видно и эдак

Автор: Сандугач 10.10.2010, 9:02

Цитата(LordN @ 8.10.2010, 16:40) *
легче не стало. и так не видно и эдак



Бурун от набегающей воды поднимается на пол-бакена примерно. То есть около полуметра. Даже если не учитывать коэффициента местного сопротивления (ну негде мне его посмотреть сейчас, хотя, для цилиндра обрезанного перпендикулярно оси, помнится он был около 2), допустив, что весь скоростной напор используется на подъём воды, получаем скорость более трёх метров в секунду. То есть почти 11 км в час. С учётом коэффициента, упомянутой мной в скобках двойки, которая под корнем, выходит около 15 км в час. По таблицам, которые есть на танкере при тех оборотах, что давал двигатель без баржи должно было быть 19. С баржой, по словам капитана примерно 15-16. Для оценки такая высокая точность у меня вызывает подозрение, может, где лопухнулся.... в главном. Типа формулы скоростного напора (жаль смайлик не грузится... ) подглядеть негде, справочников у меня с собой нету.. так что жду, когда табуретки полетят...

Автор: Kult_Ra 10.10.2010, 10:30

Цитата
так что жду, когда табуретки полетят...
По команде "Корабль к бою и походу приготовить" = тубаретки, вернее, "баночки" и столы и пр. лабуда принайтовываются (следует принайтовать к палубе). Для чего есть у каждого предмета "штатные" причиндалы. Чтоб не ждать.

Бурун от набегающей воды - иногда используется для стирки шинели! Берёшь шкерт ... и за борт с кормы - как в стиральную машину

Автор: Сандугач 10.10.2010, 16:56

Цитата(Kult_Ra @ 10.10.2010, 13:30) *
По команде "Корабль к бою и походу приготовить" = тубаретки, вернее, "баночки" и столы и пр. лабуда принайтовываются (следует принайтовать к палубе).


Эта табуретка (замечание) принимается. Теперь жду по существу задачки, которую сам себе поставил и с которой поделился.

Автор: Serg Ivanov 18.10.2010, 10:54

http://www.popmech.ru/article/7666-po-vetru-byistree-vetra/ wink.gif

Рик и Джей Би предложили фанатам управляемого полета найти внятное научное объяснение феномену движения парусника или наземного аппарата на строго попутном ветре со скоростью быстрее ветра. А еще лучше – разработать конструкцию аппарата, который способен на это.(С)
http://vimeo.com/10477373

 

Автор: HeatServ 18.10.2010, 21:52

Цитата(Serg Ivanov @ 18.10.2010, 11:54) *
http://vimeo.com/10477373

Супер!

Автор: Serg Ivanov 20.10.2010, 8:36

Опыт объясняющий принцип движения:
http://www.youtube.com/watch?v=k-trDF8Yldc&feature=player_embedded

Автор: Сергей Валерьевич 21.10.2010, 8:12

Цитата(Serg Ivanov @ 18.10.2010, 10:54) *
http://vimeo.com/10477373

Так эт что, уже сверх вечный двигатель? с кпд 350% ? Блин, ну теперь на всех автомагистралях трубы с управляемым потоком ветра и вперед!!!!! Экономия горючки, а ОВшникам вечная работа....

Автор: Илья М 21.10.2010, 9:34

Объяснение обгона попутного ветра выглядит чистой ахинеей, также как и парадокс Эсхила об Ахилле и черепахе.

ВОПРОС: Что происходит в этой ЧУДО-повозке при скорости движения равной скорости ветра?

Ответ: В это время действует только сопротивление качению колёс! ..и никакой движущей сил от воздуха, причём ни на разгон ни на торможение. Из чего можно сделать вывод, что в стационарном режиме движение против ветра без затрат энергии извне по закону сохранения энергии невозможно.

ПРАВДА есть одно НО!

В этой чудо-машине создана некая спекулятивная конструкция: ГИГАНТСКИЙ ВИНТ с управляемым шагом винта.

Это означает, что при движении в режиме медленнее ветра колёса действительно раскручивают винт за счёт энергии ветра.
При этом шаг винта подгоняют под относительную скорость ветра, чтобы он дополнительно раскручивался ПОПУТНЫМ ветром.

Как только накопленный в ГИГАНТСКОМ ВИНТЕ запас энергии будет достаточно велик, в этот момент его лопасти переключают в режим тяги.

Быстро вращающиеся лопасти начинают отталкиваться от воздуха (теряя накопленную кинетическую энергию), и винт начинает разгонять повозгу выше скорости ветра!!!.....правда не на долго, а именно пока не затормозится винт....(((

Так что никаких чудес, а лишь мелкое мошейничество в изложении статьи....(((

С тем же успехом можно сделать первоначальный разгон и накопление энергии ветра на обычном парусе, а энергию запасать от колёс на механический или электрический аккумулятор. После чего складывать парус и возвращать энергию обратно на колёса или на тяговый воздушный винт, тем самым преодолевая скорость ветра вполне по законам физики.


Мой вердикт: Ради таких текнических фокусов не стоило тратить столько сил....(((

Автор: даниил 21.10.2010, 10:07

Ерунда это все!
Тема про яйцекуров гораздо интереснее.

Автор: HeatServ 21.10.2010, 20:35

Да не, вполне реальная штуковина. Тепловые насосы же существуют? Извлекают тепло повышенного потенциала из тепла низкого потенциала? Ну вот так и здесь, только энергия механическая, а не тепловая. Никакого вечного двигателя тут нет.
А запасти механическую энергию в винте... нужно слишком тяжёлый винт.

Автор: Илья М 22.10.2010, 8:26

Цитата(HeatServ @ 21.10.2010, 21:35) *
Да не, вполне реальная штуковина. Тепловые насосы же существуют? Извлекают тепло повышенного потенциала из тепла низкого потенциала? Ну вот так и здесь, только энергия механическая, а не тепловая. Никакого вечного двигателя тут нет.
А запасти механическую энергию в винте... нужно слишком тяжёлый винт.


...вот только Тепловые насосы не нарушают закона сохранения энергии при выкачивании тепла!

Кстати, а где грань между " просто тяжёлым" и "слишком тяжёлым" винтом на ваш взгляд?
Как вы это можете определить на взгляд по фотографиям?
.....при одинаковой массе винта я могу изменить его момент инерции (и ,соответственно, запасаемый энергетический потенциал при равных скоростях вращения) в несколько раз переносом небольших грузов с концов лопостей на ось вращения.

И вообще, вам хоть какие-то цифровые параметры этой чудо-повозки известны, кроме разгона выше скорости ветра в 2..3 раза?

Автор: Serg Ivanov 22.10.2010, 8:34

Цитата(HeatServ @ 21.10.2010, 20:35) *
А запасти механическую энергию в винте... нужно слишком тяжёлый винт.

Винт лёгкий из пенопласта оклеен стеклотканью и приводится во вращение колесами. Это хорошо видно по углу атаки лопастей и направлению вращения. rolleyes.gif

Цитата(Сергей Валерьевич @ 21.10.2010, 8:12) *
Так эт что, уже сверх вечный двигатель? с кпд 350% ? Блин, ну теперь на всех автомагистралях трубы с управляемым потоком ветра и вперед!!!!! Экономия горючки, а ОВшникам вечная работа....

Увы, нет. При отсутствии ветра - не едет. wink.gif

Автор: Serg Ivanov 22.10.2010, 8:51

Цитата(Илья М @ 21.10.2010, 9:34) *
Мой вердикт: Ради таких текнических фокусов не стоило тратить столько сил....(((

Это не фокус. Рекордная скорость заезда зарегистрирована официально вполне уважаемой в мире организацией.
Внимательно посмотрите на направление вращения винта и положение лопастей - он никак не может раскручиваться попутным ветром.
Что касается движения быстрее ветра аппаратов движимых ветром - так то давно известный факт для парусников, буеров и т.п.
Новизна в том, что этот движется по ветру быстрее ветра. И здесь действительно нужен хороший винт.

Цитата(Илья М @ 22.10.2010, 8:26) *
...вот только Тепловые насосы не нарушают закона сохранения энергии при выкачивании тепла!

Так и это устройство не нарушает. Нет ветра - нет движения.

Автор: Илья М 22.10.2010, 9:09

Цитата(Serg Ivanov @ 22.10.2010, 9:51) *
Это не фокус. Рекордная скорость заезда зарегистрирована официально вполне уважаемой в мире организацией.
Внимательно посмотрите на направление вращения винта и положение лопастей - он никак не может раскручиваться попутным ветром.
Что касается движения быстрее ветра аппаратов движимых ветром - так то давно известный факт для парусников, буеров и т.п.
Новизна в том, что этот движется по ветру быстрее ветра. И здесь действительно нужен хороший винт.


ВЫ описание читали?...ВИНТ С ИЗМЕНЯЕМЫМ ШАГОМ!!!

Про сам факт превышени скорости ветра на ЭНЕРГИИ ВЕТРА никто не спорит....вот только моменты запасения энергии и использования её на разгон разнесены во времени.
Разница между нами лишь в объяснениях природы полученного эффекта.
При желании вполне можно превысить скорость ветра и в 10 раз на некоторое время!
Для этого достаточно обеспечить большее количество накопленной энергии в маховиках при более длительном разгоне на скорости ниже скорости ветра.

Вот только никакого полезного использования в комерческих целях это не даёт, так как краткосрочный разгон быстрее ветра не компенсирует более длительный разгон медленнее ветра.
Это легко выясняется паралельным заездом обычного парусника со шпинакером и этого чудо-винтокрыла. Буер со шпинакером прибудет на много раньше на финишь с о скоростью чуть менее скорости ветра.

Автор: Serg Ivanov 22.10.2010, 11:39

Цитата(Илья М @ 22.10.2010, 9:09) *
ВЫ описание читали?...ВИНТ С ИЗМЕНЯЕМЫМ ШАГОМ!!!

Про сам факт превышени скорости ветра на ЭНЕРГИИ ВЕТРА никто не спорит....вот только моменты запасения энергии и использования её на разгон разнесены во времени.
Разница между нами лишь в объяснениях природы полученного эффекта.
При желании вполне можно превысить скорость ветра и в 10 раз на некоторое время!
Для этого достаточно обеспечить большее количество накопленной энергии в маховиках при более длительном разгоне на скорости ниже скорости ветра.

Вот только никакого полезного использования в комерческих целях это не даёт, так как краткосрочный разгон быстрее ветра не компенсирует более длительный разгон медленнее ветра.
Это легко выясняется паралельным заездом обычного парусника со шпинакером и этого чудо-винтокрыла. Буер со шпинакером прибудет на много раньше на финишь с о скоростью чуть менее скорости ветра.

Ну тогда ответьте мне на простой вопрос- на видео винт начинает вращаться ПРОТИВ направления ветра. С шагом ПРОТИВ ветра, и обороты растут синхронно с оборотами колёс. Как?
Сей ветроход может сколь угодно долго ехать по ветру быстрее ветра. Иначе рекорды не регистрируются.
http://www.youtube.com/watch?v=5CcgmpBGSCI&feature=player_embedded
Главные вводные:
1) ветряк работает не как парашют, а как крыло.
2) энергия берется от разницы скоростей между землей (Vз=0) и ветром (Vв>0), даже при скорости выше скорости реального ветра.


Большинство людей знают, как работает крыло, но не понимают главного: подъемная сила создаваемая крылом по модулю больше лобового сопротивления набегающего потока и по направлению перпендикулярна направлению движения набегающего потока. Мысль эта проста, но не очевидна. Тяга создаваемая ветряком больше сопротивления создаваемого этим ветряком. (Это не парашют!!!)

Шаг лопасти ветряка на телеге выбирается таким образом, чтобы при движении телеги быстрее ветра, вращение ветряка создавало бы тягу вперед, при этом колесо, отталкиваясь от земли, вращает ветряк.

Разница скоростей земли и воздуха создают дополнительную составляющую в тяге ветряка и компенсирует потери в системе на скорости чуть больше двух скоростей ветра.(С)

Автор: Илья М 22.10.2010, 12:52

Цитата(Serg Ivanov @ 22.10.2010, 12:39) *
Ну тогда ответьте мне на простой вопрос- на видео винт начинает вращаться ПРОТИВ направления ветра. С шагом ПРОТИВ ветра, и обороты растут синхронно с оборотами колёс. Как?

Для начала я посоветовал бы вам коректнее формулировать мысль, иначе у вас начинаются сравнени длинного и зелёного по температуре...)))

ВИНТ вращается ПОПЕРЁК ВЕТРА, а вот ЛОПАСТЬ может иметь угол наклона к ОТНОСИТЕЛЬНОЙ СКОРОСТИ своего движения, причём положительный или отрицательный.
Так как мы на видео можем определить угол наклона лопости к ВЕКТОРУ скорости, если вектор- воображаемый, а воздух и его скорость на видео не видны?


Цитата(Serg Ivanov @ 22.10.2010, 12:39) *
Сей ветроход может сколь угодно долго ехать по ветру быстрее ветра. Иначе рекорды не регистрируются.
http://www.youtube.com/watch?v=5CcgmpBGSCI&feature=player_embedded

Это ложь!
На соляных озёрах нет условии для длинных заездов, а скорость регестрируют на небольшом участке трассы, оборудованном измерительной аппаратурой, так называемая "мерная миля".

Вот до этой мили ветроезд может разгоняться некоторое расстояние, а перед милей осуществляет резкий разгон до максимальной скорости.

Цитата(Serg Ivanov @ 22.10.2010, 12:39) *
Главные вводные:
1) ветряк работает не как парашют, а как крыло.
2) энергия берется от разницы скоростей между землей (Vз=0) и ветром (Vв>0), даже при скорости выше скорости реального ветра.


1) ветряк работает СНАЧАЛА как ПАССИВНЫЙ ПАРУС, а после накопления достаточной кинетической энергии на механическом (маховым) аккумуляторе как ТЯГОВЫЙ ВИНТ.
2) Это вы тут так запросто нарушили закон сохранения энергии и второе начало термодинамики одновременно.

Цитата(Serg Ivanov @ 22.10.2010, 12:39) *
Большинство людей знают, как работает крыло, но не понимают главного: подъемная сила создаваемая крылом по модулю больше лобового сопротивления набегающего потока и по направлению перпендикулярна направлению движения набегающего потока. Мысль эта проста, но не очевидна. Тяга создаваемая ветряком больше сопротивления создаваемого этим ветряком. (Это не парашют!!!)

Если люди знают как работает крыло, то и главное они понимают!....либо они вообще ничего не понимают в аэродинамике...)))

А вот вы путаете силу создающую работу(у крыла это лобовое сопротивление парлельное скорости движени) и статическую силу удержания, работы не создающей (у крыла это подъёмная сила перпендикулярная скорости и паралельная силе тяжести).
В случае с ТЯГОВЫМ винтом обе эти силы создают работу, только относительно разных векторов скорости и разных траекторий: так лобовое сопротивление лопости надо умножать на длину винтовой-спиральной линии точки на вращающемся винте, а подъёмную силу лопасти (тяговую силу винта) нужно умножать на скорость перемещения винта в пространсве вдоль оси его вращения.


Цитата(Serg Ivanov @ 22.10.2010, 12:39) *
Шаг лопасти ветряка на телеге выбирается таким образом, чтобы при движении телеги быстрее ветра, вращение ветряка создавало бы тягу вперед, при этом колесо, отталкиваясь от земли, вращает ветряк.

Разница скоростей земли и воздуха создают дополнительную составляющую в тяге ветряка и компенсирует потери в системе на скорости чуть больше двух скоростей ветра.(С)


Вы только определитесь сначала откуда телега будет брать энергию для преодоления сопротивления качению колёс и сопротивлению вращению винта в ситуации когда повозка уже едет со скоростью ветра?
При этом считаем , что запаса энергии из прошлого у повозки нет.

Похоже, что вы так хотите верить в СКАЗОЧНОЕ подвирание журналистов при изложении ими увиденного, что критически и самостоятельно анализировать суть показаных явлений просто не в состоянии.
К сожалению, но переубидить человека в чём-либо против его желания никто не в силах....((((

Автор: Serg Ivanov 22.10.2010, 13:25

http://www.youtube.com/watch?v=5CcgmpBGSCI&feature=player_embedded
24-я секунда видео, начало разгона. Ветер попутный, винт медленно вращается по часовой стрелке если смотреть по ходу движения. Угол атаки лопастей виден прекрасно. Так как он может раскручиваться попутным ветром по часовой стрелке?
Не надо столько букфф тратить. Я верю своим глазам.
тут можно почитать подробней объяснение этого кажущегося парадокса- http://www.popmech.ru/article/7666-po-vetru-byistree-vetra/

 

Автор: Serg Ivanov 22.10.2010, 13:46

Цитата(Bers @ 3.11.2006, 7:20) *
Есть такая задачка:

"Самолет (реактивный или винтовой) стоит на взлетной полосе с подвижным покрытием (типа транспортера). Покрытие может двигаться против направления взлета самолета, т.е. ему навстречу. Оно имеет систему управления, которая отслеживает и подстраивает скорость движения полотна таким образом, чтобы скорость вращения колес самолета была равна скорости движения полотна.
Внимание, вопрос: сможет ли самолет разбежаться по этому полотну и взлететь?"

Этот самолёт на видео biggrin.gif ..но без крыльев.
http://www.youtube.com/watch?v=1BRvYZd81AQ&feature=player_embedded

Автор: Vict 22.10.2010, 14:32

Илья М, взгляните http://www.livejournal.ru/themes/id/12317

Автор: Илья М 22.10.2010, 15:24

Цитата(Vict @ 22.10.2010, 15:32) *
Илья М, взгляните http://www.livejournal.ru/themes/id/12317

Посмотрел...Видео как всегда оставляет больше вопросов , чем даёт ответы...(((

В последний раз в ютубе я видел действующий макет вечного двигателя....вот только за кадром там оказался вентилятор, обдувавший вращающееся колесо лишь с одной стороны.
Здесь движение на транспортёре также будет объясняться вентилятором за кадром, скорее всего.....Ну, или какой-то другой хитрой невидимой побуждающей силой, например наклоном ленты при неверных показаниях пузырька на уровне.

Автор: Vict 22.10.2010, 15:35

а то что на асфальте...?

Автор: Илья М 22.10.2010, 15:47

Цитата(Vict @ 22.10.2010, 16:35) *
а то что на асфальте...?

Это про длинный проезд винтокрылой тележки рядом с едущим автомобилем?
Так такой постановочный фильм можно снять самыми разными способами: начиная с невидимого уклона до буксирвки малозаметным тросом.

Развод всё это!....А на соляном озере был обычный механический энергонакопитель с ветряным приводом разгона и торможения, механизм которого я описал ранее и такое же описание было в твоей ссылке...

Автор: Vict 22.10.2010, 15:52

Цитата(Илья М @ 22.10.2010, 15:47) *
Это про длинный проезд винтокрылой тележки рядом с едущим автомобилем?
нет, про другой попроще, за которым бегут... и эту же конструкцию специально на транспортере проводит сквозь "арки" что бы показать отсутствие нитей, а также моменты касания колес ленты и когда начинает крутиться пропеллер(эт я про вентилятор)... касаемо уровня - его можно и косвенно оценить по рядом стоящим оборудованием.

ПС. еще в школе когда я учился, на уроке физике нечто подобное нам демонстрировал учитель...уверяю, шо никаких приводов извне не было... тогда меня это не особо интересовало... да и сейчас тоже smile.gif

Автор: Serg Ivanov 22.10.2010, 16:07

Цитата(Илья М @ 22.10.2010, 15:47) *
Это про длинный проезд винтокрылой тележки рядом с едущим автомобилем?
Так такой постановочный фильм можно снять самыми разными способами: начиная с невидимого уклона до буксирвки малозаметным тросом.

Развод всё это!....А на соляном озере был обычный механический энергонакопитель с ветряным приводом разгона и торможения, механизм которого я описал ранее и такое же описание было в твоей ссылке...

Если Вам это удастся доказать этим лохам:
http://www.popmech.ru/article/7666-po-vetru-byistree-vetra/
Весной этого года болид был полностью готов к бою и во время восьми пробных заездов на короткой взлетке аэродрома «Новый Иерусалим» при попутном ветре в 21,73 км/ч разогнался до 61,93 км/ч, в 2,85 раза быстрее воздушного потока. Блестящий результат в первых же испытаниях воодушевил команду «Дрозда», и в мае Кавалларо начал переговоры с важными птицами из Североамериканской ассоциации скоростных наземных парусных судов NALSA об организации рекордных заездов в совершенно новом классе аппаратов. В июне согласие NALSA было получено, и вначале июля на высохшем озере Эль Мираж Blackbird, обвешанный многочисленными датчиками, несколько раз промчался со скоростями, превышающими поток в 3,5раза. Самыми неудачными в сете стали попытки с коэффициентом 2,5 при рваном ветре.

Невероятный аппарат Кавалларо и Бортона настолько понравился ребятам из NALSA, что они решили не брать с «дроздов» вступительный взнос. На момент, когда пишутся эти строки, решение NALSA об установлении официального мирового рекорда еще не получено, но то, что он состоялся,– факт. Хотя бы по той причине, что «Дрозд» – пока единственный претендент в своем классе.(С) - то, ИМХО, всю оставшуюся жизнь Вы сможете не работать. smile.gif

Автор: HeatServ 23.10.2010, 2:55

Цитата(Илья М @ 22.10.2010, 9:26) *
И вообще, вам хоть какие-то цифровые параметры этой чудо-повозки известны, кроме разгона выше скорости ветра в 2..3 раза?

У меня на кухне ходолильник есть, и я знаю, что если его открыть, то на кухне станет теплее.

Автор: BROMBA 23.10.2010, 10:54

Цитата(HeatServ @ 23.10.2010, 2:55) *
У меня на кухне ходолильник есть, и я знаю, что если его открыть, то на кухне станет теплее.

Годная попытка...
А теперь небольшое напоминание: в теме мельканула фраза о том, что, мол, винт - не парашют...

Но ведь есть такой летательный аппарат тяжелее воздуха под названием "автожир"...

У кого-нить есть пара фраз по этому поводу для журнала "Совеццкое коневодство"?

Автор: Илья М 25.10.2010, 7:11

Цитата(HeatServ @ 23.10.2010, 3:55) *
У меня на кухне ходолильник есть, и я знаю, что если его открыть, то на кухне станет теплее.


....вот только надо уточнить, что теплее на кухне от ОТКРЫТОГО холодильника станет только тогда, когда суммарные энерго затраты компрессора перекроют энергию для нагрева содержимого холодильника до комнатной температуры.

В данном случае аналогия может быть продолжена: чудо-повозка может разогнаться быстрее ветра за счёт действия ветра только тогда, когда накопленная предварительно энергия в маховых накопителях превысит потребную энергию для разгона этой повозки выше скорости ветра.

Автор: Сергей Валерьевич 25.10.2010, 7:40

Цитата(Илья М @ 25.10.2010, 7:11) *
....вот только надо уточнить, что теплее на кухне от ОТКРЫТОГО холодильника станет только тогда, когда суммарные энерго затраты компрессора перекроют энергию для нагрева содержимого холодильника до комнатной температуры.

Ну почему же.. можно холодильник и в ярангу поставить, без костра... Говорят, чукчи так греются... правда рядом генератор ставить нужно, но это мелочи..

Автор: Serg Ivanov 25.10.2010, 9:54

Цитата(Илья М @ 25.10.2010, 7:11) *
В данном случае аналогия может быть продолжена: чудо-повозка может разогнаться быстрее ветра за счёт действия ветра только тогда, когда накопленная предварительно энергия в маховых накопителях превысит потребную энергию для разгона этой повозки выше скорости ветра.

Увы, там нет маховых накопителей. wink.gif Едет без них, зараза... biggrin.gif

Автор: Илья М 25.10.2010, 11:15

Цитата(Serg Ivanov @ 25.10.2010, 10:54) *
Увы, там нет маховых накопителей. wink.gif Едет без них, зараза... biggrin.gif

А почему вы сбрасываете со счетов энергю накопленную в этом гигантском винте?
Ведь сам этот ГИГАНТСКИЙ винт и есть тот самый маховый накопитель!.....ГИГАНТСКИЙ в сравнении с остальной сверхоблегчённой повозкой...

Или вы никогда не видели ручных вертолётиков, взлетающих вверх только за счёт энергии раскрученного пропеллера?


Автор: Serg Ivanov 25.10.2010, 11:42

Цитата(Илья М @ 25.10.2010, 11:15) *
А почему вы сбрасываете со счетов энергю накопленную в этом гигантском винте?
Ведь сам этот ГИГАНТСКИЙ винт и есть тот самый маховый накопитель!.....ГИГАНТСКИЙ в сравнении с остальной сверхоблегчённой повозкой...

Или вы никогда не видели ручных вертолётиков, взлетающих вверх только за счёт энергии раскрученного пропеллера?

Потому, что я посмотрел видео. Внимательно посмотрел. Чего и Вам советую прежде чем выдвигать гипотезы. rolleyes.gif

Автор: Илья М 25.10.2010, 11:54

Цитата(Serg Ivanov @ 25.10.2010, 12:42) *
Потому, что я посмотрел видео. Внимательно посмотрел. Чего и Вам советую прежде чем выдвигать гипотезы. rolleyes.gif

И что вы увидели там такого, что опровергает закон сохранения эненргии и второе начало термодинамики?

Кстати, видеозапись даже в суде не может быть доказательством чего-либо....(((

А для научной публикации требуется описать схему и условия эксперимента, чтобы другие учёные могли воспроизвести эксперимент и подтвердить заявленый результат.

Автор: Serg Ivanov 25.10.2010, 13:29

Цитата(Илья М @ 25.10.2010, 11:54) *
И что вы увидели там такого, что опровергает закон сохранения эненргии и второе начало термодинамики?

Кстати, видеозапись даже в суде не может быть доказательством чего-либо....(((

А для научной публикации требуется описать схему и условия эксперимента, чтобы другие учёные могли воспроизвести эксперимент и подтвердить заявленый результат.

Ничего. smile.gif Движение по ветру быстрее ветра не опровергает закон сохранения энергии и второе начало термодинамики. Опыт воспроизводился многократно. Учёные не возражают. wink.gif

Автор: Илья М 25.10.2010, 14:18

Цитата(Serg Ivanov @ 25.10.2010, 14:29) *
Ничего. smile.gif Движение по ветру быстрее ветра не опровергает закон сохранения энергии и второе начало термодинамики. Опыт воспроизводился многократно. Учёные не возражают. wink.gif


Ну, да...конечно...)))

Движение сквозь среду без затрат энергии не нарушает закон сохранения и второе начало термодинамики???
Вы в это верите только потому, что вам ОЧЕНЬ ХОЧЕТСЯ ВЕРИТЬ В СКАЗКУ!

А Учёные не возражают только потому,что их туда никто не звал!!!....(((

Автор: Serg Ivanov 25.10.2010, 15:32

Цитата(Илья М @ 25.10.2010, 14:18) *
Ну, да...конечно...)))

Движение сквозь среду без затрат энергии не нарушает закон сохранения и второе начало термодинамики???
Вы в это верите только потому, что вам ОЧЕНЬ ХОЧЕТСЯ ВЕРИТЬ В СКАЗКУ!

А Учёные не возражают только потому,что их туда никто не звал!!!....(((

Боюсь, что я не готов обсуждать Вашу картину мироздания... biggrin.gif

Автор: Vict 25.10.2010, 15:35

Цитата(Serg Ivanov @ 25.10.2010, 15:32) *
Боюсь, что я не готов обсуждать Вашу картину мироздания... biggrin.gif
а про яйцо и курицу? rolleyes.gif

Автор: Serg Ivanov 25.10.2010, 15:59

Цитата(Vict @ 25.10.2010, 15:35) *
а про яйцо и курицу? rolleyes.gif

А это где было?

Автор: Илья М 25.10.2010, 15:59

Тут где-то была ссылка на статью из "Популярной механики", так там в самом начале в качестве довода за эту "ПОПУТНУЮ" конструкцию был дан пример с движением тележки с вятряком против ветра.
Подразумевается, что если одно возможно, то и другое возможно....НО ЭТО НЕ ТАК!!!

Пример движения ветряка на тележке объясняется предельно просто:
Ветряк создаё сопротивление Р потоку ветра V, при этом вырабатывает мощность N=P*V (в идеале)
Ну, а для движения тележки против ветра с сопротивлением Р и МАЛОЙ скоростью V2<<V, нужна мизерная мощность N2=Р*V2
Вот и получаем, что ветряк вполне может двигать тележку против ветра много медленнее ветра, так как при этом вполне выполняется закон сохранения энергии.

А вот при попутном движении с ветром этот эффект не срабатывает, когда достигается переход от попутного движения в опережение ветра.

Сопротивление винта Р на скорость набегания V даст всегда меньшую мощность, чем это же сопротивление Р плюс сопротивление колёс и корпуса встречному ветру умноженные на ту же скорость V.
Так что энергетический баланс при попутном огоне ветра никак не сходится...

Автор: LordN 25.10.2010, 17:14

Цитата
Так что энергетический баланс при попутном огоне ветра никак не сходится
это слова. а где цифры?

Автор: HeatServ 25.10.2010, 17:50

Цитата(Илья М @ 25.10.2010, 15:18) *
Движение сквозь среду без затрат энергии не нарушает закон сохранения и второе начало термодинамики???

А если добавить, что это происходит на границе сред?

Автор: Serg Ivanov 25.10.2010, 20:51

Цитата(Илья М @ 25.10.2010, 15:59) *
Тут где-то была ссылка на статью из "Популярной механики", т

http://www.popmech.ru/article/7666-po-vetru-byistree-vetra/
Странно устроена человеческая логика. У подавляющего большинства людей она черно-белая: мы интуитивно делим все происходящее или воображаемое на логичное и нелогичное. Логичное, как мы считаем, реально. Нелогичное – абсурдно. В повседневной жизни этот алгоритм помогает нам выжить, но одновременно лишает наш разум гибкости. К счастью, для лечения негнущихся извилин имеется отличное средство. Это головоломки, ребусы и заковыристые тесты на сообразительность. По-видимому, американцы Рик Кавалларо и его приятель Джон Бортон получали интеллектуальные таблетки регулярно: ведь задачка, которую в 2004 году они опубликовали на форуме авиамоделистов, взбаламутила весь интернет. Рик и Джей Би предложили фанатам управляемого полета найти внятное научное объяснение феномену движения парусника или наземного аппарата на строго попутном ветре со скоростью быстрее ветра. А еще лучше – разработать конструкцию аппарата, который способен на это.

Оказывается, проблема движения ПВБВ («По ветру быстрее ветра») была успешно разрешена еще в 1960-х неким Эндрю Бауэром, инженером по аэродинамике летательных аппаратов в корпорации Douglas Aircraft. В 1969 году Бауэр поспорил со своим боссом, что построит машину, которая разгонится быстрее попутного ветра. Машина Бауэра, судя по единственному сохранившемуся снимку, была далека от совершенства, но ему удалось выиграть пари, показав пусть символический, но все-таки принципиальный результат – 1,2 скорости ветра.Впрочем, одеревеневшая логика скептиков сопротивлялась до последнего – бедного Бауэра, давно почившего в бозе, окрестили мистификатором, а его аппарат – неуклюжим муляжом, сделанным на живую нитку.
....................
В 2006 году, когда дискуссия стала переходить на уровень «сам дурак», на сцене появился некий Джек Гудман, бывший инженер и заядлый яхтсмен из Флориды. Быстро устав от бесполезной болтовни, Гудман взял да и построил радиоуправляемую модельку аппарата с тремя колесами и воздушным винтом. Прямо на улице, проходящей мимо дома Гудмана, она, толкаемая вперед попутным ветерком и тягой винта, наглядно продемонстрировала правоту Кавалларо и Бортона. Испытания были сняты на камеру, а клип выложен на YouTube.
...........................
Наиболее внятный анализ концепции движения ПВБВ был представлен профессором Массачусетского технологического института Марком Дрелой в начале прошлого года. Дрела скрупулезно рассчитал аэромеханику процесса и доказал, что в нем нет никаких противоречий с законами физики и тем более не наблюдается фантома вечного движения. Залог успеха – это эффективный пропеллер, простая трансмиссия с допустимым уровнем потерь и хороший трек с поверхностью типа крупной наждачки. Никаких аккумуляторов, никаких стартовых устройств – только аппарат и ветер, единственный источник энергии в системе.

Работает аппарат так: попутный ветер сдвигает машину с места, как предмет с необтекаемой формой, и колеса начинают свое вращение, раскручивая пропеллер. Тот, в свою очередь, создает тягу и толкает машину вперед. Из-за этого колеса начинают крутиться быстрее – следовательно, растут обороты пропеллера. Пока ветер дует – аппарат разгоняется. Заметьте, пропеллер никогда не крутит колеса и не вращается за счет энергии ветра напрямую.(С)

Автор: Vict 25.10.2010, 21:31

Цитата(Serg Ivanov @ 25.10.2010, 15:59) *
А это где было?
http://forum.abok.ru/index.php?showtopic=54035 smile.gif

Автор: Илья М 26.10.2010, 7:45

http://fregimus.livejournal.com/19427.html

Про движение против ветра было оказывается в ЖЖ, а не в Популярной механике...

Автор: Vict 21.2.2011, 11:09

только что просмотрел передачу "разрушители легенд". Сделали они эксперимент и с моделью, и с реальным самолетом - взлетает! smile.gif

Автор: Serg Ivanov 2.3.2011, 17:21

Водопад Эшера:
http://www.youtube.com/watch?feature=player_embedded&v=0v2xnl6LwJE

Автор: Медведица 7.2.2012, 16:14

физический аналог задачи: человек на коньках привязанный
к стене стоящий на той же ленте транспортёра
у взлетистов тот человек-самолёт движется и взлетает
но реально это не по правилам ибо верёвка нерастяжимая а самолёт может отодвигаться

Автор: Khomenko_A_S 31.5.2012, 15:49

ВЗЛЕТИТ!!!!!!!!!
Я оживил тему! ДАААА!!!!

Автор: Bb_Britva 26.6.2012, 12:26

Цитата(Bers @ 3.11.2006, 8:20) *
Ну дык что - Взлетит или не взлетит? Есть версии?


при достаточной длине полосы взлетит

Автор: Медведица 25.12.2012, 8:47

на ютюбе оказывается давно куча видео на эту тему

Автор: HeatServ 25.12.2012, 9:34

Только ни одно из них не решает условия задачи строго математически. Да, взлетает самолётик, но только если с рывком, т.е. условие задачи нарушается.
Там упомянут некий механизм в задаче, который скорость полотна синхронизирует, в реальности это воспроизвести невозможно.

Автор: Медведица 25.12.2012, 9:46

отнюдь такой механизьм вполне реален и заключается
в кабине пилота в виде настройки на режим когда
самолёт с включённым двигателем едет по ленте
и стоит на месте относительно неподвижной земли
ежели такой режим возможен то это и есть решение
то бишь НЕТ невзлетит а взлетает нарушая условия

Автор: Khomenko_A_S 26.12.2012, 0:37

Взлетает на видео и в теории. Даже не нарушая условий задачи. Как частный случай можно взять гидроплан: колеса не крутятся ввиду отсутствия, а он взлетает

Автор: HeatServ 26.12.2012, 15:43

Цитата(Khomenko_A_S @ 26.12.2012, 0:37) *
Взлетает на видео и в теории. Даже не нарушая условий задачи.
Покажите мне видео где в конструкции полотна есть "Оно (полтно) имеет систему управления, которая отслеживает и подстраивает скорость движения полотна таким образом, чтобы скорость вращения колес самолета была равна скорости движения полотна" и я соглашусь.

Автор: and 26.12.2012, 21:50

Цитата(HeatServ @ 26.12.2012, 16:43) *
Покажите мне видео где в конструкции полотна есть "Оно (полтно) имеет систему управления, которая отслеживает и подстраивает скорость движения полотна таким образом, чтобы скорость вращения колес самолета была равна скорости движения полотна" и я соглашусь.

Вопрос совсем не про систему управления полотна. Ну пусть будет такая (поскольку не указано, что эта система управления должна быть отдельно от самолёта, сделаем её на шасси самолёта):

Малое и большое колеса жёстко закреплены на одной оси.
Малое (несущее) едет по верхней ленте полотна (и приводит его в движение).
Большое едет по грунту аэродрома, регулируя скорость конвеера.
Скорость ленты конвеера относительно грунта в данном случае всегда будет равна скорости вращения несущего колеса (скорости нижней или верхней точки поверхности колеса относительно оси колеса) и всегда будет равна половине скорости самолёта относительно грунта.

Автор: HeatServ 26.12.2012, 22:20

Цитата(and @ 26.12.2012, 21:50) *
Вопрос совсем не про систему управления полотна. Ну пусть будет такая (поскольку не указано, что эта система управления должна быть отдельно от самолёта, сделаем её на шасси самолёта):
Взлёт самолёта обеспечивают не колёса, но винты или турбины, т.е. это не колёсный привод, поскольку колёса самолёта пассивны. А система, описанная в условии задачи в реальности невозможна.
Это и есть невыполнимое условие, которое возвышает задачу в область холивара, бессмысленного и беспощадного.

Есть упрощённая задача - привязать самолёт к столбу и, разгоняя винт, поднять самолёт в воздух. Задача решаема численно. А бегущее полотно с системой - холивар.

Автор: Iroha 26.12.2012, 22:29

Я вот думаю: а утка взлетит с включенной беговой дорожки ?

Автор: HeatServ 26.12.2012, 23:05

Цитата(Iroha @ 26.12.2012, 22:29) *
Я вот думаю: а утка взлетит с включенной беговой дорожки ?
Для утки не нужен разбег. Только самолёту нужен разбег.

Автор: Iroha 26.12.2012, 23:09

они же при взлете с воды лапками помогают

Автор: HeatServ 26.12.2012, 23:13

Цитата(Iroha @ 26.12.2012, 23:09) *
они же при взлете с воды лапками помогают
Это просто проще - и без лап они взлетели бы без проблем. У них сильные крылья и довольно крутые углы подъёма. Стрелял, знаю.

Автор: Iroha 26.12.2012, 23:22

Ну ладно. Когда интересно махолеты появятся.

Автор: and 27.12.2012, 1:53

Цитата(HeatServ @ 26.12.2012, 23:20) *
Взлёт самолёта обеспечивают не колёса, но винты или турбины, т.е. это не колёсный привод, поскольку колёса самолёта пассивны. А система, описанная в условии задачи в реальности невозможна.
Это и есть невыполнимое условие, которое возвышает задачу в область холивара, бессмысленного и беспощадного.

Есть упрощённая задача - привязать самолёт к столбу и, разгоняя винт, поднять самолёт в воздух. Задача решаема численно. А бегущее полотно с системой - холивар.

1. Знаю, что в холивар лучше просто не влазить, но тянет погрешить...
2. Заинтересовало Ваше описание системы управления, с намёком на невозможность её реализации, почему и показал простейшее решение.
Только потом, прочитав первый топик, увидел, что в изначальном условии указано направление движения полотна как противоположное направлению взлёта (у Вас в топике про видео это не было указано).
3. Задача не требует строить систему управления движением покрытия, она дана в условии как уже существующая.
Мы же как-то получили среднее образование, не споря с преподавателем о невозможности решения задачи каждый раз, когда груз висел на невесомой нерастяжимой нити, или фигурировало абсолютно упругое или абсолютно чёрное тело?
И здесь достаточно указать, что колёсное шасси механически развязывает самолёт с покрытием, и любое движение последнего вдоль направления взлёта не оказывает влияния на решение задачи. Нет здесь никакой нерешаемости.

Автор: and 27.12.2012, 2:03

Предлагаю другую задачу, на мой взгляд более интересную:
Имеется большое и ровное лётное поле, и стоит безветренная погода. В центре поля стоит самолёт.
В каком направлении относительно сторон света (С, Ю, В, З) должен взлетать лётчик, чтобы взлёт самолёта (от старта до отрыва) занял наименьшее время.

Автор: Khomenko_A_S 28.12.2012, 13:03

Намек на взлет на запад и перемещение в другой "секундный пояс".
Или в направлении перпендикулярном сторонам света- вверх
Есть еще варианты с северным и южным полюсами

Автор: Khomenko_A_S 28.12.2012, 13:18

Цитата(HeatServ @ 26.12.2012, 14:43) *
Покажите мне видео где в конструкции полотна есть "Оно (полтно) имеет систему управления, которая отслеживает и подстраивает скорость движения полотна таким образом, чтобы скорость вращения колес самолета была равна скорости движения полотна" и я соглашусь.

Для этого его нужно снять.
Мой сценарий:
1.Берем перед новым годом фейерверк (побольше)
2.Приклеиваем скотчем колесики от игрушки и крылья картонные (колесики -обязательно жестко, скорость вращения =0)
3.Сооружаем что угодно, что можно назвать конвеером и обязательно заклиниваем (скорость вращения =0)
4.Поджигаем фитиль


Согласно условиям задачи:
Самолет может быть с реактивной тягой.
Двигатель может быть бесконечно мощным.
Колеса могут проскальзывать па поверхности

Автор: and 29.12.2012, 22:24

Цитата(Khomenko_A_S @ 28.12.2012, 14:03) *
Намек на взлет на запад и перемещение в другой "секундный пояс".
Или в направлении перпендикулярном сторонам света- вверх
Есть еще варианты с северным и южным полюсами

Никаких "медведьевских" шуток со временем. Начался разгон - запустили секундомер, оторвались шасси - остановили секундомер.
Время взлёта объективно зависит от выбранного направления взлёта по известным со школы законам физики. Никакой мистики или подвохов.
Примечание/подсказка:
Если разместить аэродром на северном или южном полюсе, то указанный эффект наблюдаться не будет (т.е. для всех направлений будет одинаковое время взлёта).

Автор: Khomenko_A_S 30.12.2012, 9:21

Сила Кариолиса? Да ну ладно. Сомневаюсь что она окажет сколь нибудь существенное влияние

Автор: Медведица 30.12.2012, 11:18

"В каком направлении относительно сторон света
(С, Ю, В, З) должен взлетать лётчик, чтобы
взлёт самолёта (от старта до отрыва) занял наименьшее время"
конец цытаты
ответ: конечно на ЗАПАД: там больше платят

Автор: and 30.12.2012, 12:23

Цитата(Khomenko_A_S @ 30.12.2012, 10:21) *
Сила Кариолиса? Да ну ладно. Сомневаюсь что она окажет сколь нибудь существенное влияние

Сила Кариолиса ни причём, но направление мысли на анализ малых сил верное.
Для задачи достаточно чтобы для определённого направления взлёт происходил быстрее, на сколько быстрее - неважно.

Автор: and 24.6.2013, 14:22

Поскольку народ стесняется решать, сообщаю правильный ответ.
Вес любого тела на поверхности вращающейся Земли определяется как разность силы тяжести и силы инерции вращательного движения (в просторечии центробежной силы).
Если объект перемещается горизонтально по поверхности Земли с запада на восток, то его собственная скорость складывается со скоростью вращения Земли, центробежная сила возрастает, вес снижается. Если объект перемещается с востока на запад, то его собственная скорость вычитается из скорости вращения Земли, центробежная сила снижается, вес возрастает.
В задаче лётчик взлетит быстрее, если начнёт взлёт в направлении востока.
В практике эффект используется например для доставки груза на более высокие орбиты тем же ракетоносителем при выборе орбиты с ориентацией запад-восток. Т.е. эффект не так уж мал.

Автор: nik4t 17.9.2013, 2:37

Видео к задаче никакого отношения не имеет. Просто мировой воздушный трафик. Глядя на это, столкновение двух самолётов в воздухе не кажется не возможным...
http://masterok.livejournal.com/672797.html
В комментах прога чумовая!))

Автор: and 17.9.2013, 9:59

Видео наглядно показывает размещение цивилизации на Земле, как и http://ttolk.ru/wp-content/uploads/2012/12/%D0%BA%D0%B0%D1%80%D1%82%D0%B0-%D0%B3%D0%BB.jpg.
Прога правда классная.

Автор: stasngs 8.10.2013, 11:07

Элементарно, скорость вращения колес не влияет на подъемную силу воздуха. А так спасибо за подъем настроения (представил себе самолет перебирающий колесами в воздухе), а так если он реактивный либо винтовой, при работающем двигателе взлетит

Автор: HasBolla 29.11.2013, 13:20

Задача для разминки мозга в пятницу!)

Итак- условие: на рычажных весах стоят 2 колбы с водой. В левой колбе на нитке ко дну привязан шарик для пинг-понга, в правую колбу опущен металлический шар (такого же объёма как и левый) на леске.

А теперь вопрос- в какую сторону отклонятся весы???


Автор: Khomenko_A_S 29.11.2013, 13:23

Цитата(HasBolla @ 29.11.2013, 12:20) *
Задача для разминки мозга в пятницу!)

Итак- условие: на рычажных весах стоят 2 колбы с водой. В левой колбе на нитке ко дну привязан шарик для пинг-понга, в правую колбу опущен металлический шар (такого же объёма как и левый) на леске.

А теперь вопрос- в какую сторону отклонятся весы???


Там где мячик. Колба весит больше на вес мячика и нити

Автор: Андрей 113 29.11.2013, 13:24

Цитата(HasBolla @ 29.11.2013, 14:20) *
Задача для разминки мозга в пятницу!)

Итак- условие: на рычажных весах стоят 2 колбы с водой. В левой колбе на нитке ко дну привязан шарик для пинг-понга, в правую колбу опущен металлический шар (такого же объёма как и левый) на леске.

А теперь вопрос- в какую сторону отклонятся весы???


в сторону где шарик для пинг-понга. ...наверно

Автор: CNFHSQ 29.11.2013, 19:33

вес нити и шарика думаю не учитывается.http://images.yandex.ru/yandsearch?source=20%D0%BA%D0%BE%D0%BB%D0%B1%D0%B5psearch&text=%D0%BD%D0%B0%20%D1%80%D1%8B%D1%87%D0%B0%D0%B6%D0%BD%D1%8B%D1%85%20%D0%B2%D0%B5%D1%81%D0%B0%D1%85%20%D1%81%D1%82%D0%BE%D1%8F%D1%82%202%20%D0%BA%D0%BE%D0%BB%D0%B1%D1%8B%20%D1%81%20%D0%B2%D0%BE%D0%B4%D0%BE%D0%B9.%20%D0%92%20%D0%BB%D0%B5%D0%B2%D0%BE%D0%B9%%20%D0%BD%D0%B0%20%D0%BD%D0%B8%D1%82%D0%BA%D0%B5%20%D0%BA%D0%BE%20%D0%B4%D0%BD%D1%83%20%D0%BF%D1%80

Мячик поднимется.

Автор: HeatServ 29.11.2013, 19:34

Цитата(HasBolla @ 29.11.2013, 13:20) *
Задача для разминки мозга в пятницу!)

Итак- условие: на рычажных весах стоят 2 колбы с водой. В левой колбе на нитке ко дну привязан шарик для пинг-понга, в правую колбу опущен металлический шар (такого же объёма как и левый) на леске.

А теперь вопрос- в какую сторону отклонятся весы???

Задача для шестого класса советской школы. А что за источник?

Автор: AVFRZN 29.11.2013, 20:10

Некорректно поставлена задача.
Весы в каком состоянии находились до опускания шара?

Автор: BROMBA 30.11.2013, 11:44

Цитата(HeatServ @ 29.11.2013, 19:34) *
Задача для шестого класса советской школы. А что за источник?

http://www.urod.ru/ 29 ноября Задача про 2 колбы )))

Автор: and 30.11.2013, 13:31

Исходя из условий задачи можно предположить, что до помещения в колбы (хотя это не колбы) шаров весы находились в равновесии и колбы содержали равное количество воды.

Автор: and 30.11.2013, 13:53

Помещение в колбы шаров увеличило вес каждой колбы на величину архимедовой силы (она равна, т.к. диаметры шаров равны).
Но левая чаша также стала легче на величину натяжения нити (архимедова сила минус масса теннисного мячика).
Т.к. натяжение левой нити положительно, правая чаша перевесит.

Автор: Бойко 30.11.2013, 16:37

Цитата(and @ 30.11.2013, 13:53) *
Помещение в колбы шаров увеличило вес каждой колбы на величину архимедовой силы (она равна, т.к. диаметры шаров равны).
Но левая чаша также стала легче на величину натяжения нити (архимедова сила минус масса теннисного мячика).
Т.к. натяжение левой нити положительно, правая чаша перевесит.


rolleyes.gif Т.е. возможна ситуация когда колба с мячиком взлетит?
Надо последовательно проставить все силы. Написать уравнение. И тогда окажется, что левая колба стала больше давить на подставку. Значение? =Вес стенок мячика/шарика + вес воздуха в нем.

Автор: nik4t 30.11.2013, 19:20

Цитата(Бойко @ 30.11.2013, 17:37) *
rolleyes.gif Т.е. возможна ситуация когда колба с мячиком взлетит?


Автор: Kult_Ra 30.11.2013, 20:56

http://www.youtube.com/embed/xApMwuzf2jE?hl=ru_RU


Автор: and 1.12.2013, 8:41

Уравнение не писал. Первым решением было:
Левая сторона потяжелела на вес тенн.м.
Правая сторона потяжелела на вес ст. м, но полегчала на натяжение нити (вес ст. м минус архимедова сила).
Итого:
Левая сторона + вес тенн.м, Правая сторона + архим.сила. Т.к. арх.сила > вес тенн.м. весы перевесят вправо.
Но у меня была разница (арх.сила минус вес тенн.м), а у Бойко разница просто вес тенн.м.
Но в это решение не поверил за легкостью, нашёл другое.
А как Вы предположили полёт колбы при моём первом решении?

Автор: BROMBA 1.12.2013, 13:54

Цитата(and @ 1.12.2013, 8:41) *
А как Вы предположили полёт колбы при моём первом решении?

Элементарно: подходишь, правой рукой хватаешь колбу, размахиваешься от плеча... летит? летит! прямо в форточку...

Когда-то по молодости, забавлялись гидростатическим взвешиванием на аналитических весах...
А как иначе определить удельную полтность корковой зоны слитка кипящей стали, не имея под рукой шведского ртутного порозиметра? Холодно было... при +40 развлекаться приходилось, шоб расчеты чутка упростить, формулка-то в полстраницы, однако...
Как это делается - Гугл в помощь, расчетные формулы и статистическая обработка результатов - там же.
Почитайте, и решение вот оно, на ладошке...

Автор: Бойко 1.12.2013, 14:36

Цитата(and @ 1.12.2013, 8:41) *
....
А как Вы предположили полёт колбы при моём первом решении?


Недавно пришлось долго объяснять про инерцоиды..
Безопорный движетель (2.12 мин)
http://www.youtube.com/watch?v=vk9qjhLdnig
подходы схожие.

Автор: BROMBA 1.12.2013, 15:17

Цитата(Бойко @ 1.12.2013, 15:36) *
Безопорный движетель (2.12 мин)

Угу, безопорный... Только вот упругая балочка какбэ не при делах )))))
Вот если бы вибратор на соплях держался, тогда да, безопорное...

Автор: Бойко 1.12.2013, 16:06

Цитата(BROMBA @ 1.12.2013, 15:17) *
.........
Вот если бы вибратор на соплях держался, тогда да, безопорное...


На сопле это дешево. В РФ (2008 год) инерцоид (орбиту спутника корректировать) запускали в космос.
Во как mad.gif . Сопля...

Автор: BROMBA 2.12.2013, 13:57

Ну шо, настало время ответов? )))


Автор: BROMBA 2.12.2013, 14:06

Цитата(Бойко @ 1.12.2013, 17:06) *
На сопле это дешево. В РФ (2008 год) инерцоид (орбиту спутника корректировать) запускали в космос.
Во как mad.gif . Сопля...

Злитесь, не злитесь, инерциоиды - детские игрушки...
Электрогравитационный двигатель - это посерьёзнее...
Опыт с рентгеновской трубкой без накала и вакуумные конденсаторы...

Автор: Iroha 2.12.2013, 14:14

Гравицапа инерциоид не сдвинул орбиту спутника ни на микрон.

Автор: BROMBA 2.12.2013, 14:17

Ну и чисто поржать:

Цитата
Самодельная рентгеновская трубка



В 1955-1960 годах в различных детских технических изданиях предлагали провести серию любопытных и занимательных опытов с .... (!!!) рентгеновской трубкой - изготовить которую оказывается легко может любой человек ...
ДЛЯ ЧЕГО ЭТО НУЖНО?
Для опытов по просвечиванию изучаемых предметов, для авиа или судомодельных технологий (когда нужно заглянуть внутрь закрытого объема) и даже для просвечивания костей ...(!!! опасно) собственной руки еще потребуется обычная фотобумага в черном пакетике или школьный плоский флюоресцентный экран.

Но мы то сегодня знаем, что еще проще можно эти опыты проделать с обыкновенной ... веб-камерой или старым ненужным цифровым фотоаппаратом - получив рентгеновский сканер реального времени.

Стоит ли доводить такую информацию до своих младших братиков или деток - пусть решает каждый сам.
Лично мы (представители нового общества) считаем, что стоит и нужно. Пусть дети знают все без секретов и тайн. Просто нужно вразумительно объяснить им последствия и опасность.
А такие опыты вполне можно осторожно проделать вместе с ними - ВЕДЬ МОЩНОСТЬ САМОДЕЛЬНОЙ Р-ТРУБКИ НЕ ВЕЛИКА...
Как применять этот девайс решайте сами. Сейчас не 1955 год - всякой информации о применении рентгена навалом.
КАК СДЕЛАТЬ:
Трубку изготовим из обычной лампочки накаливания (обязательно вакуумной). Подойдет даже лампочка от карманного фонаря и даже крохотная от подсветка приборов. К слову - на крохотных лампочках можно сделать малюсенькие источники рентгена. Только нужно заряжать лейденские банки (высоковольтный конденсатор) от пьезоэлемента газовой зажигалки.
И так что делаем:
1. Берем лампочку, обезжириваем ацетоном и спиртом и аккуратно без пузырей наклеиваем на нее тонкую алюминиевую фольгу. ПРАВДА так делали в 1955 году. А сейчас мы идем в магазин авто-мото или там где продают радиолюбительские прибамбасы и покупаем пузырек электропроводящего клея (или лака).
2. Берем кисточку и мажем лампочку так как мы могли наклеить фольгу...

3. ВСЕ! Девайс готов. Теперь нам потребуется любой источник высокого напряжения (5000 -10000 вольт) .

ГДЕ ВЗЯТЬ ИСТОЧНИК 5000 вольт?
Ток большой не требуется, поэтому подойдет все что не попадя: школьная электрофорная машина, блок питания от небольшого старого телевизора (не с плоским экраном а с кинескопом), даже электрошокер с высоковольтным кремневым выпрямителем или даже бытовая газовая зажигалка с лейденской банкой (на схеме это конденсатор).
Короче подключаем ток, даем накал и... имеем счастье (или проблемы).


Нуууу... Готовы с полным основанием говорить: "Да я тебя, сцюко, насквозь вижу!!!" ???)))))

Автор: Iroha 2.12.2013, 14:24

Цитата(BROMBA @ 2.12.2013, 17:17) *

5000 В не маловато ?

Автор: Бойко 2.12.2013, 18:49

Цитата(Iroha @ 2.12.2013, 14:14) *
Гравицапа инерциоид не сдвинул орбиту спутника ни на микрон.


В официальном отчете... Тяга=28 Г mad.gif

Автор: Iroha 2.12.2013, 19:15

Цитата(Бойко @ 2.12.2013, 21:49) *
В официальном отчете... Тяга=28 Г mad.gif

Академик Евгений Александров, член Комиссии по борьбе с лженаукой и фальсификацией научных исследований РАН:
”Действительно, попытки реализовать движитель без реакции опоры или отбрасываемой массы не нарушают закон сохранения энергии, зато нарушается столь же фундаментальный закон сохранения импульса.
.........

Автор: BROMBA 2.12.2013, 22:16

Цитата(Iroha @ 2.12.2013, 14:24) *
5000 В не маловато ?

А ренген при торможении электронов об стекло не слишком "мягкий"? )))

Автор: Akela 3.12.2013, 11:34

Цитата(BROMBA @ 2.12.2013, 14:57) *
Ну шо, настало время ответов? )))


ошибка экспериментатора на видео в том, что у него изначально был одинаковый уровень воды в колбах. И после погружения металлического шара в воду уровень в правой колбе стал выше - следовательно она стала тяжелее. Чтобы соблюсти условия изначальной задачи, следовало после погружения гири удалить из этой колбы количество воды, равное ее объему (чтобы уровень в колбах стал одинаковым).

На этом принципе работают судоподъемники - вес секции с кораблем и без него одинаков, если одинаков уровень воды.

 

Автор: nik4t 3.12.2013, 12:23

Цитата(BROMBA @ 2.12.2013, 14:57) *
Ну шо, настало время ответов? )))

Коряво как-то... не по-нашенски, да и опыт с задачей разнИтся. Не ответ, в общем.
Вот ответ на задачу.
Цитата(Kult_Ra @ 30.11.2013, 21:56) *
http://www.youtube.com/embed/xApMwuzf2jE?hl=ru_RU

_____________
and прав.

Автор: Бойко 3.12.2013, 16:30

Цитата(nik4t @ 3.12.2013, 12:23) *
....
and прав.


Внутреннее убеждение? rolleyes.gif
Опыты? Оба с грубыми ошибками.

Лишний раз можно убедиться..
Есть теоретическая физика.
Есть и теоия эксперимента.
Важно владеть или понимать, что там имеются знания не передаваемые нам жизненным опытом
Получается... мы отказываемся от теории/выводить аналитическое уравнение и основываемся на интуиции.
Мы отказываемся от теории эксперимента и метрологии...
Так рождаются гравицапы инерцоиды...

Очень важное наблюдение для наших "энергоаудиторов"

Автор: nik4t 3.12.2013, 22:29

Цитата(Бойко @ 3.12.2013, 17:30) *
Внутреннее убеждение? rolleyes.gif
Нет.
Школьный курс физики, который ( к своему стыду) и я забыл.
http://www.autosaratov.ru/phorum/threads/278937-Небольшой-пятничный-вынос-мозга-задачка-по-физике/page2
Главное в это задаче помнить:
1. Что вес это сила.
2. Что закон Архимеда про силу.

Автор: HeatServ 3.12.2013, 22:33

Бойко, что, серьёзно поверили в тот видос от узкоглазых?

Автор: Бойко 3.12.2013, 22:45

Цитата(nik4t @ 3.12.2013, 22:29) *
Нет.
Школьный курс физики, который ( к своему стыду) и я забыл.
http://www.autosaratov.ru/phorum/threads/278937-Небольшой-пятничный-вынос-мозга-задачка-по-физике/page2
Главное в это задаче помнить:
1. Что вес это сила.
2. Что закон Архимеда про силу.


Надо просто пытаться сделать. Писать уравнение/не бояться ошибок/читать книгу/исправлять и опять писать.

Притча
"Однажды Учитель спросил учеников:

— На бревне сидели три лягушки. Одна из них решила прыгнуть в воду. Сколько лягушек осталось на бревне?

— Три… — неуверенно ответил один из них.

— Конечно же, три, — улыбнулся Учитель. — Поскольку лягушка только решила прыгнуть, но не предприняла для этого никаких действий. Никогда не путайте действие с принятием решения. Иногда вам кажется, что вы уже прыгнули, но на самом деле вы по-прежнему сидите на бревне. Причём, в окружении таких же, как и вы сами..."

Автор: nik4t 3.12.2013, 22:48

-Следующий!
biggrin.gif

Автор: Бойко 3.12.2013, 22:56

Цитата(HeatServ @ 3.12.2013, 22:33) *
Бойко, что, серьёзно поверили в тот видос от узкоглазых?


Обижаете.
У Ра интересней, но тоже ошибок дофига.
На этом примере можно изучать важность методики измерения.
Например. Вес шарика для пин понга 2,7 грамма. Заявленная/декларированная точность редких бытовых весов (шкала 5 кГ) составляет 2-4 Г. А в действительности...? И китайских?.
Ну и так далее...
Так зак. тыча пальцем в термоманометр на вводе, вопит, что "напора нет".

Автор: HeatServ 3.12.2013, 22:59

Я однажды учился на физмате, у нас там подобные задачи были, но, Ъ, их любили поразвить. Например, в левой чаше оказывалась ртуть, высота чаш была десять метров, а погруженный на дно правой чаши (заполненной конечно же глицерином, как, ъ, без глицерина) был опущен шар радиуса R, и потом надо было найти ускорение подъёма шара из глицерина, чтобы на выходе из него чаши весов были уравновешены. Вот это на полдоски. А были и на три доски задачки...

Автор: K_a_t_r_i_n 24.12.2013, 8:51

Вот задачка встретилась . См рис.
велосипед стоит и упасть на бок условно не может. колеса не прокручиваются по поверхности. тянем за веревку привязанную к колесу - что произойдет?
1. поедет вперед
2. останется стоять
3. поедет назад
Чур в яндексе не подсматривать . И сразу ответ не обосновывать =)
сорри если задачка уже была на данном форуме
На том форуме, где я это стащила, люди дошли до натурных экспериментов, laugh.gif опытным путем доказали правильный вариант и все равно оставались люди утверждающие что всё не так laugh.gif

 

Автор: HeatServ 24.12.2013, 8:55

1, а не, колёса не прокручиваются, значит назад поедет... или стоять останется... а что значит колёса не прокручиваются? Приколочены или типа как звёздочка на цепи? Если как звёздочка на цепи, в зацеплении т.е., то останется стоять на месте.

Автор: K_a_t_r_i_n 24.12.2013, 9:04

Колеса не прокручиваются значит что коэффициент трения покрышки колеса и поверхности максимален - как бы сильно не тянули веревку колесо может КАТИТЬСЯ туда или сюда но не ПРОСКАЛЬЗЫВАТЬ по пов-ти .
просьба сразу не обосновывать свои варианты - дать поломать мозг товарищу =)

Автор: kvm_auditor 24.12.2013, 9:41

Ну как то так
http://www.youtube.com/watch?v=fx0WhwsrJk8

Цитата(K_a_t_r_i_n @ 24.12.2013, 11:51) *
На том форуме, где я это стащила, люди дошли до натурных экспериментов

Вот из натурных эксперементов:
Цитата
Только что пришёл. С соседом взяли четыре литра пива и тоже провели эксперимент.

привязали к колесу велосипеда веревку и слегка потянули за нее

Взял с машины буксирный трос. Слегка тянуть не получилось, стали сильнее, колесо проскользнуло.

Давайте считать, что он не упадет и что его колеса не будут проскальзывать

Чтобы исключить эффект проскальзывания, нагрузили велосипед, а именно на руль навешали всякого хлама и ещё сосед сел с верху. В итоге вырвал одну спицу и две согнул. Решили оставить дальнейшее проведение эксперимента до завтра, так как кончилось пиво.



Автор: Машинист 24.12.2013, 9:54

всё правильно, таким макаром только спицы и вырывать. На месте останется наш велосипед. Колесо заднее мысленно бетоном залейте, для выполнения условий "не упадёт" и "колесо не проскальзывает", ну и для наглядности просто. rolleyes.gif

Автор: HeatServ 24.12.2013, 10:08

Если заднее колесо изобразить в виде звёздочки, которая размещена на жёстко прицепленной развёрнутой на полу цепи, то по идее колесо поедет куда его потянули, потому что возникнет крутящий момент. Но в первую же долю секунды условие задачи будет нарушено, поскольку верёвка уйдёт от горизонтали. Т.е. в идеале правильный ответ - покатится назад.

Автор: and 24.12.2013, 10:24

Куда тянут, туда и поедет

Автор: Машинист 24.12.2013, 11:19

Цитата
Куда тянут, туда и поедет

Поедет назад, если веревку немного повыше перевязать. А насколько выше? wink.gif


А в нашем случае куда поедет? Получается, вперёд? blink.gif
Ан нет, вперед тоже не поедет!
Правы были мужики, которые спицы выломали.

Автор: HeatServ 24.12.2013, 11:36

Цитата(Машинист @ 24.12.2013, 11:19) *
Поедет назад, если веревку немного повыше перевязать. А насколько выше? wink.gif


А в нашем случае куда поедет? Получается, вперёд blink.gif
В любом случае поедет, не поедет если только привязать верёвку к точке зацепления с поверхностью, которая по условию задачи у нас без проскальзывания.

Автор: Машинист 24.12.2013, 11:45

Цитата(HeatServ @ 24.12.2013, 12:36) *
В любом случае поедет, не поедет если только привязать верёвку к точке зацепления с поверхностью, которая по условию задачи у нас без проскальзывания.

Точно! Мало кругов нарисовал.
Из любой точки поедет, кроме нижней правой четверти ? rolleyes.gif

Автор: and 24.12.2013, 13:03

Цитата(Машинист @ 24.12.2013, 12:45) *
Из любой точки поедет, кроме нижней правой четверти ? rolleyes.gif

Не поедет, только если вектор силы тяги проходит через точку опоры (Пара сил на одной оси не создадут вращ. момента).
Если вектор тяги проходит выше точки опоры (как сейчас) поворот по часовой.
Если вектор тяги пройдёт ниже точки опоры, поворот против часовой.
Для простоты понимания мысленно уберите велосипед и свободное колесо, и оставьте от привязанного колеса только фрагмент обода от точки опоры (на грунт) до точки привязки. Рассмотрите первое движение оставшегося бруска при натяжении верёвки.

Автор: HeatServ 24.12.2013, 13:40

Цитата(Машинист @ 24.12.2013, 11:45) *
Точно! Мало кругов нарисовал.
Из любой точки поедет, кроме нижней правой четверти ? rolleyes.gif
Поедет абсолютно из любой точки, кроме точки касания (без трения которая), у колеса же есть поступательное ещё движение. Как у паровозов ведь принцип-то.

Автор: K_a_t_r_i_n 24.12.2013, 14:19

Попробую Вас вывести к свету ну или к тьме =)
смотрим на картинку : нас интересует по сути из всего колеса три точки: втулка , точка опоры колеса о поверхность и точка за которую тянем. соединены эти три точки спицами и ободом и получается треугольник . и с одной стороны все логично - тянем и вся конструкция движется в сторону куда тянем. но линия втулки опускается ниже самой себя - колесо зарывается под землю ? biggrin.gif а чтобы оно не зарывалось - только волочить этот "треугольник" волоком ? Нарушаем условие задачи? Где подвох

 

Автор: HeatServ 24.12.2013, 14:23

Цитата(K_a_t_r_i_n @ 24.12.2013, 14:19) *
Попробую Вас вывести к свету ну или к тьме =)
смотрим на картинку : нас интересует по сути из всего колеса три точки: втулка , точка опоры колеса о поверхность и точка за которую тянем. соединены эти три точки спицами и ободом и получается треугольник . и с одной стороны все логично - тянем и вся конструкция движется в сторону куда тянем. но линия втулки опускается ниже самой себя - колесо зарывается под землю ? biggrin.gif а чтобы оно не зарывалось - только волочить этот "треугольник" волоком ? Нарушаем условие задачи? Где подвох
Точка зацепления верёвки на колесе движется по циклоиде, в этом и подвох.


Автор: and 24.12.2013, 14:31

Это не подвох, а обоснование.
Вот если бы колесо было с ободом, а верёвку привязать к наружной точке обода, то циклоида была бы такая (см. рис.2):

И при нахождении точки крепления верёвки ниже горизонта было бы "обратное" вращение колеса, а выше - прямое (куда тянем, туда едет).
Т.е. с ж/д колёсной парой вопрос был бы интереснее.

Автор: and 4.9.2015, 21:49

https://www.youtube.com/watch?v=VxvJcPToXb0
Физика процесса?

Автор: Kagamine Len 4.9.2015, 22:42

Не взлетит - нет подъемной силы на крыло. Несложный вопрос.

Автор: Vano 4.9.2015, 22:45

Луганские пацаны - я смотрю на на них и думаю, ну хорошо живы.
https://www.youtube.com/user/kreosan

Автор: Kagamine Len 4.9.2015, 22:53

Пацаны сильно рискуют, свч излучение вызывает помутнение хрусталика глаза. Бывали случаи заглядывания в открытый волновод свч передатчика, заканчивались в великой печали.

Автор: дренаж 4.9.2015, 22:57

решение задачи в возможности самолёта с включенным двигателем

стоять на месте относительно земли при движущейся навстречу ленте

здесь и будет случай соответствующий условию задачи

а у взлетистов самолёт "взлетает" всего лишь нарушив условия задачи

Автор: and 5.9.2015, 11:05

Дался вам этот самолёт.
Я тему поднял по поводу ракеты на пропано-кока-кольном топливе. https://www.youtube.com/watch?v=VxvJcPToXb0 же есть.
Как работает топливо? Что его инициирует?

Автор: Kagamine Len 5.9.2015, 11:58

Пропан расширяется после нагрева от жидкости.

Автор: and 5.9.2015, 15:04

Пока бутылку не перевернут, он как-то не особо торопится расширяться...

Автор: Kagamine Len 5.9.2015, 20:55

Цитата(and @ 5.9.2015, 16:04) *
Пока бутылку не перевернут, он как-то не особо торопится расширяться...

оно и так расширяется, только , когда переворачивают, начинает расширятся в замкнутом объеме, выталкивая жидкость

Автор: and 6.9.2015, 10:20

Повторил эксперимент с бутылкой. Никакого эффекта. Вообще.

Автор: LordN 6.9.2015, 10:38

посмотрите видос разрушителей с бутылками с водой и сжатым воздухом.
берем бутыль, наливаем туда треть-четверть воды, переворачиваем на плотную конусную пробку со штуцером подключённым к воздуху высокого давления. открываем подачу воздуха. воздух попадает в пространство меж водой и бутыллкой. толкает воду - получается приличная реактивная тяга.
я года три тому испытывал такое. летает smile.gif

Автор: and 6.9.2015, 22:46

Раньше игрушка была такая - ракета. Заполнялась водой (не полностью), и накачивалась велосипедным насосом. Летала конкретно и опасно. Корпус алюминиевый, с резиновым наконечником. Но если в человека попасть, наконечник не шибко поможет. Удивлялся, как такое допустили к продаже, тем более как игрушку.
Но в фокусе с бутылкой нет предварительно созданного давления. На видео переворачивают бутылку без крышки.

Автор: Kagamine Len 8.9.2015, 23:13

Пропан расширяется, больше там нечему расширяться.

Автор: HeatServ 9.9.2015, 8:59

Цитата(Kagamine Len @ 8.9.2015, 23:13) *
Пропан расширяется, больше там нечему расширяться.
Пропан служит катализатором для выделения углекислоты, которой в коле полстакана, летает бутылка не на пропане, а на углекислоте.

Автор: HeatServ 9.9.2015, 9:24

Существуют способы беспламенного взрывания у горняков.
Ка́рдокс — способ беспламенного взрывания, основанный на переходе жидкой углекислоты, заключенной в стальном патроне (который также называется кардоксом), в газообразное состояние
Тут тоже что-то подобное происходит за счёт пропана.

Автор: Kagamine Len 9.9.2015, 14:17

Цитата(HeatServ @ 9.9.2015, 9:59) *
Пропан служит катализатором для выделения углекислоты, которой в коле полстакана, летает бутылка не на пропане, а на углекислоте.

А как процесс происходит , есть где почитать ?

Автор: HeatServ 9.9.2015, 14:27

Цитата(Kagamine Len @ 9.9.2015, 14:17) *
А как процесс происходит , есть где почитать ?
Где читать? Да не знаю, знаю только, что для беспламенного взрыва используют резкий нагрев, тут скорее всего тоже нужно подогреть. Газ, когда его наливают в бутылку, имеет низкую температуру, когда происходит взбалтывание, то происходит его нагрев, видимо он же подогревает углекислоту, может ещё и связывает её каким макаром.
Сложно сказать... ведь если просто в бутылку с водой залить газа, то такого эффекта не будет?

Автор: Kagamine Len 9.9.2015, 16:51

Не не расширение, высвобождение углекислого газа почти как взрыв , может химическая реакция присутствует, в коле не только ведь вода, есть ортофосфорная кислота.

Автор: HeatServ 9.9.2015, 17:20

Цитата(Kagamine Len @ 9.9.2015, 16:51) *
Не не расширение, высвобождение углекислого газа почти как взрыв , может химическая реакция присутствует, в коле не только ведь вода, есть ортофосфорная кислота.
Достаточно быстрый переход из жидкой фазы в газообразную. Не взрыв.

Автор: Kagamine Len 9.9.2015, 17:25

Взрывообразно наверно точнее будет.
http://www.youtube.com/watch?v=33vkelhr18Y
в детстве карбидом баловались. самой большой снаряд был ведро. красиво летало.

Автор: HeatServ 9.9.2015, 19:28

Короче, моё видение летающей колы такое: пропан, который выпускают из баллона, имеет низкую температуру, при взбалтывании и перемешивании с колой, имеющей более высокую температуру, он резко расширяется, при этом выделяя достаточное количество теплоты для перехода углекислого газа из состава колы в газообразное состояние, а для этого нужно очень незначительная энергия, например, достаточно чуть тряхнуть стакан, чтобы газ начал выходить.
Так вот, в литре колы содержится около пяти кубиков жидкой углекислоты, жидкая углекислота имеет плотность порядка 800 кг/м3, а газообразный углекислый газ при нормальном давлении уже около 2 кг/м3, т.е. единомоментно происходит увеличение объёма в сотни раз, что и создаёт реактивную тягу.

Автор: Vano 9.9.2015, 19:52

Т.е. можно и минералку с газом?

Автор: Kagamine Len 9.9.2015, 20:01

Цитата(HeatServ @ 9.9.2015, 19:28) *
Короче, моё видение летающей колы такое: пропан, который выпускают из баллона, имеет низкую температуру, при взбалтывании и перемешивании с колой, имеющей более высокую температуру, он резко расширяется, при этом выделяя достаточное количество теплоты для перехода углекислого газа из состава колы в газообразное состояние, а для этого нужно очень незначительная энергия, например, достаточно чуть тряхнуть стакан, чтобы газ начал выходить.
Так вот, в литре колы содержится около пяти кубиков жидкой углекислоты, жидкая углекислота имеет плотность порядка 800 кг/м3, а газообразный углекислый газ при нормальном давлении уже около 2 кг/м3, т.е. единомоментно происходит увеличение объёма в сотни раз, что и создаёт реактивную тягу.

у меня гипотеза , что после взбалтывания пропан расширяясь создает центры пониженного давления и вызывает выделение углекислого газа из жидкости. Сразу во многих местах.

Автор: HeatServ 9.9.2015, 20:16

Цитата(Vano @ 9.9.2015, 19:52) *
Т.е. можно и минералку с газом?
Да, любую жидкость с углекислотой, любую газировку.

Автор: HeatServ 9.9.2015, 20:53

На даче есть баллон туристический, завтра в баню париться поеду, попробую с минералкой такое провернуть.
Если что - прошу считать меня коммунистом.

Автор: and 9.9.2015, 21:09

Цитата(HeatServ @ 9.9.2015, 19:28) *
пропан, ... резко расширяется, при этом выделяя достаточное количество теплоты ...

Наоборот, поглощает большое количество теплоты
Цитата(Kagamine Len @ 9.9.2015, 20:01) *
пропан расширяясь создает центры пониженного давления

Он повышает давление, а не понижает.

Что-то у нас все доказательства от противного...

Автор: HeatServ 9.9.2015, 21:23

Цитата(and @ 9.9.2015, 21:09) *
Наоборот, поглощает большое количество теплоты

Что-то у нас все доказательства от противного...
Не понял, Вы не в курсе, что газ при расширении совершает работу? Или Вас испугало слово "теплота"? Теплота не выделяется, согласен, скорее это адиабатный процесс.

Автор: Kagamine Len 9.9.2015, 21:23

Цитата
Что-то у нас все доказательства от противного...

у вАС И ТАКИХ НЕТ ( ЩУТКА). дОВАЙТЕ СВОИ ВЕРСИИ.

Автор: Vano 9.9.2015, 21:47

Цитата(HeatServ @ 9.9.2015, 20:53) *
На даче есть баллон туристический, завтра в баню париться поеду, попробую с минералкой такое провернуть.
Если что - прошу считать меня коммунистом.

Я про минералку и спросил - она легче отстирывается - у самого руки чешутся.

Автор: HeatServ 9.9.2015, 21:49

Цитата(Vano @ 9.9.2015, 21:47) *
Я про минералку и спросил - она легче отстирывается - у самого руки чешутся.
Не, однозначно надо экспериментировать.

Автор: Vano 9.9.2015, 22:04

Там у этих ребят из Луганска анонс - то что вылетает из бутылки можно еще и поджечь.
Но это уже страшно старше 18 - надо посмотреть что там у них.

Автор: and 9.9.2015, 22:30

Цитата(Kagamine Len @ 9.9.2015, 21:23) *
дОВАЙТЕ СВОИ ВЕРСИИ.

Ладно, дОЮ.
Имеем бутылке 0,4кг воды (100г слили) температурой 20С.
Наливаем в бутылку 25г (50 см2) жидкого пропана. При нормальном давлении температура кипения пропана -42,1С, соответственно именно такую температуру он и имеет (за счёт испарения с поверхности).
Поскольку жидкий пропан вдвое легче воды, он налит на поверхность воды.
Вода для пропана жутко горячая (Ткипения + 60С). Но закипающий от контакта с водой пропан создаёт газовую прослойку, и теплопередача от воды жидкому пропану резко уменьшается (так капля воды довольно долго бегает по раскалённой сковородке).
Когда мы переворачиваем бутылку, вода оказывается наверху и стремится вниз, лёгкий пропан оказывается внизу и рвётся вверх, и обе жидкости интенсивно перемешиваются. Площадь поверхности контакта резко увеличивается. Часть теплоты воды расходуется на парообразование пропана.
0,025кг пропана*484,5кДж/кг=12кДж
При этом 0,4кг воды остынут на 12000/0,4кг/4,2=7,2К.
Вот именно тепловая энергия воды и является источником энергии для полёта.
0,025кг пропана = 6,25Литров газа (в поллитровой бутылке вместе с 0,4 кг воды).

Автор: Kagamine Len 9.9.2015, 22:39

Тогда летательный аппарат (бутылка) должен на обычной воде работать, как с этим обстоит ?

Автор: and 9.9.2015, 22:55

Я уже писал, у меня и Кока-Кола не полетела, но я с дочкой вышел уже по темноте (не утерпели). Я напшикал пропана на слух (по булькам), и видимо он весь испарился, а я в темноте не увидел, что нет в бутылке жидкой фазы (выше воды всё запотело).
Потом днём планируем пойти, и попробуем простую воду тоже.
Возможно с газировкой интенсивность перемешивания двух жидкостей будет выше, а может и нет. Ну добавит к 6 литрам пропана 1 л углекислого газа.

Автор: Kagamine Len 9.9.2015, 23:11

Очень интересно, какие будут результаты.

Автор: and 10.9.2015, 18:26

Цитата(Kagamine Len @ 9.9.2015, 22:39) *
Тогда летательный аппарат (бутылка) должен на обычной воде работать, как с этим обстоит ?

Сегодня сделали четыре попытки, пока газ не кончился, одна кола и три воды из-под крана. Две лучшие попытке - обе на воде. Т.е. нет разницы.
Советы: лить по центру в воду бесполезно, сразу кипит и испаряется. Нужно брызгать на стенки бутылки, там остаётся газовый снег.
Жидкой фазы на поверхности воды замечено не было, а снег на стенках - был. Когда переворачиваете бутылку, снег заливает водой.
Я использовать бутылки 0,5л, летают не так эффектно как большие. Может быть нужно было меньше воды, чтобы больше стенки под снег оставить.
.

Автор: nik4t 10.9.2015, 18:47

Круть!

Автор: Vano 10.9.2015, 19:42

Здорово. rolleyes.gif
Ждем дальнейших отчетов.
Как я понимаю Хит и товарищи предложат новую теорию.

Автор: HeatServ 10.9.2015, 21:54

Сегодня взял две по полтора литра минералки, три попытки, две неудачные, одна весьма неплоха, но на бутылке 0,5 литра. Походу, действительно, углекислота не при делах, резкое расширение происходит именно с пропаном. Вывод: больше пропанов, хороших и разных.
Хотел продолжить, но в баню было пора.
Надо попробовать с шариком, запихиваешь шарик в бутылку с жидкостью, "заливаешь" в шарик газ, переворот, протыкаем шарик спицей и поехали. Короче, нужны масштабные эксперименты.

Автор: and 10.9.2015, 22:07

Цитата(and @ 10.9.2015, 18:26) *
Я использовать бутылки 0,5л, летают не так эффектно как большие. Может быть нужно было меньше воды, чтобы больше стенки под снег оставить.

По просьбе дочки сгонял за новым баллончиком.
Попробовали уменьшить кол-во воды - сразу упал удельный импульс, не хватает рабочего тела.
Решили увеличить калибр. Размер имеет значение. 1,5 литра сразу улетели за забор ближайшей воинской части, чего я с самого начала старался избежать. Будет кому-то: "Дежурный! Почему мусор на плацу?!"
Ускорение такое, что в полёте с бутылки наклейку сорвало. На старте бутылка просто исчезает из кадра.
И да, всё на воде из-под крана. Импортозамещение, блин.
Цитата(HeatServ @ 10.9.2015, 21:54) *
Надо попробовать с шариком, запихиваешь шарик в бутылку с жидкостью, "заливаешь" в шарик газ, переворот, протыкаем шарик спицей и поехали. Короче, нужны масштабные эксперименты.

Я думал про резиновый напаличник... в аптеках продаётся. Подавать газ по трубке прям из баллончика пока напалечник не лопнет. Он маленький, в отличие от известного изделия.

Автор: Kagamine Len 10.9.2015, 22:17

Все таки пропан создает такое давление.

Автор: GraNiNi 10.9.2015, 23:33

Цитата(and @ 9.9.2015, 22:30) *
Ладно, дОЮ.
Имеем бутылке 0,4кг воды (100г слили) температурой 20С.
Наливаем в бутылку 25г (50 см2) жидкого пропана. При нормальном давлении температура кипения пропана -42,1С, соответственно именно такую температуру он и имеет (за счёт испарения с поверхности).
Поскольку жидкий пропан вдвое легче воды, он налит на поверхность воды.
....

Механизм описан верно, только там не пропан.

Пропан, при испарении из баллончика, не сможет самоохладиться до жидкого состояния (-42,1 С), так как для этого нужно охладить до этой температуры слой воды (колу), на которой он конденсируется. Но при этой температуре вода просто замерзнет и закупорит бутылку.

А вот бутан - может. Именно бутан и используется в таких баллончиках (или его смесь с пропаном 80/20). Но лучше использовать чистый бутан.

У бутана температура конденсации - около +0,5 С.
Бутан, выходя из баллончика, сначала испаряется и охлаждает только верхний слой жидкости (колы) почти до замерзания (0 С), а затем консенсируется и сам в виде жидкого слоя сверху, так как его плотность ниже, чем у воды.
При этом нижний слой жидкости в бутылке остается относитеольно теплым, так как за такое короткое время он не успел охладиться и это имеет всущественное значение.
Когда бутылку переворачивают, жидкий бутан пытается всплыть, но сразу же контактирует с более теплой жидкостью, что приводит к его резкому нагреву и мгновенному испарению - превращению в газообразное состояние, что и выталкивает воду из бутылки в виде реактивной струи.

Автор: and 11.9.2015, 7:59

Про пропан-бутановую смесь известно, но их свойства похожи (потому они плохо разделяются и при производстве попадают в одну фракцию). На ход корабля не влияет, ИМХО.

Нет в бутылке никакого жидкого газа. Предположение о слое жидкого газа на поверхности воды экспериментально не подтвердилось. При "заправке" бутылки, газ осаждается на стенках в виде "снежной шубы". Как максимум - мокрый снег. Соответственно он никуда не стекает, и никуда не всплывает. Он малоинтенсивно "парит". Для его активации (для обеспечения интенсивного испарения) газовый снег нужно размывать потоком воды. Если скорость потока будет маленькой, то отработавшая(остывшая) вода не успевает отойти от места теплообмена к соплу, и замерзает прямо на газовой шубе, замещая газовый снег обычным снегом. При этом уменьшается площадь поверхности контакта газовой шубы с жидкой водой.

При слишком спокойном переворачивании маленькой бутылки (0,5л) из-за малой высоты падения воды, скорость её потока может быть недостаточной, и двигатель "не запускается". Выроненная из рук бутылка падает на землю. Если бутылка падает вертикально и ударится о землю соплом, то вода, которая при падении бутылки приобрела вертикальную скорость, интенсивно размывает шубу, и бутылка стартует от земли.
В любом случае (обычный или ударный пуск двигателя), после начала работы двигателя бутылка движется со значительным ускорением, а это обеспечивает гравитационное разделение смеси (воды - к соплу, парообразного газа - к голове). При этом также обеспечивается высокая скорость движения воды вдоль газовой шубы.

Конструктивно не помешало бы иметь в бутылке в районе газовой шубы центральное тело, а так бОльшая часть воды вылетает по оси бутылки без контакта с шубой, и без отдачи тепла.
Также центральное тело увеличило бы объем газовой шубы (за счёт поверхности тела) и скорость движения воды вдоль шубы (за счёт уменьшенного сечения).

Автор: GraNiNi 11.9.2015, 11:14

Цитата
Про пропан-бутановую смесь известно, но их свойства похожи (потому они плохо разделяются и при производстве попадают в одну фракцию). На ход корабля не влияет, ИМХО.

Пропан и бутан производят отдельно.
Их специально смешивают в определенном соотношении для создания удобной в эксплуатации смеси, есть летние и зимние смеси.

На одном пропане вообще ничего не получится - он не конденсируется.

Цитата
Нет в бутылке никакого жидкого газа. Предположение о слое жидкого газа на поверхности воды экспериментально не подтвердилось. При "заправке" бутылки, газ осаждается на стенках в виде "снежной шубы".

Слоя конденсата может быть и не видно, так как при добавлении жидкого газа в бутылку, он интенсивно испаряется, смешивается с водой, которая при этом замерзает в виде снежной массы.
Получается смесь жидкого бутана с кристаллами снега (замерзшей воды) - "шуба".

Ну а дальше все также - при переворачивании бутылки теплая вода мгновенно нагревает шубу, освобождая бутан в виде газа.

Автор: and 11.9.2015, 19:01

А про жидкую фазу газа на воде мысль была правильной:
https://www.youtube.com/watch?v=CFzeRZP7oto&feature=iv&src_vid=VxvJcPToXb0&annotation_id=annotation_3047867839

Автор: Kagamine Len 11.9.2015, 19:27

Цитата(Kagamine Len @ 5.9.2015, 12:58) *
Пропан расширяется после нагрева от жидкости.

я первый отеетил

Автор: Vano 12.9.2015, 19:47

Луганские разместили новое видео - тоже пришли к просто воде, но они стали нагревать воду и у них виден слой пропана в бутылке.
http://www.youtube.com/watch?v=CFzeRZP7oto

Автор: and 13.9.2015, 8:16

Чувствуется масштаб... Мы тут с баллончиками для зажигалок и примусов, а эти просто литрами льют.
Дочка первый раз тоже горячую воду набрала, но в тот раз я газ по оси баллона наливал прямо на воду, он весь испарился. Потом, памятуя о разорванной бутылке, только холодную наливали.

Автор: and 13.9.2015, 8:58

Цитата(GraNiNi @ 10.9.2015, 23:33) *
Механизм описан верно, только там не пропан...А вот бутан - может. Именно бутан и используется в таких баллончиках (или его смесь с пропаном 80/20). Но лучше использовать чистый бутан.

Мерял температуру смеси при атмосферном давлении. Правда датчик очень инерционный. Плеснул газ в узкий стакан, температура рухнула до -30С (видимо от интенсивного испарения при заливе газа), и сразу стала подниматься. Процесс почти экспоненциальный, кривая приближалась примерно к -8С (возможно это характеризует доли пропана/бутана в моём баллончике), но не совсем экспоненциальный, кривая понемногу задиралась вверх. Видимо пропан выпаривался. Жаль не додумался скриншот снять. Потом прервал эксперимент (начал в комнате, но запах...).
Минут через 20 заглянул в стакан, осталось жидкости на дне. Температура -0,5С. Бутан или вода? Подождали ещё пару минут - в стакане сухо. Так что бутан.
Получать бутан выпариванием бОльшей части газа, конечно накладно. Но с ним не нужно торопиться на старте.
Так что возможно чистый бутан действительно лучше. Правда экспериментально подтвердить не могу.

Автор: GraNiNi 13.9.2015, 17:19

Цитата(and @ 13.9.2015, 8:58) *
Мерял температуру смеси при атмосферном давлении. Правда датчик очень инерционный. Плеснул газ в узкий стакан, температура рухнула до -30С (видимо от интенсивного испарения при заливе газа), и сразу стала подниматься.

Действительно, скорей всего там смесь пропана, бутана и еще (возможно) изобутана.

Более удобно сжиженный газ наливать в бутылки-ракеты через силиконовую трубочку, надев ее на клапан.
Вот здесь об этом и некоторых опытах с сжиженными газами.
http://chemistry-chemists.com/N3_2012/P3/ChemistryAndChemists_3_2012-P3-1.html

Автор: Машинист 14.9.2015, 15:28

Поставим вопрос иначе - поплывёт или не поплывёт? rolleyes.gif
https://www.youtube.com/watch?t=317&v=PolBtziCURs

Автор: HeatServ 14.9.2015, 16:09

Цитата(Машинист @ 14.9.2015, 15:28) *
Поставим вопрос иначе - поплывёт или не поплывёт? rolleyes.gif
https://www.youtube.com/watch?t=317&v=PolBtziCURs
Перегрели движки? Для летней езды вроде подготовку надо какую-то делать...
Ъ, это у нас чтоль?

Автор: Машинист 14.9.2015, 16:51

Цитата(HeatServ @ 14.9.2015, 16:09) *
Перегрели движки? Для летней езды вроде подготовку надо какую-то делать...
Ъ, это у нас чтоль?

Ну, один-то почти доплыл. Да, Театральное озеро похоже)))
А наши-то мужики до сих пор по старинке, по реке на лодках плавают, темнота laugh.gif
Ан нет! Показал коллеге, говорит, сульфатские тоже так умеют) такшта, не лаптем единым щи хлебает архангельский мужик wink.gif

Автор: HeatServ 14.9.2015, 17:12

Цитата(Машинист @ 14.9.2015, 16:51) *
Ну, один-то почти доплыл. Да, Театральное озеро похоже)))
"Театралы на Театральном" надо было назвать)) Утопили тетраэдр.

Автор: kdu 20.2.2016, 10:13

раз уж отошли от темы самолёта, вот вам ещё задачка...

 

Автор: and 20.2.2016, 11:21

Направо.
Сила натяжения левой нити нарушает равновесие.

Автор: HasBolla 20.2.2016, 12:59

Налево. Шарик пинг-понга добавляет массу, а железный просто висит. Систему можно рассмотреть как два пустых стакана в одном из которых (левом) доп. груз - шарик. Натяжение не влияет - это как Мюнхаузен себя за волосы из болота тянул smile.gif

Автор: coverart 20.2.2016, 14:46

Цитата(HasBolla @ 20.2.2016, 14:59) *
Налево. Шарик пинг-понга добавляет массу, а железный просто висит. Систему можно рассмотреть как два пустых стакана в одном из которых (левом) доп. груз - шарик. Натяжение не влияет - это как Мюнхаузен себя за волосы из болота тянул smile.gif


Согласен

Автор: дренаж 21.2.2016, 13:48

вес нитки привязывающей плавающий шарик со своим весом

создаст жуткий перевес и разрушит зыбкое равновесие

Автор: инж323 21.2.2016, 13:54

Никуда весы не сместятся, если пренебрегаем весом пингпонговского и нити, на которой висит он.

Автор: дренаж 21.2.2016, 13:59

? а если НЕпренебрегаем: "весом пингпонговского и нити, на которой висит он" ?

? тогда что ?

Автор: HeatServ 22.2.2016, 0:25

Решение простое, воды же одинаковый объём? Просто уберите воду из стаканов.

Автор: дренаж 17.11.2016, 2:11

аэросани и мы

 

Автор: nick2 18.11.2016, 15:23

взлетит или не взлетит?
Взлетит!
http://forum.abok.ru/index.php?s=&showtopic=29228&view=findpost&p=1252405

Автор: дренаж 30.12.2016, 15:56

декабрьские события видимо
тождественны 8-годовой давности

и модель тождественна самолёту
на наклонной плоскости
с изменяемым углом наклона

и вновь делаю вывод что в задачке
самолёт взлетает нарушая правила задачи

а правильная модель задачи в виде
самолёта стремящегося стоять на месте

Автор: Ashihara 3.11.2017, 12:05

Лень читать всё, но задача тривиальна. Самолёт отталкивается от воздуха, а не от взлётной полосы. Поэтому в общем случае ему наплевать, движется она или нет.

Будут, конечно, потери в подшипниках колёс, что затруднит разбег и в дальнейшем приведёт к превышению допустимой скорости в подшипниках и поверхности покрышек (т.е. разрушение стоек шасси), но задача не подразумевает учёта таких вещей.

Автор: HeatServ 4.11.2017, 0:27

Цитата(Ashihara @ 3.11.2017, 12:05) *
Лень читать всё, но задача тривиальна. Самолёт отталкивается от воздуха, а не от взлётной полосы. Поэтому в общем случае ему наплевать, движется она или нет.

Будут, конечно, потери в подшипниках колёс, что затруднит разбег и в дальнейшем приведёт к превышению допустимой скорости в подшипниках и поверхности покрышек (т.е. разрушение стоек шасси), но задача не подразумевает учёта таких вещей.
Ты просто пал перед очевидностью решения, как акционер МММ. А решения нет. В этом дело. Нет решения, как и в любой задаче по классической физике, потому как физика лишь предметная часть математики и пользуется её решениями.

Автор: Ashihara 7.11.2017, 12:21

Цитата(HeatServ @ 4.11.2017, 0:27) *
Ты просто пал перед очевидностью решения, как акционер МММ. А решения нет. В этом дело. Нет решения, как и в любой задаче по классической физике, потому как физика лишь предметная часть математики и пользуется её решениями.


Каковы ваши доказательства, сэр? tomato.gif

Автор: LordN 7.11.2017, 12:31

так гвоздями колесы к полосе прибиты и шурупьями пришуруплены. что там доказывать? smile.gif

Автор: HeatServ 7.11.2017, 12:52

Цитата(Ashihara @ 7.11.2017, 12:21) *
Каковы ваши доказательства, сэр? tomato.gif
Условия задачи являются неосуществимыми в реальной жизни, механизм такой невозможен, всегда будут запаздывания или опережения. А если рассматривать идеальную задачу, т.е. условиться, что механизм такой возможен, то недостаёт ещё кучи данных. Типичная задачка-мозгоклюйка.

Автор: испытатель 7.11.2017, 14:25

Если про безумную ситуацию про аэросани на конвеере, то как практик уверен - сгорит привод конвеера или лопнет норийная лента, а сани продолжат движение laugh.gif

Автор: Машинист 7.11.2017, 14:29

Цитата(испытатель @ 7.11.2017, 14:25) *
Если про безумную ситуацию про аэросани на конвеере, то как практик уверен - сгорит привод конвеера или лопнет норийная лента, а сани продолжат движение laugh.gif

Или в санях бензин кончится laugh.gif

Автор: экспорт 7.11.2017, 15:08

вместо ленты наклонную плоскость

и внезапно обнаружим нормально
нечто с работающим двигателем
стоящее на месте

и некоторые в реальности ...
не взлетели

список картинок темы
http://forum.abok.ru/index.php?act=attach&code=showtopic&tid=8956

Автор: Машинист 7.11.2017, 16:38

Цитата(экспорт @ 7.11.2017, 15:08) *
вместо ленты наклонную плоскость

С переменным углом наклона от нуля до 90... и опаньки! самолет превратился в ракету, но все-таки взлетел! Если мощи движков хватило преодолеть собственный вес.
А если не хватило - при достижении некоторого значения угла скатился по наклонной в преисподнюю, оглашая рёвом всё вокруг.

Автор: HeatServ 7.11.2017, 16:55

На самом деле идеально имитирует условия задачи тот же самолёт, но привязанный за хвост. И тут понятно, что взлёт или невзлёт зависит только от геометрии самолёта, мощности двигателя и тому подобных пердимоноклей. А решать чисто теоретическую задачу с крутящимися шасси, т.е. с начальными условиями, это только время тратить.

Автор: Машинист 7.11.2017, 17:07

Цитата
На самом деле идеально имитирует условия задачи тот же самолёт, но привязанный за хвост.

Собаку на цепи он имитирует.

А вот с наклонной плоскостью, мне кажется, ближе к теме. Стоит на горизонтальной - никуда не катится. Пошёл на разгон - плоскость наклоняем навстречу, так чтобы оставался на месте, и в пределе плоскость становится вертикальной, а самолет при растущей тяге двигателя больше не удерживается ничем. И взлетает. Вертикально вверх.
То же самое и с бегущей лентой, только граничная точка другая по уровню тяги. Не вес самолета, а трение. Трение наглухо заторможенных колес по полосе. Преодолеет его - взлетит. Пренебрегая ненужными здесь техническими подробностями в виде рваных/жженых покрышек и развалившихся/заклинивших подшипников - их отбрасываем.

Автор: HeatServ 7.11.2017, 17:11

Цитата(Машинист @ 7.11.2017, 17:07) *
Собаку на цепи он имитирует.
Именно. Т.е. неподвижен относительно земли. Как раз по условию задачи.

Автор: Машинист 7.11.2017, 17:23

Цитата(HeatServ @ 7.11.2017, 17:11) *
Именно. Т.е. неподвижен относительно земли. Как раз по условию задачи.

Но ты его приколачиваешь как бы. Просто смещая граничную точку, которая всё равно есть и всё равно не абстрактна, а вполне понятна и естественна - в твоём случае это будет прочность цепи. Порвёт цепь - взлетит.

Автор: HeatServ 7.11.2017, 17:29

Цитата(Машинист @ 7.11.2017, 17:23) *
Но ты его приколачиваешь как бы.
Условия задачи его приколачивают, т.е. делают неподвижным относительно земли. И какая нафиг тогда разница, крутятся колёса или нет?

Автор: Машинист 7.11.2017, 17:32

Цитата(HeatServ @ 7.11.2017, 17:29) *
Условия задачи его приколачивают, т.е. делают неподвижным относительно земли. И какая нафиг тогда разница, крутятся колёса или нет?

Важно понимать границу, условие этой неподвижности. В этом и соль. Поймёшь предел - решишь задачу.
С того самого момента, когда тяга двигателя становится достаточна, чтобы потащить самолёт волоком на заблокированных колёсах - становится "без разницы, крутятся они или нет". И величина эта вполне себе конкретна и определяема. Так что всё, задача решена. Тему можно закрывать)))
11 лет мусолили))))

Автор: HeatServ 7.11.2017, 17:37

Цитата(Машинист @ 7.11.2017, 17:32) *
Важно понимать границу, условие этой неподвижности. В этом и соль. Поймёшь предел - решишь задачу.
Можно привязать самолёт к дереву через магнитную муфту, потерялась земля, т.е. касание к земле - магнит отцепился, самолёт улетел.

Автор: Машинист 7.11.2017, 17:40

Цитата(HeatServ @ 7.11.2017, 17:37) *
Можно привязать самолёт к дереву через магнитную муфту, потерялась земля, т.е. касание к земле - магнит отцепился, самолёт улетел.

А вот это уже пустопорожняя болтология и трата времени.
Спасибо экспорту, за добрый пинок в направлении понимания задачи.
Я выхожу из дискуссии, поехал домой rolleyes.gif

Автор: Ashihara 8.11.2017, 13:18

Цитата(HeatServ @ 7.11.2017, 17:29) *
Условия задачи его приколачивают, т.е. делают неподвижным относительно земли. И какая нафиг тогда разница, крутятся колёса или нет?


Это крокодил от земли отталкивается, а не самоль. Самоль отталкивается от воздуха, а воздух неподвижен. На этом задача и построена.

Если в задаче заменить самоль на крокодила, то да, крокодил всегда будет стоять на месте.

Автор: HeatServ 8.11.2017, 13:58

Цитата(Ashihara @ 8.11.2017, 13:18) *
Это крокодил от земли отталкивается, а не самоль. Самоль отталкивается от воздуха, а воздух неподвижен. На этом задача и построена.
Чтобы самолёт оторвался от земли надо приложить к нему силу, у нас в деревне это называется подъёмная сила крыла, которую описывает теорема Жуковского. Создаётся она за счёт разности скоростей воздуха, который омывает крыло. В задаче по сути спрашивается следующее: сможет ли подъёмная сила крыла поднять привязанный за хвост самолёт в воздух, если она создаётся только за счёт работы его пропеллера.
А остальное - про шасси, про ленту, синхронизированную с шасси и прочий бред с наклонными плоскостями это болезненное восприятие физических процессов.

Автор: Ashihara 8.11.2017, 14:48

Цитата(HeatServ @ 8.11.2017, 13:58) *
Чтобы самолёт оторвался от земли надо приложить к нему силу, у нас в деревне это называется подъёмная сила крыла, которую описывает теорема Жуковского. Создаётся она за счёт разности скоростей воздуха, который омывает крыло. В задаче по сути спрашивается следующее: сможет ли подъёмная сила крыла поднять привязанный за хвост самолёт в воздух, если она создаётся только за счёт работы его пропеллера.
А остальное - про шасси, про ленту, синхронизированную с шасси и прочий бред с наклонными плоскостями это болезненное восприятие физических процессов.


Не вижу я этого в "дано" задачи.

Вижу только движение поверхности под колёсами. Для самоля это что дышло для закона, ему наплевать на поверхность эту, он не отталкивается от неё, а всего лишь катится по ней. И катится всё быстрее, т.к. движки отталкиваются от "несинхронизированного" воздуха. Чем и создаёт ДПС-Ж - Деревенскую Подъёмную Силу Жуковскогоsmile.gif

Вот в случае наклонной плоскости - это да, тут движкам придётся накапливать в массе самоля потенциальную энергию, что чревато падением с вертикальной плоскости при недостаточности тяговооружённости.


Автор: HeatServ 8.11.2017, 14:54

Цитата(Ashihara @ 8.11.2017, 14:48) *
Не вижу я этого в "дано" задачи.

Вижу только движение поверхности под колёсами. Для самоля это что дышло для закона, ему наплевать на поверхность эту, он не отталкивается от неё, а всего лишь катится по ней. И катится всё быстрее, т.к. движки отталкиваются от "несинхронизированного" воздуха. Чем и создаёт ДПС-Ж - Деревенскую Подъёмную Силу Жуковскогоsmile.gif

Вот в случае наклонной плоскости - это да, тут движкам придётся накапливать в массе самоля потенциальную энергию, что чревато падением с вертикальной плоскости при недостаточности тяговооружённости.
Т.е. даже понимания нет, что по условию задачи самолёт относительно земли неподвижен?

Автор: KYV86 8.11.2017, 15:09

Нет. Тут надо вырастить дерево на ковре-самолете. Привязать к нему за хвост самолет. Разогнать ковер до скорости взлета...., а затем смотреть, обгонит самолет ковер с деревом, или нет... biggrin.gif

Автор: HeatServ 8.11.2017, 15:22

Или вместо дерева использовать вертикальную штангу, по которой катается вверх-вниз блок.
А чтобы полотно не снилось никому, надо убрать полотно, а самолёт поставить на лыжы.
Я помню сам, как кукурузник зимой на встречном ветре с полтычка в воздух поднимается. Без всяких колёс.

Автор: Ashihara 8.11.2017, 15:29

Цитата(HeatServ @ 8.11.2017, 14:54) *
Т.е. даже понимания нет, что по условию задачи самолёт относительно земли неподвижен?


Да. Нет понимания, что он неподвижен. Объясните, пожалуйста, почему он должен быть неподвижен. На что расходуется энергия двигателей?

Автор: HeatServ 8.11.2017, 16:07

Цитата(Ashihara @ 8.11.2017, 15:29) *
Да. Нет понимания, что он неподвижен. Объясните, пожалуйста, почему он должен быть неподвижен. На что расходуется энергия двигателей?
На трение качения.

Автор: Ashihara 8.11.2017, 16:13

Цитата(HeatServ @ 8.11.2017, 16:07) *
На трение качения.



В первом же моём сообщении: "Будут, конечно, потери в подшипниках колёс, что затруднит разбег и в дальнейшем приведёт к превышению допустимой скорости в подшипниках и поверхности покрышек (т.е. разрушение стоек шасси), но задача не подразумевает учёта таких вещей".

Если представить какой-нибудь четырёхдвигательный мухолёт в виде АН-124 или Б-747, то разрушение стоек наступит задолго до того момента, как трение качения достигнет той же силы, что и тяга двигателей.

Поэтому моё мнение, что по духу задачи трение не нужно учитывать. Там же нет слов "пренебречь разрушением ВВП и шасси".

Автор: HeatServ 8.11.2017, 16:23

Ну что вы все уцепились за эти шасси? Задача ж элементарная, поднимет набегающий от лопастей воздух самолёт или не поднимет. В пределе, т.е. с фиксированной массой самолёта и ничем не ограниченной мощностью - поднимет.

Автор: Akela 8.11.2017, 18:30

Цитата(Ashihara @ 8.11.2017, 15:29) *
... Нет понимания, что он неподвижен. Объясните, пожалуйста, почему он должен быть неподвижен.


Это прямо следует из условия задачи. Постройте эпюру скоростей точек на ободе колеса для обычного самолета и для нашего случая, и увидите, что в данной задаче ось колеса неподвижна относительно земли.
Следовательно, самолет не наберет взлетную скорость = не взлетит.

Исключение - если это самолет вертикального взлета.

Автор: defuser 9.11.2017, 8:38

Полоса - это просто приспособа, чтобы крутить колесики. Давайте самолет краном подымем и колесики покрутим. Думаю, самолет вырвется из цепких лап крановщицы и улетит.

Автор: Ashihara 9.11.2017, 10:57

Цитата(Akela @ 8.11.2017, 18:30) *
Это прямо следует из условия задачи. Постройте эпюру скоростей точек на ободе колеса для обычного самолета и для нашего случая, и увидите, что в данной задаче ось колеса неподвижна относительно земли.
Следовательно, самолет не наберет взлетную скорость = не взлетит.


Всё верно, но только если самоль на авто заменить. Т.к. авто отталкивается колёсами от опоры-дороги, то он всё время будет неподвижен.

Но самоль-то не от дороги отталкивается. Ему вообще глубоко фиолетово, движется она или нет. Имхо, это прямо следует из условия задачи, которое составлено так, чтобы запутать человека до такой степени, чтобы человек забыл, что самоль не от дороги отталкивается, а от воздуха.

Правильно defuser написал. По вашей логике, если самолю покрутить колёсики, то он должен вертикально взлететь не сдвигаясь с места.


Я пытаюсь донести, что двигатели самолёта не колёса крутят. Это у авто двигатель крутит именно колеса и авто вполне корректно можно оставить стоять на месте.

Поэтому тут даже вторая нестыковка: нельзя двигать полосу назад, компенсируя вращение колёс, т.к. колёса у него вращаются свободно и любой сдвиг самолёса автоматом раскрутит его колёса и полосу до сверхсветовой скорости - тут уже полоса будет раскручивать колесо, а не наоборот, самолёт-то сам по себе двигается.

Если пренебречь трением и разрушением, конечно.

Аналогичные процессы будут и у авто, если его спустить с горки на нейтралке. Тут его разгонять будет сила тяжести, а не двигатель. Т.е. сила, приложенная НЕ к точке опоры.

Автор: HeatServ 9.11.2017, 11:03

Причём тут кран? Кран это уже другая задача. Неподвижен самолёт относительно земли, там специальное мифическое устройство установлено, которое линейную скорость полотна должно уравнивать с линейной скоростью на поверхности колеса (в физике называется тангециальная составляющая вектора скорости).

Автор: Ashihara 9.11.2017, 11:12

Цитата(HeatServ @ 9.11.2017, 11:03) *
Неподвижен самолёт относительно земли, там специальное мифическое устройство установлено, которое линейную скорость полотна должно уравнивать с линейной скоростью на поверхности колеса (в физике называется тангециальная составляющая вектора скорости).


Нет в условии неподвижности. Только мифическое устройство с чётко объяснёнными результатами своей работы: синхронизация колёс и полотна.

Про неподвижный самолёт это уже додумки. Собственно, я против этих додумок и выступаю)


PS. Кстати, скорость колёс и полотна у самолёта всегда одинаковы и без мифических устройств) Они проскальзывают относительно ВВП разве что в момент касания при посадке wink.gif

Автор: HeatServ 9.11.2017, 11:20

Просто волшебно. Не понять условия задачи и кинуться ее решать.

Автор: Ferdipendoz 9.11.2017, 12:13

Ну, кстати, да. Скорость поверхности полотна и линейная скорость поверхности колеса равны только при неподвижном самолёте (относительно устройства с полотном). Если самолёт будет двигаться навстречу полотну, линейная скорость поверхности колеса обязательно будет больше скорости полотна. А в условии их скорости уравниваются. Значит либо он будет стоять на месте и тяги будет хватать лишь на преодоление трения в подшипниках и покрышках, либо попрёт с проскальзыванием, либо ещё что-то

Автор: and 9.11.2017, 12:24

Цитата(HeatServ @ 9.11.2017, 11:03) *
.. там специальное мифическое устройство установлено, которое линейную скорость полотна должно уравнивать с линейной скоростью на поверхности колеса (в физике называется тангециальная составляющая вектора скорости).

Вариант мифического устройства уже 5 лет как http://forum.abok.ru/index.php?showtopic=8956&view=findpost&p=847149..
Оно выполняет требование по равенству скоростей подвижной полосы (конвейера) и колеса.
Таких устройств, которые гоняют конвейер под колёсами по какому угодно закону, можно придумать много. А самолёт во всех случаях взлетает как обычно, относительно неподвижного воздуха.

Автор: HeatServ 9.11.2017, 12:43

Цитата(and @ 9.11.2017, 12:24) *
Вариант мифического устройства уже 5 лет как http://forum.abok.ru/index.php?showtopic=8956&view=findpost&p=847149..
Хрень с колёсами Вы там показали.

Автор: Ashihara 9.11.2017, 12:44

Цитата(HeatServ @ 9.11.2017, 11:20) *
Просто волшебно. Не понять условия задачи и кинуться ее решать.



"Самолет (реактивный или винтовой) стоит на взлетной полосе с подвижным покрытием (типа транспортера). Покрытие может двигаться против направления взлета самолета, т.е. ему навстречу. Оно имеет систему управления, которая отслеживает и подстраивает скорость движения полотна таким образом, чтобы скорость вращения колес самолета была равна скорости движения полотна.
Внимание, вопрос: сможет ли самолет разбежаться по этому полотну и взлететь?"


Я читаю, что вижу. Вижу мифическое устройство, которое ничего не делает по своей сути, т.к. ВВП всегда движется под самолётом в "обратную сторону" (привет ОТО) и скорость колёс всегда равна скорости движения полотна. Дерева и верёвки не вижу, значит самоль не закреплён. А раз он не закреплён, то со стороны опоры его движению ничего, кроме силы трения, не препятствует.

Возвращаю вам вашу же подколку про непонимание условия задачи.

Автор: HeatServ 9.11.2017, 12:55

Цитата(Ashihara @ 9.11.2017, 12:44) *
"Самолет (реактивный или винтовой) стоит на взлетной полосе с подвижным покрытием (типа транспортера). Покрытие может двигаться против направления взлета самолета, т.е. ему навстречу. Оно имеет систему управления, которая отслеживает и подстраивает скорость движения полотна таким образом, чтобы скорость вращения колес самолета была равна скорости движения полотна.
Внимание, вопрос: сможет ли самолет разбежаться по этому полотну и взлететь?"


Я читаю, что вижу. Вижу мифическое устройство. Дерева и верёвки не вижу, значит самоль не закреплён. А раз он не закреплён, то со стороны опоры его движению ничего, кроме силы трения, не препятствует.

Возвращаю вам вашу же подколку про непонимание условия задачи.
Какое-то дикое непонимание, физика ж была в школе? Система управления всегда, абсолютно всегда, а это означает, что сегодня, вчера, завтра, послезавтра, с воскресенья на понедельник и даже в новый год уравнивает линейную скорость полотна (относительно земли) с тангенциальной составляющей скорости покрышки, т.е. точки касания колесо с полотном (тоже относительно земли). И даже через неделю будет подстраивать скорость. Т.е. самолёт относительно земли НЕ-ПОД-ВИ-ЖЕН.
В конце концов, если самостоятельно это понимание не даётся, можно посмотреть какой-нибудь псевдонаучпоп, тех же разрушителей мифов. Там они это дело хоть и корявенько, но воспроизвели.

Автор: and 9.11.2017, 12:56

Цитата(and @ 9.11.2017, 12:24) *
Таких устройств, которые гоняют конвейер под колёсами по какому угодно закону, можно придумать много.

Рассказываю идеальное мифическое устройство для нашей задачи:
Покрытие полосы - водяной лёд.
Материал колёс самолёта - фторопласт. Подшипники клиним.
Включай двигатель и лети хоть к чёртовой матери - все условия выполняются.
Про отсутствие проскальзывания в условии задачи ничего не сказано.
А если претензии к условиям задачи, в части возможности создания мифического устройства отпали, то решение - самолёт взлетает как обычно, независимо от скорости полосы.

Автор: HeatServ 9.11.2017, 12:58

Цитата(and @ 9.11.2017, 12:56) *
Рассказываю идеальное мифическое устройство для нашей задачи:
Покрытие полосы - водяной лёд.
Материал колёс самолёта - фторопласт. Подшипники клиним.
Включай двигатель и лети к хоть к чёртовой матери - все условия выполняются.
Почти победа. Только надо, чтобы самолёт относительно льда был неподвижен. И для этого его надо.... привязать за хвост. И тогда фторопласт там, лёд там, колёса там или резиновые сапоги уже неважно.

Автор: Ashihara 9.11.2017, 13:01

Цитата(HeatServ @ 9.11.2017, 12:58) *
Только надо, чтобы самолёт относительно льда был неподвижен.


Кому надо-то? Из чего это проистекает?


PS. По физике я на общегородских олимпиадах побеждал, даже грамоты какие-то есть, если в гараже порыться. Утешение слабое, но позволяет на что-то надеяться)))

Автор: HeatServ 9.11.2017, 13:06

Цитата(Ashihara @ 9.11.2017, 13:01) *
Кому надо-то? Из чего это проистекает?
Из условия задачи, если прочтение не даётся, то посмотрите какой-нибудь видос, где это наглядно отображено и не оспаривается.

Цитата(Ashihara @ 9.11.2017, 13:01) *
PS. По физике я на общегородских олимпиадах побеждал, даже грамоты какие-то есть, если в гараже порыться.
А лучше найдите грамоты и верните своему учителю физики. С извинениями.

Автор: Ashihara 9.11.2017, 13:16

Цитата(HeatServ @ 9.11.2017, 13:06) *
Из условия задачи, если прочтение не даётся, то посмотрите какой-нибудь видос, где это наглядно отображено и не оспаривается.


Аргументировано.

Пойду посмотрю видос о том, что нас похищают НЛО. Тоже наверняка сущая правда.

Автор: and 9.11.2017, 13:22

Цитата(HeatServ @ 9.11.2017, 12:58) *
Почти победа. Только надо, чтобы самолёт относительно льда был неподвижен. И для этого его надо.... привязать за хвост. И тогда фторопласт там, лёд там, колёса там или резиновые сапоги уже неважно.

Если вам хочется менять условия задачи по ходу решения, хотя бы потрудитесь их описать.
Для взлёта с ледяного поля никого привязывать за хвост не нужно.
Скорость вращения колёс - 0, скорость полотна - 0. всё выполняется.
Кстати, а вам есть что вернуть своему учителю физики, если что?

Автор: HeatServ 9.11.2017, 13:30

Цитата(and @ 9.11.2017, 13:22) *
Если вам хочется менять условия задачи по ходу решения, хотя бы потрудитесь их описать.
Для взлёта с ледяного поля никого привязывать за хвост не нужно.
Скорость вращения колёс - 0, скорость полотна - 0. всё выполняется.
Кстати, а вам есть что вернуть своему учителю физики, если что?

Да, он мне радиодеталей задарил два ящика. Могу вернуть. Один. Второй в схемах.

Автор: ceril 9.11.2017, 13:49

Взлетит если по типу вертолета. вертолет то взлетает, а за счет закрылков не влетит

Автор: экспорт 9.11.2017, 13:53

между прочим наклонная плоскость
показывает решение:

шарик на наклонной плоскости вверху
и шарик скатывается вниз с ускорением

? возможно ли задержать шарик на плоскости ?

в идеале должен быть шарик скатывающийся
вниз с наклонной плоскости и одновременно
шарик двигается вверх относительно воздуха

на малых скоростях по-моему реально

а повышенные скорости окажутся
противоречащими заданию

Автор: HeatServ 9.11.2017, 13:59

Короче говоря, условие некорректно, кто как хочет, тот так и читает, отсюда вариантов увиденного минимум дофига и смысла решать задачу нет. Разве только времени сильно много и совсем заняться нечем.

Автор: Akela 9.11.2017, 14:25

Цитата(Ashihara @ 9.11.2017, 13:01) *
... По физике я на общегородских олимпиадах побеждал, даже грамоты какие-то есть, ... позволяет на что-то надеяться


Это было давно и неправда.
Тут нужна не школьная физика, а теоретическая механика (1й или 2й курс института).
И не надо плодить и выдумывать лишние сущности ( про лед, колеса из фторопласта и прочее). Стройте эпюры скоростей и все станет ясно.

Автор: Ashihara 9.11.2017, 15:05

Цитата(Akela @ 9.11.2017, 14:25) *
Это было давно и неправда.
Тут нужна не школьная физика, а теоретическая механика (1й или 2й курс института).
И не надо плодить и выдумывать лишние сущности ( про лед, колеса из фторопласта и прочее). Стройте эпюры скоростей и все станет ясно.


На олимпиадах большинство задач идут с такими вот запутанными заданиями. Чтобы сбить с толку.

Так и тут. Первый посыл, сбивающий с толку, это параллель с авто. Чтобы решающий задачу человек "не вспомнил", что самолёт от опоры не отталкивается. Второй посыл, сбивающий с толку, это машина, синхронизирующая то, что априори не может быть рассинхронизировано.

Этих двух посылов достаточно, чтобы понять, что задача вырождается в обычный рядовой взлёт самолёта. Тут, сознаюсь, и я облажался, второй посыл не сразу раскусил. Возраст, наверное, да и теормех тоже палки в колёса вставил.

Обычная олимпиадная задача на смекалку, короче.

Автор: экспорт 9.11.2017, 16:20

нобелевская премия сама себя не получит

и смотрим картинку



где под влиянием полотна шарик двигается вправо
и самолёт олицетворяемый шариком не взлетит

хотя по мнению тех у кого самолёт взлетит
шарик двигается только под влиянием тяготения

главное обнаружить: взлетевшие нарушили условия

Автор: Ashihara 9.11.2017, 18:23

Цитата(экспорт @ 9.11.2017, 16:20) *
и смотрим картинку


Поясните, пожалуйста. Розово-лиловое - это что? И синее.

Остальное понятно.

Хотя нет. Не понятно. Красное полотно на второй картинке заканчивается?

Автор: экспорт 9.11.2017, 18:38

нобелевская премия сама себя не получит

красное полотно выдернули из-под шарика
и катящийся шарик двигался вспять

что контролируем падением шарика
например в лунку позади старта

и заодно картинка показывает как удобно
размещать картинки на своём сайте

Автор: and 10.11.2017, 2:08

Ну ладно, не хотите разрешать самолёту разбежаться, будем поднимать в воздух практически неподвижный самолёт. Не абсолютно неподвижный, некоторый стартовый сдвиг ему сделать удаётся (пусть пренебрежимо малый), иначе привод полосы навстречу самолёту не запустится.
Вот он сдвинулся, привод мифического устройства запустился, и полоса начала движение навстречу самолёту.
Теперь рассмотрим эту систему. Кому так удобнее, теперь могут мысленно привязать самолёт верёвкой за хвост - уже неважно.
Как уже было показано выше коллегами, полоса будет двигаться непрерывно разгоняясь. И да, ей удаётся удерживать самолёт практически на одном месте (при необходимости дальше можно будет показать, что это действительно возможно). Ускорение полосы нас не интересует, главное, что скорость растёт.
На поверхности полосы у нас лежит (двигаясь вместе с ней) пограничный слой воздуха. За счёт вязкого трения пограничный слой увлекает за собой вышележащие слои воздуха. Чем ниже слой воздуха (ближе к пограничному), тем ближе его скорость к скорости полосы. Т.е. у поверхности движущейся полосы образуется градиент скорости ветра (относительно неподвижной Земли). Чем выше, тем медленней.
Нас собственно интересует слой воздуха на уровне плоскостей (крыльев) самолёта. Скорость воздуха в этом слое ниже скорости движения полосы, однако растёт вместе с ней. По мере роста скорости обтекания этим потоком воздуха плоскостей неподвижного самолёта, растёт создаваемая этим потоком подъёмная сила.
И наконец, о чудо!, самолёт взлетает как воздушный змей (отрывается от полосы). Точка.
Теперь о том, как мифическому устройству удаётся удерживать самолёт практически на одном месте.
В задаче схематично описано аппаратное обеспечение мифического устройства как датчик скорости вращения колеса, датчик скорости полосы и управляемый привод полосы.
Как уже было показано выше коллегами, успешная реализация заданного алгоритма управления эквивалентна удержанию самолёта от смещения вдоль полосы от стартовой точки. В этом случае равенство скоростей колес и полосы обеспечивается автоматически.
Сомнения у коллег вызывала возможность как-то влиять на сам самолёт, воздействуя на полосу.
Вернёмся к нашим движущимся слоям воздуха, расположенным над пограничным слоем. Кроме создания подъемной силы, эти слои (по сути встречный ветер) также создадут приложенную к самолёту силу аэродинамического сопротивления, направленную против силы тяги двигателя, также приложенной к самолёту. В случае равенства этих сил самолёт будет удерживаться у стартовой точки.
Попытка взлёта начинается с того, что пилот начинает увеличивать силу тяги двигателя от нулевого значения.
Мифическому устройству, в рамках выполнения своей целевой функции, придётся в каждый момент времени обеспечивать такую скорость полотна, чтобы скорость создаваемого встречного самолёту ветра обеспечивала аэродиномическое сопротивление равное силе тяги двигателя в данный момент. Тогда самолёт не сдвигается вдоль полосы и необходимое условие выполняется.
Фактически получается, что скоростью полосы управляет пилот, оперируя рукояткой регулирования тяги двигателя.
Вышесказанное доказывает, что мифическое устройство, описанное в условии задачи, работоспособно и теоретически реализуемо.

Автор: timofeyprof 10.11.2017, 6:26

Один из вариантов ответа. Если самолет буржуйский, то не взлетит однозначно. Если самолет отечественный, то наши инженеры - самые лучшие в мире - строят самые лучшие самолеты в мире, и если партия/президент/Абрамович сказал "надо чтобы взлетел", взлетит вопреки и несмотря на.

Автор: экспорт 10.11.2017, 7:33

чтобы уйти от само обмана
про мощнейший самолёт
обязательно двигающийся вперёд

на примере катящегося шарика:

? взлетит ли самолёт скатывающийся ?

постоянная сила создающая ускорение
присутствует зато нет само обмана
про мощнейший самолёт

Автор: HeatServ 10.11.2017, 9:05

А самолёт взлетал, колёса тёрлися.
А мы не звали вас, а вы припёрлися.

Автор: экспорт 10.11.2017, 10:19




Автор: Ashihara 10.11.2017, 10:32

Цитата(экспорт @ 9.11.2017, 18:38) *
нобелевская премия сама себя не получит

красное полотно выдернули из-под шарика
и катящийся шарик двигался вспять

что контролируем падением шарика
например в лунку позади старта


Ну, полотно двигаем с постоянным ускорением, равным g, приведённым к углу наклона полотна. Трение качения не даёт шарику скатываться. Если ускорение полотна превышает расчёт, то шарик двигается вправо, если ускорение полотна меньше расчётного, то шарик скатывается влево.

Вопрос-то в чём? Что будет, когда полотно закончится? Тут вступит закон сохранения импульса. Если у шарика нулевая в этот момент будет горизонтальная скорость, то он упадёт строго вертикально вниз.



Цитата(HeatServ @ 10.11.2017, 9:05) *
А самолёт взлетал, колёса тёрлися.
А мы не звали вас, а вы припёрлися.


И рифмоплёты в теме пёрлися
Толи над нами, толь над собой.

Автор: Машинист 10.11.2017, 11:43

Пока впустую месит воздух самолёт,
С бегущей полосы взлететь не в силах,
В словесных взлётах, дюже ему милых,
Преуспевает псевдо-рифмоплёт.

Уж дуб посажен, цепь на дубе том,
Для важности к хвосту её привяжем,
Да песенку споём, да сказку скажем,
Прикинувшися сказочным котом.

Потом сколотим самолёту лыжи,
Потом всемА поссым на полосе,
Пусть будет лёд, пусть радуются все, -
Всёж не взлетает самолёт бесстыжий!

Катают шарик, радостно стебутся,
Достали школьных грамот вороха
Вдруг гул, и взрыв, и стала даль тиха...
Он улетел, но обещал вернуться.

smile.gif
10.11.2017

Автор: HeatServ 10.11.2017, 12:55

Браво. Посвящение Экзюпери, не иначе))

Автор: экспорт 10.11.2017, 13:29

даже только ради стихов стоило
начинать тему на всех форумах

а на предыдущей странице
наконец-то в слове "если"
намёк на решение разностороннее

нобелевская премия сама себя не получит

Автор: Машинист 10.11.2017, 13:42

Цитата(HeatServ @ 10.11.2017, 12:55) *
Браво. Посвящение Экзюпери, не иначе))

Всем, кто способен взлетать с бегущих навстречу полос, с цепями на хвосте и кратным перевесом неверующих в успех в толпе зевак.

Автор: Ashihara 10.11.2017, 14:17

Цитата(Машинист @ 10.11.2017, 13:42) *
Всем, кто способен взлетать с бегущих навстречу полос, с цепями на хвосте и кратным перевесом неверующих в успех в толпе зевак.


Моё почтение за стих smile.gif Достойно))

Автор: and 11.11.2017, 0:32

Стих великолепен!

Однако, коллеги, кто-нибудь укажет логические и иные ошибки в http://forum.abok.ru/index.php?showtopic=8956&view=findpost&p=1332779 о взлёте неподвижного самолёта за счёт набегающего встречного ветра, увлекаемого движением полосы? Выступит оппонентом, так сказать.. Или все переключились на анализ модели со сферическим конём?
Извиняюсь, что прозой.

Автор: HeatServ 11.11.2017, 1:50

Цитата(and @ 11.11.2017, 0:32) *
Стих великолепен!

Однако, коллеги, кто-нибудь укажет логические и иные ошибки в http://forum.abok.ru/index.php?showtopic=8956&view=findpost&p=1332779 о взлёте неподвижного самолёта за счёт набегающего встречного ветра, увлекаемого движением полосы? Выступит оппонентом, так сказать.. Или все переключились на анализ модели со сферическим конём?
Извиняюсь, что прозой.
Об этих простых вещах ещё лет 120 назад рассказано. Чего тут обсуждать.

Автор: Машинист 11.11.2017, 9:02

А я недавно решил, что взлетит при условии преодоления силы трения, которая скольжения, колёс по полосе. И пока не вижу оснований сомневаться. А что там нагонит на крыло двигатель, а тем более полоса, на мой взгляд это далеко вторично.

Автор: Kotlovoy 11.11.2017, 9:24

Цитата(Bers @ 3.11.2006, 7:20) *
скорость вращения колес самолета была равна скорости движения полотна.
........
Видел эту задачку на нескольких форумах и практически нигде не пришли к единому мнению.

А как можно приравнивать радианы в секунду к метрам в секунду? И давно ли научная истина выявляется голосованием?

Автор: Машинист 11.11.2017, 9:51

Цитата(Kotlovoy @ 11.11.2017, 9:24) *
А как можно приравнивать радианы в секунду к метрам в секунду? И давно ли научная истина выявляется голосованием?

Издревле. И опровергается сожжением на костре, как научных трудов так и их авторов.
А окружную-то скорость никто не запретить приравнять?

Автор: Kotlovoy 11.11.2017, 10:07

Цитата(Машинист @ 11.11.2017, 9:51) *
А окружную-то скорость никто не запретить приравнять?

Так задачи не формулируются. Окружную - так и напиши окружную. Строже надо, строже. Или теперь так и в учебниках-задачниках излагают? Как хочешь, так и понимай? Или разведи дебаты с голосованиями?
К тому же окружная скорость колеса и на обычном аэродроме равна скорости самолета. Или полотна, если за систему отсчета принять самолет. Если, конечно, исключить короткий момент юза при касании при посадке.

Автор: экспорт 11.11.2017, 10:30

хокку

не взлетит если
взлетит если
пишите медлееееннноооо

Автор: HeatServ 11.11.2017, 10:49

Цитата(Kotlovoy @ 11.11.2017, 9:24) *
А как можно приравнивать радианы в секунду к метрам в секунду? И давно ли научная истина выявляется голосованием?
Задача изначально американская, там обороты в секунду приравняны к метрам в секунду, километрам в час и, на всякий случай, к футам на метр на секунду в квадрате.

Автор: Машинист 11.11.2017, 11:37

Короче, вот вам лекция, и да спадут ваши цепи с ваших хвостов)))
 ___________4.pdf ( 4,06 мегабайт ) : 5

Смотрим на характер зависимости сопротивления качению от скорости движения.
Качение неизбежно перейдёт в скольжение, и тогда влияние встречного движения полосы на самолёт пропадает, появляется поступательная скорость, подъёмная сила и происходит взлёт.
Единственное условие - происходит всё это при диких значениях тяги, с точки зрения традиционного взлёта, но всё же далёких от каких-то бесконечностей и вполне определяемых расчетным путем.

Но мне же никто не ограничивал мощность двигателя, не задавал массу самолёта, количество колёс, давления в шинах. Поэтому принципиально, параметрически, лично я - взлетаю.

Русская версия Invision Power Board (http://nulled.ws)
© Invision Power Services (http://nulled.ws)